SlideShare ist ein Scribd-Unternehmen logo
1 von 243
Question 1: The Botanical Garden was a very _________ place where
we made a short tour round the garden and took some pictures of
the whole class.
A. beautify B. beautifully C. beautiful D. beautification
A. beautify /’bju:tifai/ (v): làm đẹp
B. beautifully /’bju:tifli/ (a): đẹp, hay
C. beautiful /’bju:tifl/ (a): đẹp, hay
D. beautification /bju:tifi’kei∫n/ (n): sự làm đẹp
Căn cứ vào cấu trúc câu vì vậy vị trí ô trống cần điền là một tính
từ để tạo thành một cụm danh từ.
Adj + N
Dịch nghĩa: Botanical Garden đã là một nơi rất đẹp, nơi mà chúng ta
thực hiện những tua ngắn quanh vườn và chụp nhiều bức ảnh của
toàn bộ lớp.
Question 2 :
She doesn’t imagine that I have much money, _______?
A. does she B. do I C. haven’t I D. doesn’t she
Dịch nghĩa: “Cô ấy không nghĩ rằng tôi có nhiều tiền phải không?”
Câu bắt đầu với she + doesn’t thì thành lập hỏi đuôi sẽ dùng does
she.
Question 3: After feeling off for days, Tom finally went to see his
doctor.
A. Food B. color C. fitness D. balance
Kiến thức về thành ngữ
feel off color = under the weather: không được khỏe
Tạm dịch: Sau vài ngày cảm thấy không khỏe, Tom cuối cùng
mới chịu đi bác sĩ.
.
Question 4: This policewoman is famous ____ her courage and
strong will.
A. with B. for C. on D. about
Dịch nghĩa: Nữ cảnh sát này nổi tiếng vì lòng can đảm và ý chí
mạnh mẽ.
=> Theo cấu trúc: be famous for: nổi tiếng vì cái gì
Question 5: There were 79 killed and 230 ________in a bomb
explosion at the embassy.
A. injured B. wounded C. hurt D. ached
Đáp án B
Kiến thức: về từ cùng trường nghĩa
A. injured: bị thương thể chất do tai nạn giao thông, làm tổn
thương đến cảm xúc cà tinh thân
B. wounded: bị thương ngoài da do vũ khí trong chiến tranh hay
bom đạn, bị xúc phạm
C. hurt : bị tổn thương về thể xác
D. ached: bị đau đau một bộ phận trên cơ thể do sử dụng quá nhiều
đến nó)
Tạm dịch: Có 79 người tử vọng và 230 người bị thương trong vụ nổ
bom tại đại sứ quán.
Question 6: ____ many international agreements affirming their
human rights, women are still much more likely than men to be poor
and illiterate.
A. Although B. Despite C. Because D. Since
Dịch nghĩa: Mặc dù nhiều xác nhận các thỏa thuận quốc tế quyền
con người của họ, nữ giới vẫn nhiều hơn nam giới về số lượng
nghèo và mù chữ.
Xét các đáp án:
A. Although + clause: mặc dù
B. Despite + N/Ving: mặc dù
C. Because + clause: bởi vì
D. Since = because + clause: bởi vì
*Note: “affirming” là danh động từ, có chức năng là một danh từ
chứ không phải Ving, cùng với “agreements” tạo thành cụm danh
từ, và cả cụm “many….rights” là một cụm danh từ hoàn thiện, do đó
ta chọn B.
Question 7: Lara goes to a (n) _____________ temple every week.
A. great big ancient Buddhist B. Buddhist great big ancient
C. ancient great big Buddhist D. big great ancient Buddhist
*Theo quy tắc trật tự tính từ trong câu: OSASCOMP
: great - Opinion; big - Size; ancient -A
*Note: Buddhist temple (phr.n): ngôi đền tín đồ đạo Phật
Tạm dịch: Lara tới ngôi đền tín đồ Phật cổ rất to và đẹp mỗi tuần.
Question 8: The meeting appeared to be going well until Sarah
______________ the issue of holiday pay.
A. brought about B. brought up C. brought off D. brought on
Đáp án B
A. brought about: đem lại, gây ra
B. brought up: nuôi dưỡng, đề cập đến
C. brought off: giải quyết việc khó
D. brought on: dẫn đến, gây ra
+ bring sb on = help someone to improve, especially through
training or practice
+ bring on something = to cause something to happen, esp.
something bad
Dịch nghĩa: Cuộc họp dường như đã diễn ra tốt đẹp cho đến khi
Sarah đề cập đến vấn đề trợ cấp ngày nghỉ.
Question 9: When I came to her house last night, she _____ dinner.
A. is having B. was having C. has had D. had had
Đáp án B
– Căn cứ vào đây là mệnh đề trạng ngữ chỉ thời gian:
When S + V (quá khứ đơn), S + V (quá khứ tiếp diễn). ( Diễn tả một
hành động đang xảy ra thì có hành động khác xen vào).
Vế chứa “when” chia thì quá khứ đơn => Vế còn lại chia thì quá khứ
tiếp diễn.
– Dịch: Khi tôi đến nhà cô ấy tối qua, cô ấy đang ăn tối.
Question 10. _______, I will give her a gift.
A. When he will come B. When she comes here
C. Until he will come D. No sooner she cames
10. Đáp án B
Kiến thức: Mệnh đề chỉ thời gian / Sự hòa hợp thì trong mệnh đề
Giải thích:
Mệnh đề chính chia thì tương lai => động từ trong mệnh đề chỉ thời
gian chia thì hiện tại (hiện tại đơn, hiện tại hoàn thành).
When +S+Vo/s/es +…, S+ will + Vo.
Loại đáp án A,C, sai thì, và đáp D chưa đúng nghĩa .
Question 11: We have been working hard. Let’s a break.
A. make B. find C. do D. take
Question 11. Chọn đáp án D
Giải thích: take a break: nghỉ ngơi.
Dịch nghĩa: Chúng ta đã làm việc thật chăm chỉ. Nghỉ ngơi thôi!
Question 12: It is parents' duty and responsibility to ______
hands to take care of their children and give them a happy home.
A. give B. hold C. join D. shake
Câu 12: Đáp án C
Kiến thức về cụm từ cố định
give/ lend sb a hand = do sb a favor = help sb: giúp ai
hold hands: nắm tay
join hands = work together: chung tay, cùng nhau
shake hands: bắt tay
Question 13. Harry Potter and the Goblet of Fire ________ by J K
Rowling.
A. written B. was written C. wrote D. were written
13 (TH)
Kiến thức: Bị động thì quá khứ đơn
Giải thích: “Harry Potter and the Goblet of Fire”: chủ ngữ chỉ sự vật
– nên câu ở trạng thái bị động của động từ “write”
Cấu trúc câu bị động thì quá khứ đơn: S + was/ were + Vp2 (by +
O).
Tạm dịch: Harry Potter và the Goblet of Fire được viết bởi J K
Rowling
Question 14. ________ his composition, Louie handed it to his
teacher.
A. Having written B. Having been written
C. Being written D. Writing
Kiến thức: Mệnh đề phân tử / Rút gọn mệnh đề đồng ngữ
Giải thích:
Khi 2 mệnh đề có cùng chủ ngữ (she) thì có thể rút gọn 1 trong 2
mệnh đề về dạng:
- V-ing / Having P2: nếu mệnh đề được rút gọn mang nghĩa chủ
động
- P2 (quá khứ phân từ): nếu mệnh đề được rút gọn mang nghĩa bị
động
Chủ ngữ “He” có thể làm chủ (tự thực hiện hành động “write”
(viết) => nghĩa chủ động.
Tạm dịch: Sau khi viết xong bài của mình, Louie giao nó cho giáo
viên của mình.
Question 15: The faster we walk, ____________ we will get there.
A. the soonestB. the soon C. the more soon D. the sooner
15.Đáp án D
HD: Dịch nghĩa: Chúng ta đi bộ càng nhanh thì chúng ta sẽ đến đó
càng sớm.
Xét các đáp án:
A. the soonest→ Đây là cấu trúc so sánh kép nên chỗ trống cần
điền từ có dạng so sánh hơn
B. the soon → Đây là cấu trúc so sánh kép nên chỗ trống cần điền từ
có dạng so sánh hơn
C. the more soon → soon có 1 âm tiết nên ta sử dụng cấu trúc:
adj/adv + đuôi -er + than
D. the sooner → Cấu trúc so sánh kép khi nói về 2 người hoặc sự
vật:
The more/-er + S + V, the more/-er + S + V
Question 16: "Why do you like pop music?” - “_______.”
A. Pop music originated in the Western world.
B. Because it is soft and beautiful.
C. Westlife is a pop band.
D. Since the 1950s, pop has had some new elements.
Câu 16
Đáp án B
“Tại sao bạn thích nhạc pop?” -"____________’’
A. Nhạc POP có nguồn gốc từ những nước phương Tây.
B. Bởi vì nó nhẹ nhàng và hay.
C. Westlife là một nhóm nhạc pop.
D. Từ những năm 1950, nhạc pop đã có nhiều yếu tố mới.
Question 17: “What’s your neighbourhood like?” - “_______."
A. I live in Ha Noi. B. Sounds good. I like living there.
C. It’s good. I love it. D. No, there is no hospital there.
Câu 17
Đáp án C
Tạm dịch: “Khu phố của bạn thế nào?”
A. Tôi sống ở Hà Nội.
B. Nghe tốt đấy. Tôi thích sống ở đây.
C. Tốt lắm. Tôi yêu nó.
D. Không, không có bệnh viện ở đây
Question 18: A. occurB. prefer C. apply D. surface
Từ “surface” có trọng âm rơi vào âm tiết thứ nhất Các từ còn lại có
trọng âm rơi vào âm tiết thứ hai
Question 19:
A. domination
B. adventure
C. grammatical
D. emergency
Từ “domination” có trọng âm rơi vào âm tiết thứ 3 Các từ còn lại
có trọng âm rơi vào âm tiết thứ 2
Question 20:
A. mended
B. faced
C. objected
D. waited
Từ “faced” có phần gạch chân phát âm là /t/ Các từ còn lại có
phần gạch chân phát âm là /id/
Question 21:
A. family
B. famine
C. famous
D. fabricate
Từ “famous” có phần gạch chân phát âm là /ei/ Các từ còn lại có
phần gạch chân phát âm là /æ
Question 22: Life expectancy for both men and women has
improved greatly in the past twenty years.
A. Living standard B. Longevity C. Life Skills D. Lifeline
Từ đồng nghĩa - Kiến thức về từ vựng
Tạm dịch: Tuổi thọ của cả nam lẫn nữ đều đã tăng đáng kể trong
vòng 20 năm qua.
=> Life expactancy: tuổi thọ
Xét các đáp án:
A. Living standard: tiêu chuẩn sống B. Longevity: tuổi thọ
C. Life skills: kĩ năng sống D. Lifeline: dây an toàn, dây cứu đắm
Question 23: Young college and university students participate in
helping the poor who have suffered badly in wars or natural
disasters.
A. get involved in B. interfere with C. join hands D. come into
Từ đồng nghĩa
Tạm dịch: Những sinh viên cao đẳng, đại học tham gia giúp đỡ những
người nghèo, người chịu ảnh hưởng nặng nề từ chiến tranh và thiên
tai.
=> Participate in: tham gia
Xét các đáp án:
A. get involved in: tham gia B. interfere with: cản trở
C. join hands: giúp đỡ D. come into: thừa kế
Question 24: As a brilliant and mature studentMarie harboured
the dream of a scientific career which was impossible for a woman
at that time.
A. Kept B. had C. abandoned D. deleted
Câu 24: Đáp án C
Từ trái nghĩa
Tạm dịch: Là một sinh xuất sắc và chín chắn, Marie ấp ủ giấc mơ sự
nghiệp khoa học, cái mà là không
thể cho một người phụ nữ vào thời điểm đó.
=> harbour the dream of st: ấp ủ/nuôii dưỡng giấc mơ gì
=> Đáp án C (harbor >< abandon: từ bỏ)
Question 25: After the tragic death of Pierre Curie in 1906, she not
only took charge of educating her two children but also took the
position which her husband had finally obtained at the Sorbonne.
A. brilliant B. lovely C. fascinating D. happy
Từ trái nghĩa
Tạm dịch: Sau cái chết bi thương của ông Pierre Currie vào năm
1906, bà không chỉ đảm nhận trách
nhiệm dạy dỗ hai con mà còn kế tục sự nghiệp của chồng ở
Sorbonne.
=> tragic /'trædʒɪk/ (a): bi kịch, bi thương
brilliant /’briljənt/ (a): tài giỏi, lỗi lạc
lovely /'lʌvli/ (a): đẹp đẽ, xinh xắn
fascinating /'fæsineitiη/ (a): hấp dẫn, lôi cuốn, quyến rũ
happy (a): hạnh phúc
=> Đáp án D (tragic >< happy
Question26: It was not until the sun was shining brightly that the little girl
woke up.
A. No sooner was the sun shining brightly than the little girl woke up.
B. Not until the little girl woke up was the sun shining brightly.
C. Not until the sun was shining brightly did the little girl wake up.
D. As soon as the little girl woke up, the sun hasn’t shone brightly yet.
Đảo ngữ Not until +s+ V2/ed did+S+V0
Dịch nghĩa: Mãi đến khi mặt trời chiếu sáng rực rỡ, cô bé mới tỉnh dậy.
Xét các đáp án:
A. No sooner was the sun shining brightly than the little girl woke up. → Mặt trời
vừa chiếu sáng rực rỡ thì cô bé tỉnh dậy, sai nghĩa.
B. Not until the little girl woke up was the sun shining brightly. → Mãi đến khi cô bé
tỉnh dậy thì mặt trời mới chiếu sáng rực rỡ, sai nghĩa.
C. Not until the sun was shining brightly did the little girl wake up. → Mãi đến khi
mặt trời chiếu sáng rực rỡ, cô bé mới tỉnh dậy, đúng nghĩa.
D. As soon as the little girl woke up, the sun hasn’t shone brightly yet. → Ngaykhi
cô bé tỉnh dậy, mặt trời đã tỏa sáng rực rỡ, sai nghĩa.
Question27: You must read the instructions. You won’t know how to use this
machine without reading them.
A. Unless you read the instructions, you won’t know how to use this machine.
B. You will know how to use this machine unless you read the instructions.
C. Without reading the instructions, the use of this machine won’t be known.
D. Reading the instructions, so you will know how to use this machine.
Kiến th ức: Câu điều kiệ n loại 1
Giải thích:
Câu điều kiện loại 1 diễn tả sự việc có thể x ả y ra ở hiện tại hoặc tương lai.
Cấu trúc: If/ Unless + S + V (thì hiện tại đơn), S + will/ can + V
Tạm dị ch: Bạn phải đọc hướng dẫn. Bạn sẽ không biết làm thế nào để sử d ụng
máy này mà không đọc chúng.
A. Nếu bạn không đọc các hướng dẫn, b ạn sẽ không biết làm thế nào để sử dụng
máy này. => đúng
B. Bạn sẽ biết cách sử dụ ng máy này nếu bạn khô ng đọc hướng dẫn. => sai nghĩa
C. Nếu không đọc hướng dẫn, việc sử dụng máy này s ẽ không được biết. => sai
nghĩa
D. Đọc các hướng d ẫn, vì vậ y b ạn sẽ biết cách sử dụng máy này. => sai nghĩa
Question 28. I visit Mexico and the United States last year.
A. visit B. Mexico C. United States D. and
Trong câu có trạng ngữ last year là thì quá khứ đơn, do vậy động
từ phải chia “ visited”
Tạm dịch. Tôi đã đến thăm Mexico và Hoa Kỳ vào năm ngoái.
Tạm dịch. Mẹ của Minh thích gặp trực tiếp bạn bè của bà ấy hơn là
gọi điện thoại.
Question 29: We are having her house painted by a construction
company.
A B C D
Question 29: Đáp án B
Nhận biết lỗi sai : Kiến thức về sự hoà hợp giữa chủ ngữ và tính
từ sở hữu cách
Phải sửa: her → our
Question 30: She is becoming increasingly forgetable. Last night
she locked herself out of the house twice.
Nhận biết lỗi sai : Kiến thức về dùng từ đúng ngữ cảnh
Forgetable ( a) có thể quên thay bằng Forgetful (a) :Hay
quên
Question 31: It started to rain at 4 o’clock and it is still raining.
A. It has been raining at 4 o’clock.
B. It has been raining since 4 o’clock
C. It has been raining for 4 o’clock.
D. It has been raining in 4 o’clock.
Question 31: B
Đề: Trời đã bắt đầu mưa lúc 2 giờ và vẫn đang mưa.
Viết lại câu dùng thì HTHTTD. Dùng “since” trước một mốc thời
gian → chọn B
Dịch: Trời mưa suốt từ 4 giờ.
Question 32: “Mum, please don’t tell Dad my mistake!” the girl
said.
A. The girl insisted her mother not tell her father his mistake.
B. The girl told her mother not to mention her mistake any more.
C. The girl asked her mother not to tell her father her mistake.
D. The girl wanted her mother to keep her mistake in her heart.
Question 32: C
Kiến thức về cảu trực tiếp
Ask Sb to + V/ not + to +V +st: đề nghị ai đó làm gì
Question 33: You are able to go out with your friend this evening
A. You musn’t go out with your friend this evening.
B. You should go out with your friend this evening.
C. You needn’t go out with your friend this evening
D. You can go out with your friend this evening.
Question 33: D
Kiến thức modal verb : To be able to + V = Can + V + St
Musn’t + V + St = are not allowed to + V
Should + V + St:
Needn’t + V + St = don’t/ doesn’t have + V +St
Question 34.
Not (34) _____________ body recognises the benefits of new
developments in communications technology.
A. nobody B. others C. every D. all
Everybody
Question 35.
...So widespread has texting become, however, that even pigeons
have started doing it.____________, in this case, it’s difficult to
view the results as anything but positive:
A. Therefore B. What’s more C.whereas D. That is
Xét về nghĩa thì “What’s more” phù hợp.
A. Therefore (adv): vì vậy,do đó
B. What’s more (adv): Quan trọng hơn
C. Whereas (conj): trong khi đó
D. That is - mệnh đề này thiếu, không có nghĩa.
Tuy nhiên nhắn tin trở nên quá phổ biến đến nỗi thậm chí những con
bồ câu đang bắt đầu nhắn tin. ____________, trong trường hợp này,
rất khó để xem xét các kết quả không có gì khác là tích cực.
Cần dùng một trạng từ nối câu vì chỗ trống nằm giữa một dấu chấm
và một dấu phấy.
Question 36
...each____________with sensor equipment and a mobile phone
A. armed: được trang bị (to be armed with sth)
B. loaded: được nạp, được chất lên (to be loaded with sth)
C. granted: được trợ cấp,được ban cho
D. stocked:dự trữ, chất kho
mỗi____________với thiết bị cảm ứng
và một chiếc điện thoại di động. Dùng mệnh đề phân từ nối hai
câu khác chủ ngữ. Trong trường hợp này cả 4 phương án đều là
phân từ 2 nên chỉ cần xét nghĩa. Đáp án A phù hợp.
Question 37
….The readings made by the sensors will be automatically converted
into text messages and beamed to the Internet____________ -
they will appear on a dedicated “pigeon blog
A. when (adv): khi mà
B. which (pron): cái mà
C. where (adv): nơi mà
D. what (pron): cái mà
”: Những số liệu thu từ máy cảm ứng sẽ tự động đổi thành tin nhắn
và phát vào Internet ______________chúng sẽ xuất hiện ở mục nhật
kí bồ câu.
Câu sau chỗ trống không thiếu thành phần câu nào nên chỉ có thể
thêm một trạng ngữ bắt đầu mệnh đề quan hệ. Chọn where vì
Internet là không gian mạng, không dùng để chỉ thời gian.
Question 38
Each bird will carry these a miniature backpack, (37) _____________
, that is, from the camera, which will hang around its neck.
A. instead B. except C. apart D. besides
A. instead of: thay vào đó
B. except for: ngoại trừ
C. apart from: ngoài ra/ bên cạnh
D. besides: bên cạnh đó
Giới từ đề bài sẵn cho là “from” → chọn C và tạm dịch câu thành:
Mỗi chú chim sẽ mang những dụng cụ này trong một chiếc balo
siêu nhỏ được đeo ở cổ bên cạnh chiếc camera.
Question39. what is the main topic of this passage?
A. The benefits of solar and wind power over conventional energy sources.
B. Two types of alternative energy sources that should be further utilized.
C. How energy resources are tapped from nature.
D. Examples of the use of energy sources worldwide.
Câu hỏi: Nội dung/ Chủ để chính của bài là gì?
A. Những ích lợi vượt trội hơn của năng lượng mặttrời và năng lượng gió so với các
nguồn năng lượng truyền thống.
B. Hai nguồn năng lượng sạch nên được khai thác nhiều hơn nữa.
C. Cách mà những nguồn năng lượng được lấy ra từ tự nhiên.
D. Các ví dụ về cách sử dụng các nguồn năng lượng trên khắp thế giới.
Nhìn vào cấu trúc bài viết, nội dung bài xoay quanh việc phân tích ích lợi khi sử dụng 2
nguồn năng lượng sạch và cách thức khai thác chúng. Đáp án B.
Không chọn A do trong bài không nêu rõ việc so sánh này.
Không chọn C do đây chỉ là một ý của bài mà thôi.
Không chọn D do bài chỉ nêu ra ít ví dụ về khai thác năng lượng mặt trời và địa nhiệt.
Chú ý: Với câu hỏi chủ đề chính của bài, đáp án đúng phải nếu được nội dung xuyên
suốt của toàn bài, các phương án chỉ nêu một ý nhỏ hay một ý nào đó xuất hiện ở
từng đoạn đơn lẻ không phải là đáp án đúng. Có thể để lại câu hỏi này làm sau
cùng, khi đã đọc bài nhiều lần và hiểu rõ nội dung của bài.
Question 40. which of the following words could best replace the
word “abundant”?
A. a lot B. scarce C. little D. enough
Câu hỏi: Từ nào đồng nghĩa/ có thể thay thế tốt nhất từ “abundant”?
A. a lot: nhiều
B. scarce: hiếm
C. little: quá it
D. enough: đủ
Đáp án A. abundant: dồi dào, phong phú = a lot: nhiều.
Chú ý: đây là câu hỏi từ vựng, nếu biết nghĩa của từ đang được hỏi sẽ rất thuận lợi,
nhưng dù có biết hay không thì luôn phải đặt từ cần tìm vào câu và ngữ cảnh của
nó để dùng thông tin luận ra nghĩa.
“Solar energy, which reaches the earth through sunlight, is so abundant that it
could meet the needs of worldwide energy consumption 6,000 times over.”
Năng lượng mặt trời, nguồn năng lượng mà tới trái đất thông qua ánh nắng, là
nguồn năng lượng cực dồi dào đến mức mà nó có thể đáp ứng được nhu cầu sử
dụng năng lượng của toàn thế giới gấp tận 6000 lần.
Tips: Cũng có thể thay lần lượt từng phương án lên thay cho từ cần tìm nghĩa để
phán đoán.
Question41. The word “it” in the second paragraph refers to_____________
A. solar energy B. the earth C. sunlight D. energy consumption
Question41. Đáp án A
Câu hỏi. Từ “it” nằm ở đoạn 2 thay thế cho từ nào?
Các phương án khác.
A. năng lượng mặt trời
B. trái đất
C. ánh nắng
D. mức tiêu thụ năng lượng.
Đây là câu hỏi suy luận về đại từ dùng để thay thế cho danh từ đã xuất hiện phía
trước nhưng không muốn nhắc lại danh từ đó. Ta xem lại câu chứa từ “it”:
Solar energy, which reaches the earth through sunlight, is so abundant that it could
meet the needs of worldwide energy consumption 6,000 times over. Năng lượng
mặt trời, nguồn năng lượng mà tới trái đất thông qua ánh nắng, là nguồn năng
lượng cực dồi dào đến mức mà nó có thể đáp ứng được nhu cầu sử
dụng năng lượng của toàn thế giới gấp tận 6000 lần.
“Nó” ở đây là năng lượng mặt trời,
Question 42. According to the passage, how is solar energy
production similar to geothermal energy production?
A. They both require the use of a generator.
B. They both use heat from the earth’s surface.
C. They both require fairly simple technology.
D. They are both conventional and costly.
Câu hỏi: Theo như bài viết, cách tạo ra năng lượng mặt trời giống
với cách tạo ra năng lượng địa nhiệt như thế nào?
A. Chúng đều cần sử dụng máy phát điện.
B. Chúng đều sử dụng nhiệt từ bề mặt trái đất.
C. Chúng đều cần công nghệ khá đơn giản
D. Chúng đều là năng lượng thông thường và khá đắt đỏ.
Thông tin chúng ta tìm ở đoạn 3 dòng 5 và 6. “... and as with solar
power, the technology needed to utilize geothermal energy is fairly
simple.”: và cũng như năng lượng mặt trời, công nghệ cần có để
khai thác năng lượng địa nhiệt là khá đơn giản, phù hợp nhất với
đáp án C.
Các phương án khác không đúng theo như ý của bài.
Question43. what best describes the author’s purpose in writing the passage?
A. To warn people about the hazards of fossil fuel use.
B. To convince people of the benefits of developing alternative energy
sources.
C. To describe the advantages and disadvantages of alternative energy use.
D. To outline the problems and solutions connected with global warming.
Câu hỏi. Câu nào miêu tả mục đích của tác giả trong bài viếtnày?
A. Để cảnh báo con người về mối nguy hại của việc sử dụng năng lượng hóa thạch.
B. Để thuyết phục con người về những lợi ích của các nguồn năng lượng thay thế
đang phát triển.
C. Để nói vể ưu điểm và nhược điểm của việc sử dụng nguồn năng lượng thay thế.
D. Để kể ra các vấn đề và các giải pháp liên quan đến việc nóng lên của toàn cầu.
Rõ ràng ta thấy bài văn tập trung nói về ích lợi của 2 nguồn năng lượng sạch vừa dồi
dào, bất tận vừa thân thiện môi trường. Phần kết của bài tác giả cũng khẳng định lại
những ưu điểm của năng lượng sạch vượt trội hơn so với năng lượng thông thường
và nhấn mạnh chúng ta cần đầu tư hơn nữa vào sự phát triển những nguồn năng
lượng này trên toàn cầu → Đáp án B.
Question 44. what does the passage mainly discuss?
A. How leadership differs in small and large groups
B. The role of leaders in social groups
C. The problems faced by leaders
D. How social groups determine who will lead them
Câu hỏi: Bài viết tập trung thảo luận vấn đề gì?
A. Cách thức mà sự lãnh đạo khác nhau trong các nhóm nhỏ và
nhóm lớn.
B. Vai trò của người lãnh đạo trong các nhóm xã hội.
C. Các vấn đề mà các nhà lãnh đạo phải đối mặt.
D. Cách thức mà các nhóm xã hội tìm ra ai là người lãnh đạo họ.
Nội dung cả bài xoay quanh thảo luận về người lãnh đạo trong
nhóm, vai trò và cách thức lãnh đạo. Đáp án đúng là B.
Loại A và D vì chỉ là ý nhỏ của đoạn 1; loại C do không đề cập nhiều
trong bài
Question45. The passage mentions all of the following ways by which people
can become leaders EXCEPT___________
A. recruitment B. specific leadership training
C. traditional cultural patterns D. formal election process
Câu hỏi: Bài viết đề cập tất cả các cách sau để người ta có thể trở thành người lãnh
đạo TRỪ____________
A. tuyển dụng
B. sự huấn luyện lãnh đạo riêng biệt
C. kiểu mẫu văn hóa truyền thống
D. tiến trình bầu cử trang trọng
Đây là câu hỏi có chứa từ phủ định. Thông tin tìm ở đoạn 1, dùng phương pháp loại
trừ, cách thức nào không được nhắc trong bài sẽ là đáp án chọn. Loại các phương án
A, C, D do chúng lần lượt xuất hiện trong các câu:
In the family, traditional cultural patterns (phương án C) confer leadership on one or
both of the parents.
In larger groups, leaders are usually chosen formally through election (phương án D)
or recruitment (phương án A). Trong gia đình, cách thức lãnh đạo truyền thống chọn
lãnh đạo có thể là bố, mẹ hoặc cả hai. Trong các nhóm lớn hơn, những người lãnh
đạo thường được chọn một cách chính thống thông qua bầu cử hoặc tuyển dụng.
Question 46. which of the followingstatements about leadershipcan be inferredfrom
paragraph 2?
A. person can best learn how to be an effective leader by studying research on
leadership.
B. Most people desire to be leadersbut can produce little evidence of their
qualifications.
C. A person who is an effective leader of a particulargroup may not be an effective
leader in another group.
D. Few people succeed in sharing a leadershiprole with another person.
Câu hỏi: Phát biểu nào dưới đây về sự lãnh đạo mà có thể được suy ra từ đoạn 2:
A. Một người có thể học làm người lãnh đạo hiệu quả tốt nhất bằng việc học các nghiên cứu về
lĩnh vực lãnh đạo.
B. Hầu hết mọi người đều khát khao trở thành lãnh đạo nhưng có quá ít bằng chứng về năng
lực.
C. Một người có thể là lãnh đạo tốt của nhóm này nhưng chưa chắc đã là lãnh đạo hiệu quả ở
một nhóm khác.
D. Có quá ít người thành công trong việc chia sẻ sự lãnh đạo với một người khác.
Đoạn 2 nhấn mạnh không có những phẩm chất chung cho tất cả các nhà lãnhđạo của các
nhóm “there is no set of personal qualitiesthat all leaders have in common”; Mỗi nhóm khác
nhau lại có yêu cầu riêng và người lãnh đạo của nhóm ấy phải hội tụ đủ các phẩm chất riêng
biệt ấy để dẫn dắt nhóm của mình “any person may be recognized as a leader if the person
has qualitiesthat meet the needs of that particular group”
Xâu chuỗi các ý lại ta có thể suy ra phương án C là phù hợp nhất với thông tin ở đoạn 2.
Question47. The passage indicates that instrumental leaders generally focus
on___________
A. sharing responsibility with group members
B. achieving a goal
C. ensuring harmonious relationships
D. identifying new leaders
Câu hỏi: Bài viết chỉ ra rằng người lãnh đạo “instrumental leaders” thường tập
trung vào_____________
A. Chia sẻ trách nhiệm với các thành viên khác trong nhóm.
B. Đạt được mục tiêu.
C. Đảm bảo một mối quan hệ hòa thuận giữa các thành viên trong nhóm.
D. Xác định ra những người lãnh đạo mới.
Thông tin có thể được tìm thấy rải rác ở đoạn 3 và 4 nhưng có thể thấy rõ ở đoạn 4
những dòng đầu “Instrumental leaders are likely to have a rather secondary
relationship to other group members. They give orders and maydiscipline group
members who inhibit attainment of the group’s goals.”
Người lãnh đạo “instrumental leaders” có tình cảm xa cách hơn với các thành viên
khác trong nhóm. Họ đưa ra mệnh lệnh và nhiều kỉ luật với các thành viên để có
thể đạt được mục tiêu chung của nhóm.
→ Chọn đáp án B
Question 48. The word “collective” in paragraph 3 is closest in
meaning to___________
A. necessary B. group C. particular D. typical
Câu hỏi: Từ “collective” ở đoạn 3 gần nghĩa nhất với từ nào?
A. cần thiết B. nhóm
C. cụ thể D. điển hình
Câu hỏi từ vựng: đoán nghĩa của từ, ta đặt từ vào văn cảnh và câu
chứa nó để đoán nghĩa. Expressive leadership, on the other hand, is
leadership that emphasizes the collective well-being of a social
group’s member. Người lãnh đạo kiểu “Expressive” mặt khác lại tập
trung vào sự hạnh phúc của mỗi thành viên trong cả nhóm → Chọn
đáp án B
Question49. It can be understoodthat___________
A. There is lots of tension and conflict in an election of a leader in the family.
B. There is usually an election to choose leaders in a family as well as in larger
groups.
C. It has been said that there must be a set of personal qualities that all leaders
have in common.
D. Leaders are sometimes chosen formally or informally.
Câu hỏi: Có thế hiếu rằng_____________
A. Có rất nhiều căng thẳng và mâu thuẫn trong sự lựa chọn người lãnh đạo trong
gia đình.
B. Thường có sự tuyển chọn người lãnh đạo trong gia đình cũng như trong các
nhóm lớn hơn.
C. Người ta thường nói rằng phải có một tiêu chuẩn các phẩm chất năng lực chung
của những người lãnh đạo.
D. Người lãnh đạo được chọn ra một cách trang trọng hoặc không trang trọng.
Đáp án D là đúng vì chúng ta có 2 cách để chọn ra người lãnh đạo, trang trọng ở các
nhóm, tổ chức lớn; không trang trọng ở gia đình và nhóm bạn bè.
A. Bài không nhắc tới.
B. Sai, do gia đình không cần thông qua “election: tuyển chọn” người lãnh đạo.
C. Đoạn 2 khẳng định không có tiêu chuẩn chung cho mọi nhà lãnh đạo.
Dùng phương pháp loại trừ ta chọn được đáp án D.
Question 50. The word “resolve” in paragraph 4 is closest in
meaning to___________
A. talk about B. find a solution for
C. avoid repeating D. avoid thinking about
Câu hỏi: Từ “resolve” ở đoạn 4 đồng nghĩa với từ nào?
A. Nói về
B. Tìm ra cách giải quyết cho
C. Tránh sự lặp lại
D. Tránh nghĩ về
Câu hỏi về từ gần nghĩa nhất, đặt vào câu văn chứa từ resolve ta có.
They offer sympathywhen someone experiences difficulties or is subjected to
discipline, are quick to lighten a serious moment with humor, and try toresolve
issues that threaten to divide the group.
Họ thường có sự thông cảm với những thành viên đang gặp khó khăn hoặc những
người đang phải kỉ luật, họ nhanh chóng làm giảm nhẹ đi những tình huống căng
thẳng với sự hài hước và cố gắng tìm cách giải quyết cho những vấn đề đe dọa gây
chia rẽ nhóm.
→ Chọn đáp án B
Question 1: As a ______ and mature student, Marie Curie earned a
Physics degree with flying colors.
A. brilliant B. brilliantly C. brilliantnessD. brilliance
A. brilliant /’briliənt/ (a): xuất sắc, tài ba
B. brilliantly /’briliəntli/ (a): một cách xuất sắc, một cách lỗi lạc
D. brilliance /’briliəns/ (n): sự xuất sắc, sự tài ba
=> Căn cứ vào mạo từ “a” vào liên từ “and” vì vậy dựa vào cấu trúc
song song thì vị trí còn trống cần sử dụng một tính từ. Ta loại được
phương án B, C, D.
Dịch nghĩa: Là một học sinh xuất sắc và chín chắn, Marie Curie đã
nhận được tấm bằng vật lý với thành tích tuyệt vời.
Question 2: How beautiful the queen is, ________?
A. isn’t she B. is she C. is it D. isn’t it
Dịch nghĩa: “Nữ hoàng thật lộng lẫy, phải không?”
Với câu cảm thán, lấy danh từ trong câu đổi thành đại từ, phần hỏi
đuôi dùng is/ am/ are tùy vào chủ ngữ.
Question 3. We know that we are at fault for our third consecutive
defeat, so there is no need to________salt into the wound.
A. spread B. rub C. apply D. put
Chúng ta có thành ngữ cố định rub salt into the wound có nghĩa là
“xát muối vào vết thương” nên phương án phù hợp nhất là B.
Dịch nghĩa:
Chúng tôi biết rằng chúng tôi có lỗi trong thất bại thứ ba liên tiếp của
chúng ta, vì vậy không cần phải xát muối vào vết thương đâu.
Question 4 .Students are________ less pressure as a result of
changes in testing procedures.
A. under B. above C. upon D. out of
Sau vị trí cần điền là cụm danh từ less pressure (ít áp lực hơn) nên
ta cần một giới từ với nghĩa ở dưới để diễn tả ý chịu áp lực.
Trong bốn phương án, phương án C là giới từ upon nghĩa là ở trên
và phương án D là giới từ out of có nghĩa là ra khỏi nên không phù
hợp. Phương án A và B đều mang nghĩa là ở dưới, tuy nhiên under
được dùng với nghĩa rộng hơn below và đặc biệt ià đi với các từ
ngữ trừu tượng.
Question 5: It is reported that the building was
completely________by fire.
A. spoilt B. ruined C. damaged D. destroyed
Đáp án D
A. spoilt: làm hỏng phá hỏng ( dùng như ruin)
B. ruined : làm hỏng, phá hủy giá trị ( giá trị, cơ hội thú vị…)
C. damaged: làm hư hỏng ( thường nhà của do thiên tai…)
D. destroyed : phá hủy, tàn phá ( làm hư hại phá hủy đến không dùng
được nữa…)
Trong câu có completely:hoàn toàn nên ta dùng đáp án D.
Question6: ____ ASEAN is becoming more integrated, investors should be
aware of local preferences and cultural sensitivities.
A. Because B. However C. Despite D. Although
Dịch nghĩa: Mặc dù ASEAN đang trở nên hội nhập hơn, các nhà đầu tư
nên nhận thức được các ưu tiên địa phương và sự nhạy cảm về văn
hóa.
Xét các đáp án:
A. Because → Bởi vì ASEAN đang trở nên hội nhập hơn, các nhà đầu tư
nên nhận thức được các ưu tiên địa phương và sự nhạy cảm về văn
hóa. (không hợp lý)
B. However → Tuy nhiên, ASEAN đang trở nên hội nhập hơn, các nhà
đầu tư nên nhận thức được các ưu tiên địa phương và sự nhạy cảm về
văn hóa. (không hợp lý)
C. Despite → Liên từ chỉ sự đối lập, sau Despite (mặc dù) = In spite of +
noun/noun phrase/V_ing (DT/cụm DT/V_ing), không phải clause
(mệnh đề)
D. Although → Mệnh đề chỉ sự nhượng bộ, các liên từ được dịch là
Question 7: It is really a (n)_____________ which is suitable for my
daughter.
A. undergraduate interesting economicscourse
B. economicscourse interesting undergraduate
C. interesting economicsundergraduate course
D. interesting undergraduate economics course
*Theo quy tắt trật tự tính từ trong câu: OSASCOMP
interesting - O; economics - P
Và tính từ luôn đứng trước danh từ.
*Note: undergraduate course (phr.n): khóa học đại học
Tạm dịch: Đó thực sự là một khóa học đại học phù hợp cho con gái
tôi.
Question 8: A man was arrested for _________________ the
White House in November 2017.
A. backing down B. breaking into
C. bringing off D. bargaining for
A. backing down: rút lại ý kiến, thừa nhận mình sai/thất bại
B. breaking into: đột nhập
C. bringing off = succeed in doing something difficult: giải quyết việc
khó
D. bargaining for/on st = expect or be prepared for something: mong
đợi, chờ đón
Dịch nghĩa: Một người đàn ông đã bị bắt vì đột nhập vào Nhà Trắng
vào tháng 11 năm 2017.
Question 9: While my father ________________ a film on TV, my
mother was cooking dinner.
A. watched B. was watching C. had watched D. watches
Đáp án B
– Căn cứ vào đây là mệnh đề trạng ngữ chỉ thời gian:
While S + V (quá khứ tiếp diễn), S + V (quá khứ tiếp diễn). (Diễn tả
hai hành động xảy ra song song cùng lúc trong quá khứ).
Vế không chứa “while” chia thì quá khứ tiếp diễn => Vế còn lại chia
thì quá khứ tiếp diễn.
– Dịch: Trong khi bố của tôi đang xem phim trên TV, mẹ của tôi
đang nấu ăn.
Question 10: Mary will have worked at this school for 25 years
________.
A. by the time she retires B. when she retired
C. as soon as she had retired D. after she had retired
Kiến thức: Mệnh đề chỉ thời gian / Sự hòa hợp thì trong mệnh đề
Giải thích:
Mệnh đề chính chia thì tương lai => động từ trong mệnh đề chỉ thời
gian chia thì hiện tại (hiện tại đơn, hiện tại hoàn thành).
Loại luôn được các phương án B, C, D do sai thì.
by the time: cho đến khi
when: khi
as soon as: ngay khi
after: sau khi
Tạm dịch: Mary sẽ đã làm việc được ở ngôi trường này được 25
năm cho đến khi bà ấy nghỉ hưu.
Chọn A.
Question 11: After the visit to that special school, we friends
with some studentswith reading disabilities.
A. acquainted B. had C. made D. realised
Kiến thức về cụm từ cố định
Make friends with sb: kết bạn với ai đó
Tạm dịch: Sau khi tới thăm ngôi trường dành cho học sinh
khuyết tật, tôi đã kết bạn được với một số bạn
có vấn đề về kĩ năng đọc.
Các cấu trúc cần lưu ý khác:
Be acquainted with sb: quen với ai Make acquaintance: 1àm
quen
Question 12. In a formal interview, it is essential to maintain good
eye___________with the interviewers.
A. link B. connection C. touch D. contact
Ta có cụm từ “eye contact”: sự giao tiếp bằng mắt
A. link (n): mối liên lạc
B. connection (n): sự liên quan, mối quan hệ
C. touch (n): sự chạm, sờ
D. contact (n): sự giao tiếp, sự tiếp xúc
Tạm dịch: Trong một cuộc phỏng vấn trang trọng, cần duy trì giao
tiếp tốt bằng mắt với người phỏng vấn.
Question 13. Only half of the exercises ________ so far, but the rest
will have been finished by Saturday.
A. are being done B. were done C. have been done D. have done
Đáp án C. dựa vào so far là dấu hiện dật biết thì hiện tại hoàn thành
loiaj đáp án A,B
Xét đến chủ ngữ “only half of the exercises”( sự việc )- trạng thái bị
động
Tạm dịch : Chỉ một nửa số bài tập đã được thực hiện cho đến nay,
nhưng phần còn lại sẽ được hoàn thành vào thứ Bảy
Question 14. ____________the homework, he was allowed to go
out with his friends.
A. Finishing B. Finish C. To finish D. Having finished
Question 14. Đáp án D
Kiến thức được hỏi: Rút gọn mệnh đề cùng chủ ngữ dùng Having P2.
Khi hai mệnh đề cùng chủ ngữ, và câu muốn nhấn mạnh hành động
phía trước được hoàn thành xong trước rồi hành động phía sau mới
xảy ra thì chúng ta dùng công thức: Having +P2, S+Ved. Đáp án D.
Tạm dịch: Sau khi hoàn thành xong bài tập về nhà, anh ấy được phép
đi chơi với những người bạn của mình.
Question 15. The country is rapidly losing its workers
as____________people are emigrating.
A. the most and the most
B. the more and the more
C. more and more
D. most and most
Question 15. Đáp án C
Kiến thức được hỏi: so sánh kép
More and more + N: càng ngày càng nhiều cái gì
More and more + adj: càng ngày càng thế nào
Trong các phương án duy nhất C đúng.
Tạm dịch: Đất nước đang dần dần bị sụt giảm số lượng người lao
động vì ngày càng nhiều người di cư.
Question 16. “Good luck with the exam, Minh!” - “_______, Dad.”
A. By no means B. I wish so C. Never mind D. Thank you
HD: “Chúc may mắn với bài thi, Minh!”. “______________,Ba”
A. Không có gì.
B. Con hy vọng như vậy.
C. Đừng bận tâm.
D. Cảm ơn.
Question 17. “Dad. I’ve passed the English test.” - “_______.”
A. Good job! B. Good test!C. Good luck! D. Good way!
HD: “Bố. Con vừa qua bài kiểm tra tiếng Anh.”
A. Làm tốt lắm!
B. Bài kiểm tra rất tốt!
C. Chúc may mắn!
D. Một cách tốt!
Question 18. A. sleepy B. trophy C. facial D. exact
Trọng âm đáp án D rơi vào âm tiết số hai, còn các phương án còn lại
trọng âm rơi vào âm thứ nhất.
A. sleepy /ˈsliːpi/ (adj): buồn ngủ
B. trophy /ˈtrəʊfi/ /'troʊfi/ (n): chiếc cúp
C. facial /ˈfeɪʃl/ (adj): thuộc về khuôn mặt
D. exact /ɪɡˈzækt/ (adj): chính xác
Lưu ý: Thường thì danh từ và tính từ có hai âm tiết trọng âm rơi hầu
hết vào số một, tuy nhiên ở đây “exact” là trường hợp ngoại lệ, cần
làm nhiều và tìm tòi bổ sung các trường hợp ngoại lệ khác nữa như
các tính từ: polite, alone, alive danh từ: machine, mistake, police,
idea...
Question 19.
A. authority B. necessity C. academic D. commercially
Trọng âm đáp án C rơi vào âm tiết số 3, còn các phương án còn lại
trọng âm rơi số hai.
A. authority /ɔːˈθɒrəti/ (n): quyền lực, có thẩm quyền
B. necessity /nəˈsesəti/ (n): sự cần thiết
C. academic /ˌækəˈdemɪk/ (adj): có tính chất học thuật, (thuộc) học
viện, (thuộc) trường đại học
D. commercially /kəˈmɜːʃəli/ (adv): về phương diện thương mại
Lưu ý: Đuôi ty, gy, cy, phy, fy thường là các duỗi trọng âm sẽ rơi âm
tiết số ba tính từ dưới lên. Ta có đuôi “ate” cũng vậy.
Đuôi “ic”,“cial”,“tial”,“tion”,“sion”, “cian” trọng âm rơi âm tiết ngay
trước.
A. workshop /ˈwɜːkʃɒp/ (n): hội thảo
B. worm /wɜːm/ (n): sâu/ giun
C. worry /ˈwʌri/ (n)/ (v): mối lo/ lo lắng
D. wordless /ˈwɜːdləs/ (a): không nói nên lời
* Mở rộng: worry có hai phiên âm, là / 'wʌri/ và /'wɜ:ri/. Tuy nhiên,
trong trường hợp này, ba đáp án còn lại đều có phần gạch chân là âm
/ɜ:/ nên từ worry cần có phần gạch chân là âm /ʌ/ để đảm bảo đáp
án.
Question 20. A. borrowed B. helped C. dismissed D. booked
Question 21. A. workshop B. worm C. worry D. wordless
A. borrowed /ˈbɒrəʊ/: mượn
B. helped /helpt/: giúp đỡ
C. dismissed /dɪsˈmɪst/: giải tán
D. booked /bʊk/: đặt vé/ đặt trước
Question 22. The football final has been postponed until next
Sunday due to the heavy snowstorm.
A. cancelled B. changed C. delayed D. continued
Đáp án C đúng vì ta có postpone (v) có nghĩa là hoãn lại = delay (v).
Đây là câu hỏi tìm từ đồng nghĩa nên chọn đáp án C
A. cancelled: hủy bỏ
B. changed: thay đổi
C. delayed: hoãn, làm chậm trễ
D. continued: tiếp tục
Tạm dịch: Trận chung kết bóng đá bị hoãn đến chủ nhật tuần tới do
bão tuyết lớn
Question 23. The discovery of the new planet was regarded as a
major breakthrough in astronomy.
A. promised B. doubted C. considered D. refused
Đáp án C đúng vì ta có: (to) regard st1 as st2 = (to) consider st1 st2: coi
cái gì là cái gì. Đây là câu hỏi tìm từ đồng nghĩa nên ta chọn phương án
C
A. promised: hứa hẹn
B. doubted: nghi ngờ
C. considered: cân nhắc
D. refused: từ chối
Tạm dịch: Sự phát hiện ra hành tinh mới đó được coi như một bước
đột phá lớn trong ngành thiên văn học.
Question 24. The loss of his journals had caused him even more
sorrow than his retirement from the military six years earlier.
A. Grief B. joy C. comfort D. sympathy
Dữ liệu của câu: Việc đánh mất nhật kí của mình khiến ông ấy sorrow
hơn cả việc xuất ngũ 6 năm trước.
sorrow (n): nỗi buồn (danh từ của từ sorry).
grief (n): nỗi đau buồn
joy (n): niềm vui
comfort (n): sự thoải mái
sympathy (n): sự đồng cảm
sorrow (nỗi buồn) trái nghĩa với joy (niềm vui). Đáp án B.
Tạm dịch: Việc đánh mất nhật kí của mình khiến ông ấy buồn hơn cả
việc xuất ngũ 6 năm trước.
Question 25. As a newspaper reporter, she always wanted to get
information at first hand
A. indirectly B. directly C. easily D. slowly
Dữ liệu của bài: Là một phóng viên báo chí, cô ấy luôn muốn nhận
được thống tin at first hand.
Logic: Phóng viên thì muốn tiếp cận thông tin như thế nào?
“At first hand”: trực tiếp
• indirectly (adv): một cách gián tiếp
• directly (adv): trực tiếp
• easily (adv): dễ dàng
• slowly (adv): chậm rãi
→ at first hand (trực tiếp) trái nghĩa với indirectly (một cách gián
tiếp)
Tạm dịch: Là một phóng viên báo chí, cô ấy luôn muốn nhận được
thông tin một cách trực tiếp.
Câu 26: I had just come home. Then I heard the phone ringing.
A.No sooner had I come home then I heard the phone ringing.
B.Scarcely had I come home when I heard the phone ringing.
C.I had barely come home than I heard the phone ringing.
D.Hardly barely come home did I hear the phone ringing.
Câu 26. Chọn đáp án B
Giải thích: Đáp án B: Tôi vừa vào nhà thì tôi nghe thấy tiếng chuông
điện thoại reo.
Đáp án A sai ở chữ “then” Đáp án C sai ở chữ “than” Đáp án D sai
cấu trúc.
*Cấu trúc câu đảo ngữ:
No sooner + QKHT + than + QKĐ: vừa mới.... thì......
= Hardly/Barely/Scarcely + QKHT + when/before +QKĐ
Dịch nghĩa: Ngay khi tôi vào nhà thì tôi nghe thấy tiếng chuông điện
thoại reo.
Question 27: You don’t try to work hard. You will fail in the exam.
A. Unless you don’t try to work hard, you will fail in the exam.
B. Unless you try to work hard, you won’t fail in the exam.
C. Unless you try to work hard, you will fail in the exam.
D. Unless do you try to work hard, you will fail in the exam.
Dịch nghĩa. Bạn không cố gắng học chăm chỉ. Bạn sẽ trượt kỳ thi.
Trừ phi bạn không cố gắng học chăm chỉ, bạn sẽ trượt kì thi.
Trừ phi bạn cố gắng học chăm chỉ, bạn sẽ không trượt kì thi.
Trừ phi bạn cố gắng học chăm chỉ, bạn sẽ trượt kì thi.
Sai cấu trúc.
Question 28: Exceeding speed limits and fail to wear safety belts
are two common causes of automobile death.
A. Exceeding B. fail C. safety belts D. causes
Ta dùng danh động từ (V-ing) như 1 danh từ để làm chủ ngữ của câu
Chữa lỗi: fall → falling
Dịch: Vượt quá giới hạn tốc độ và không đeo dây an toàn là hai
nguyên nhân phổ biến gây tử vong cho
người lái ô tô.
Question 29: It was suggested that John studies the material more
thoroughly before attempting to pass the exam.
A. studies B. more C. attempting D. to pass
Cấu trúc: It was suggested that + S + V (nguyên thể): ai đó được
gợi ý nên làm gì
Chữa lỗi: studies → study
Dịch: John được gợi ý nen học tài liệu ký lưỡng hơn trước khi cố
gắng để đỗ được kỳ thi.
Question 30: They are the shocking news in newspapers that is
what people are talking about this morning.
A. They are B. is C. are D. about
30: A
Danh từ được đề cập đến trong câu là “news” – danh từ không đếm
được → dùng như danh từ số ít
Chữa lỗi: They are → It is
Dịch: Đó là một tin gây sốc trên báo chí, là những gì mà mọi người đã
bàn tán sáng nay
Question 31. I haven't met my grandparents for five years.
A.I often met my grandparents five years ago.
B.I last met my grandparents five years ago.
C.I have met my grandparents for five years.
D.I didn't meet my grandparents five years ago.
Tôi chưa gặp ông bà tôi các đây 5 năm rồi
A.Tôi thường gặp ông bà của tôi 5 năm trước ( sai nghĩa)
B.Lần cuối tôi gặp ông bà của tôi là vào 5 năm trước ( đúng)
C.Tôi gặp ông bà của tôi trong 5 năm( sai nghĩa)
D.Tôi đã không gặp ông bà của tôi 5 năm trước đây( sai nghĩa)
Question 32. “I will help you with the housework”, Mai said.
A. Mai promised to help me with the housework.
B. Mai asked me to help her with the housework.
C. Mai begged to help me with the housework.
D. Mai insisted on helping me with the housework.
Mai “ Tôi sẽ giúp bạn làm công việc nhà”
A.Mai hứa giúp tôi làm công việc nhà
B.May yêu cầu tôi giúp cô ấy làm công việc nhà
C.Mai cầu xin giúp tôi cùng làm công việc nhà
D.Mai khăng khăng giúp tôi làm công việc nhà
Question 33. There was no need for him to have left the house in
such weather.
A.He shouldn’t leave the house in such weather.
B.He didn’t have to leave the house in such weather.
C.He didn’t need to leave the house in such weather.
D.He needn’t have left the house in such weather.
There’s no need for somebody to do something:không cần thiết phải
làm gì. There was no need for somebody to have done something
= Needn’t have done something: đáng lẽ ra không cần thiết phải làm
gì, nhưmg thực tế đã làm điều đó
trong quả khứ.
Đáp án D
Tạm dịch: Lẽ ra bạn đã không cần phải rời nhà trong thời tiết như vậy
Question 34:
The first thing that is included in the "living together” (34) ______
is the expected good relations with your family. This also involves
sharing equally the housework.
A. custom B. tradition C. notion D. trend
Kiến thức về từ vựng
A. custom/ˈkʌs.təm/ (n): phong tục, tập quán
B. tradition /trəˈdɪʃn/ (n): truyền thống
C. notion /ˈnoʊʃn/ (n): quan niệm, niềm tin, ý tưởng hoặc sự hiểu
biết về cái gì
D. trend /trend/ (n): xu hướng, phương hướng
Tạm dịch: “The first thing that is included in the “living together”
(1)___________ is the expected good relations with your family.”
(Điều đầu tiên được bao gồm trong quan niệm “sống chung” là
niềm mong đợi về các mối quan hệ tốt đẹp trong gia đình bạn.)
Question 35:
. This also involves sharing equally the housework. (35) _______ of
people think that everyone should share the housework equally,
but in many homes parents do most of it.
A. lots B. few C. little D. a lot of
Kiến thức về lượng từ
Tạm dịch: “Lots of people think that everyone should share the
housework (2)________, but in many homes parents do most of
it.”
(Nhiều người nghĩ rằng mỗi thành viên nên chia sẻ việc nhà một
cách đồng đều cho nhau, nhưng trong nhiều gia đình thì cha mẹ
làm hầu hết mọi việc.)
Question 36:
(36)_________, housework'scontributions of the teenager make
him more responsible.
A. In addition B. However C. In contrast D. In case
Kiến thức về liên từ
A. In addition: thêm vào đó
B. However: tuy nhiên
C. In contrast: trái lại, trái với
D. In case: phòng khi; trong trường hợp
Tạm dịch: (Theo nhiều quan điểm nhất định, nhiều gia đình không thể
chia sẻ việc nhà cho nhau trong khi lẽ ra họ nên làm vậy. Trên thực tế,
chia sẻ việc nhà một cách đồng đều giữa các thành viên là điều
dường như không thể bởi vì thời gian biểu của các gia đình. Do đó,
theo cách nào đó người ta tin rằng con cái và cha mẹ phải cùng nhau
làm mọi việc. Để làm được điều này họ có thể thiết lập một kế hoạch
cho công việc nhà cửa. Thêm vào đó, những đóng góp trong việc nhà
của thanh thiếu niên cũng làm cho họ trở nên có trách nhiệm hơn.)
Vì hai đoạn văn nối với nhau có ý tưởng trợ, bổ sung lẫn nhau nên ta
Question 37:
According to researchers, teenagers should share the housework
because (37) ______ will help them when they have to establish
their own family in the future.
A. which B. what C. that D. who
Kiến thức về đại từ chỉ định
Xét cấu trúc câu:
(Theo các nhà nghiên cứu, thanh thiếu niên nên sẻ chia việc nhà cho
nhau bởi vì điều đó sẽ giúp ích cho họ khi họ phải lập gia đình riêng
cho mình trong tương lai.)
Nhận thấy câu có ba vế có chủ vị rõ ràng được nối với nhau bởi hai
liên từ là “because” và “when”. Do đó đây không phải dạng đại từ
quan hệ nên loại A, D. Vị trí cần điền là một đại từ chỉ định để thay
thế cho mệnh đề phía trước “teenagers should share the
housework”.
Người ta dùng đại từ chỉ định “that” để thay thế cho điều đã nói
phía trước khi họ không muốn lặp lại câu đó bởi sẽ làm câu nói lủng
củng, rườm rà.
Question 38:
It can be (38) ________ concluded that many parents don't really
prepare their children for future, because they don't stimulate them
to learn how to run a house.
A. likely B. probably C. auspiciously D. possibly
Kiến thức về từ vựng
Xét các đáp án:
A. likely /ˈlaɪkli/ (adv) = probably: có thể, có khả năng xảy ra hoặc trở thành hiện
thực
B. probably /ˈprɑːbəbli/ (adv): có thể, có khả năng xảyra hoặc trở thành hiện
thực
C. auspiciously /ɔːˈspɪʃəsli/ (adv): có khả năng thành công trong tương lai
D. possibly /ˈpɒsəbli/ (adv): có thể (ở mức độ không chắc chắn)
Người ta dùng “possibly” đi với “can (’t)/could(n’t)” trong câu để nhấn mạnh
khả năng cái gì có khả năng/không có khả năng làm gì.
(Có thể kết luận rằng nhiều cha mẹ không thực sự chuẩn bị cho tương lai con cái
họ, bởi vì họ không khích lệ con cái mình học cách làm như nào để quán xuyến
việc nhà.)
Question 39: Which of the following is the best title for the passage?
A. Common Complaints About Work B. Accomplishing Trivial Matters
C. Achieving Job Satisfaction D. Learning to Manage Time
Câu 39 : Đáp án D
Learning to Manage Time = học cách quản lý thời gian
Question 40: According to the passage, why do many people never seem
to have enough time to accomplish things?
A.They do not prioritize tasks.
B. They get tied down by one difficult problem
C. They fail to deal with trivial matters
D. They do not seek the advice of time management experts
A. They do not prioritise tasks.
Thông tin ở đoạn 1: “People commonly complain that they never have
enough time to accomplish tasks. The hours and minutes seem to slip
away before many planned chores get done. According to time
management experts, the main reason for this is that most people fail to
set priorities about what to do first.” (Mọi người thường than phiền rằng
họ chẳng bao giờ có đủ thời gian để hoàn thành các công việc. Hàng giờ
và hàng phút dường như trôi qua đi trước khi các công việc đã được lên
kế hoạch hoàn thiện. Theo các chuyên gia quản lý thời gian, nguyên nhân
chính cho việc này chính là hầu hết mọi người không thể ưu tiên làm việc
gì trước.)
Question 41: In paragraph 2, the word those refers to .
A. daily lists B. trivial matters C. priorities D. people
Thông tin ở đoạn sau: “One simple solution... accomplished daily.”
(Một giải pháp thường được những người đó sử dụng đầu tiên chính là
để cho danh sách các công việc được hoàn thành hàng ngày.
Question 42: The passage states that one solution to time management
problems is to .
A. consult a time management expert
B. accomplish time - consuming matters first
C. keep daily lists of priorities and check them regularly
D. spend only a short time on each task
C. keep daily lists of priorities and check them regularly
Thông tin ở đoạn 2: “One simple solution... progress.” (Một giải
pháp thường được những người đó sử dụng đầu tiên chính là để
cho danh sách các công việc được hoàn thành hàng ngày. Những
danh sách này sắp xếp các công việc từ cần thiết nhất đến ít cần
thiêtết nhất và được kiểm tra thường xuyên qua từng ngày để tiến
hành quá trình.)
Question 43. The paragraph following the passage most probably
discusses .
A. mental and physical health problems B. another solution to time
management problems
C. ways to achieve a sense of fulfilment D. different types of lists
Câu 43: Đáp án B
another solution to time management problems = Một giải
pháp khác để quản lý thời gian
Question 44: What topic does the passage mainly discuss?
A. The way how to write the resume for job application.
B. The mistakes people make when applying for a job.
C. The common way to make impression in a job interview.
D. The necessary skills for job application.
Đoạn văn chủ yếu thảo luận về chủ đề gì?
A. Cách viết bản sơ yếu lý lịch để xin việc.
B. Những lỗi mọi người thường gặp phải khi nộp đơn xin việc.
C. Cách thông thường để tạo ấn tượng tốt trong một cuộc phỏng
vấn xin việc.
D. Những kĩ năng cần thiết để xin việc.
Căn cứ vào thông tin đoạn 1:
There are many mistakes that people make when writing their
resume (CV) or completing a job application. Here are some of the
most common and most serious.
(Có nhiều lỗi mà mọi người thường gặp khi viết bản sơ yếu lí lịch hay hoàn thành
hồ sơ xin việc. Dưới đây là một vài lỗi thường gặp và nghiêm trọng nhất.)
Question 45: The word “executing” in paragraph 2 is closest in
meaning to ________________.
A. Enumerating B. determining C. completing D. implementing
Từ “executing” trong đoạn 2 gần nghĩa nhất với từ __________.
A. liệt kê
B. xác định, tìm ra
C. hoàn thành
D. thực hiện, thi hành
Từ đồng nghĩa: executing (thực hiện) = implementing
They do not necessarily know the specific skills you used in
executing them, nor do they know what results you achieved -
both of which are essential.
(Họ không cần thiết phải biết những kĩ năng cụ thể bạn đã sử dụng
để thực hiện các nhiệm vụ đó, họ cũng không cần biết kết quả bạn
đã đạt được là gì – cả hai thứ đó đều rất quan trọng).
Question 46: The word “concrete” in paragraph 3 could be best
replaced by ______________.
A. indeterminate B. specific C. substantial D. important
Từ “concrete” trong đoạn 3 được thay thế tốt nhất bởi từ
_________.
A. mơ hồ, không rõ
B. cụ thể, rõ ràng
C. chủ yếu, thiết yếu
D. quan trọng
Từ đồng nghĩa: concrete(cụ thể) = specific
The more concrete information you can include, the better. (Thông
tin bạn cung cấp càng cụ thể càng tốt).
Question 47: What does the word “it” in paragraph 3 refer to?
A. organization money B. Information C. productivity D. percentage
Từ “it” trong đoạn 3 đề cập đến từ nào?
A. tiền của tổ chức
B. thông tin
C. năng suất
D. tỉ lệ phần tram
Căn cứ thông tin đoạn 3:
If any innovations you introduced saved the organization money, how
much did they save? If you found a way of increasing productivity, by
what percentage did you increase it?
(Nếu bạn đã đưa ra được ý tưởng đổi mới nào giúp tiết kiệm được
tiền cho tổ chức, thì họ đã tiết kiệm được bao nhiêu tiền? Nếu bạn đề
xuất được 1 cách tăng năng suất, vậy bạn đã làm tăng nó lên bao
nhiêu phần trăm?).
Question 48: According to the passage, what information should
candidates include in their resume?
A. specific skills for previous jobs B. the past achievements
C. previous positions D. future objective
Theo đoạn văn, thông tin nào những người xin việc nên bao hàm trong bản sơ yếu lí lịch của
họ?
A. các kĩ năng cụ thể cho các công việc trước đây
B. các thành tựu đạt được trong quá khứ
C. các chức vụ, vị trí công việc trước đây
D. mục tiêu trong tương lai
Từ khóa: information/ includein their resume
Căn cứ các thông tin trong đoạn văn:
The biggest problem is perhapslisting the duties for which you were responsible in a past
position:all this tells yourpotentialemployersis what you were supposed to do. They do not
necessarily know the specific skillsyou used in executing them, nor do they know what results
you achieved - both of which are essential. (Vấn đề lớn nhất có lẽ là liệt kê các công việc mà
bạn đã làm trong chức vụ trước đây: tất cả những điều này nói cho nhà tuyển dụng tiềm năng
của bạn biết những gì bạn có thể làm được. Họ không cần thiết phải biết những kĩ năng cụ thể
bạn đã sử dụng để thực hiện các nhiệm vụ đó, họ cũng không cần biết kết quả bạn đã đạt
được là gì – cả hai thứ đó đều rất quan trọng).
Writing what you are trying to achieve in life - your objective - is a waste of space. (Viết về
những điều bạn đang cố gắng đạt được trong cuộc sống – mục tiêu của bạn – là một sự lãng
Question 49: Why did the author mention that applicants should
write a good brief career summary?
A. To make the employers interested in what they want.
B. To make the interviewers more curious about you.
C. Because the employers do not care for what you want to achieve.
D. Because it can provide their specific skills in their positions.
Câu 49: Đáp án B
Tại sao tác giả lại đề cập trong đoạn văn rằng những người nộp đơn xin việc nên
viết một bản tóm tắt sự nghiệp hay mà ngắn gọn?
A. Để khiến nhà tuyển dụng quan tâm đến những gì họ muốn.
B. Để khiến những người phỏng vấn tò mò hơn về bạn.
C. Bởi vì những người tuyển dụng không quan tâm bạn đã đạt được thành công gì.
D. Bởi vì nó có thể cung cấp các kĩ năng cụ thể ở vị trí của họ.
Từ khóa: applicants/ write a good brief career summary
Căn cứ thông tin đoạn 4:
Instead, use that space for a career summary. A good one is brief - three to four
sentences long. A good one will make the person reviewing your application want
to read further. (Thay vào đó, hãy sử dụng chỗ giấy đó để viết bản tóm tắt về sự
nghiệp. Một bản tóm tắt phù hợp là tầm 3 đến 4 câu ngắn gọn. Một bản tóm tắt
hay sẽ khiến cho người đọc hồ sơ xin việc của bạn muốn đọc nhiều hơn nữa).
Question 50: According to the passage, which of the following is NOT true?
A. The abilityto negotiateeffectively is as significantas technicalskills.
B. Candidatesmust study the job they are applying carefullybefore writing the CV.
C. Applicantsshould not apply for a distinct job from what they are doing.
D. The information interviewees present should be relatedto the job they are applying.
Theo đoạn văn, câu nào sau đây là không đúng?
A. Khả năng đàm phán hiệu quả cũng quan trọng như các kĩ năng thực hành.
B. Người xin việc cần phải nghiên cứu công việc họ đang nộp đơn xin một cách kĩ càng trước khi
viết bản sơ yếu lí lịch.
C. Người xin việc không nên nộp đơn xin một công việc khác với công việc mà họ đang làm.
D. Những thông tin mà người đi phỏng vấn trình bày nên có liên quan đến công việc mà họ đang
xin.
Từ khóa: not true
Căn cứ vào các thông tin trong đoạn văn:
However, yourabilityto negotiate effectively, for example, can be just as important as your
technicalskills. (Tuy nhiên, khả năng đàm phán hiệu quả của bạn cũng quan trọng như các kĩ
năng thực hành.)
All information you give should be relevant, so carefully consider the job for which you are
applying.If you are applying for a job that is somewhat different than your current job, it is up to
you to draw a connectionfor the resume reviewer, so that they will understandhow your skills
will fit in their organization.(Tất cả các thông tin bạn cung cấp nên có mối liên quan với nhau, vì
vậy hãy nghiên cứu công việc bạn định xin một cách kĩ càng. Nếu bạn đang xin một công việc, về
mặt nào đó, khác với công việc hiện tại của bạn, bạn nên tạo sự liên kết giữa 2 công việc để
người đọc bản sơ yếu lý lịch hiểu được các kĩ năng của bạn phù hợp với tổ chức của họ).
Question 1. Susan has achieved great in her career thanks to
her hard work.
A. successfully. B. successful. C. succeed. D. success
Ta thấy phía trước vị trí trống là tân ngữ của câu và câu trước nó là
một tính từ nên chỉ có thể điền một danh từ.
A. successfully.(adv)
B. successful.(adj)
C. succeed.(v)
D. success (n)
Tạm dịch: Susan đã đạt được thành công lớn trong sự nghiệp của cô
ấy nhờ sự chăm chỉ làm việc của
mình.
Question 2: You’ve been punished of bad marks, _________?
A. have you B. haven’t you C. do you D. don’t you
Dịch nghĩa: “Bạn chưa bao giờ bị phạt vì điểm thấp phải không?”
Câu bắt đầu với you have thì thành lập hỏi đuôi sẽ dùng haven’t
you.
Question 3: Hearing about people who mistreat animals makes me
go hot under the .
A. shoes B. collar C. T-shirt D. vest
Kiến thức về thành ngữ
Ta có thành ngữ: “to be hot under the collar”: điên tiết, cáu tiết
Tạm dịch: Nghe về những người đối xử không tốt với động vật khiến
tôi điên tiết.
Question 4: The sign warns people ______ the dangers of
swimming in this river.
A. about B. from C. with D. to
Dịch nghĩa: Biển báo cảnh báo mọi người về sự nguy hiểm của việc bơi
lội trên dòng sông này.
* Căn cứ vào cấu trúc:
- warn sb about sth: cảnh báo ai về cái gì.
Question 5: Can the sales team meet its financial________?
A. purposes B. aimsC. goals D. objectives
A. purposes: Lý do làm gì hay cái gì tồn tại
B. aims: mục tiêu, đích nhắm (cái đã lên kế hoạch, hạnh động với hy
là đạt được)
C. goals: mục tiêu ( “dùng aim” và thường dùng với “ achieve goals”
D. objectives: mục tiêu, chủ đích cái đề ra để đạt được hay làm
được)
Tạm dịch: đội ngũ bán hàng có thể đáp ứng các mục tiêu tài chính
đã đề ra hay không?
Question 6: The residents of the village are living a happy life
_______ they lack modern facilities.
A. despite B. although C. therefore D. because of
Dịch nghĩa: Dân làng đang sống một cuộc sống hạnh phúc mặc dù họ
thiếu các trang thiết bị tiện nghi hiện đại.
Xét các đáp án:
A. despite = in spite of + N/Ving: mặc dù
B. although + clause: mặc dù
C. therefore: do đó, do vậy
D. because of + N/Ving: bởi vì
=> Theo nghĩa cần một liên từ mang nghĩa “mặc dù” và sau nó là
một mệnh đề, do đó chỉ có B thỏa mãn
Question 7: – “Can I help you, sir?”
– “I’m looking for a ___________ desk for my son.”
A. wooden round fashionable B. round fashionable wooden
C. wooden fashionable round D. fashionable round wooden
*Theo quy tắc trật tự tính từ trong câu: OSASCOMP
: fashionable - O; round - Shape; wooden - M
Dịch: -‘’Tôi có thể giúp gì, thưa ông?’‘ – ’‘Tôi đang tìm một chiếc bàn
tròn hợp thời trang bằng gỗ cho con trai,’’
Question 8: The United States __________ some 150,000 military
reservists when the war broke out.
A. came about B. caught on C. called up D. carried out
A. came about: xảy đến, xảy ra
B. caught on: trở nên phố biến
C. called up: gọi điện, gọi đi lính/nghĩa vụ quân sự
D. carried out: tiến hành
Dịch nghĩa: Mỹ đã gọi đi lính khoảng 150.000 người lính dự bị khi
chiến tranh nổ ra.
Question 9: We______for going home when it began to rain.
A. were preparing B. are preparing C. will be preparing
D. have prepared
– Căn cứ vào đây là mệnh đề trạng ngữ chỉ thời gian:
When S + V (quá khứ đơn), S + V (quá khứ tiếp diễn).
Vế chứa “when” chia thì quá khứ đơn => Vế còn lại chia thì quá khứ
tiếp diễn.
– Dịch: Chúng tôi đang chuẩn bị về nhà thì trời mưa.
Question 10. _______, he will get ready to go to work.
A. When he will have breakfast tomorrow
B. Until he would have breakfast tomorrow
C. After he has breakfast tomorrow
D. As soon as he had breakfast tomorrow
-Căn cứ mệnh đề chính S+ will+Vo và các đáp án bên dưới ta sẽ kết
hợp với câu có S+V( hiện tại đơn)
Vậy ta loại đáp án A,B, D sai thì.
Tạm dịch : Sau khi anh ấy ăn sáng vào ngày mai anh ấy sẽ đi làm
Question 11: You should more attention to what your
teacher explains.
A. make B. pay C. get D. set
Kiến thức về cụm từ cố định
Pay attention to = take notice of: chú ý
Tạm dịch: Bạn nên chú ý tới điều giáo viên giải thích.
Question 12: I'm not going to go ice skating! I'd only fall over and
a fool of myself.
A. create B. show C.do D. make
Kiến thức về cụm từ cố định
make a fool (out) of sb/yourself: khiến ai đó trông như kẻ ngốc
Tạm dịch: Tôi sẽ không đi trượt băng đâu! Tôi chỉ bị ngã và khiến
mình trông như kẻ ngốc.
Question 13. French ________ at their schools as a compulsory
lesson for 20 years until last year.
A. is being taught B. was being taught
C. had been taught D. has been taught
Kiến thức: Câu bị động
Giải thích:
Dấu hiệu : last year, thì quá khứ =>loại A, D, for 20 years (thì quá khứ
hoàn thành) => loại B.
Công thức: S + had been + Ved/V3.
Tạm dịch: Cho đến năm ngoái, Tieesng Pháp đã được dạy như là
môn học bắt buộc ở các trường của họ được 20 năm .
Question 14. ________ all the papers already, Sarah put them back
in the file.
A. To have photocopied B. To photocopy
C. Photocopying D. Having photocopied
Kiến thức: Mệnh đề phân tử / Rút gọn mệnh đề đồng ngữ
Giải thích:
Khi 2 mệnh đề có cùng chủ ngữ (Sarah) thì có thể rút gọn 1 trong 2
mệnh đề về dạng:
- V-ing / Having P2: nếu mệnh đề được rút gọn mang nghĩa chủ động
- P2 (quá khứ phân từ): nếu mệnh đề được rút gọn mang nghĩa bị
động
Chủ ngữ “she” có thể làm chủ (tự thực hiện hành động
“photocopied=> nghĩa chủ động. đã xảy ra hành động nên ta dùng
Having +V.P2
Question 15: The more she practices, _____________ she
becomes.
A. the greater confidence B. more confidently
C. the more confident D. the most confident
Dịch nghĩa: Cô càng luyện tập, cô càng trở nên tự tin.
Xét các đáp án:
A. the greater confidence → Vì cấu trúc become + adj nên
confidence sai
B. more confidently → thiếu mạo từ the và sai từ confidently (sau
become + adj)
C. the more confident → Cấu trúc so sánh kép khi nói về 2 người
hoặc sự vật:
The more/-er + S + V, the more/-er + S + V
D. the most confident → Sai cấu trúc so sánh kép
Question 16: “Take care! Have a safe trip back!” - “_______.”
A. Thanks for coming. B. Sounds good.
C. Thanks, bye. D. Good luck next time.
Tình huống: “Giữ gìn sức khỏe nhé! Chúc đi an toàn nhé!”
A. Cảm ơn vì đã tới. B. Nghe hay đấy.
C. Cảm ơn, tạm biệt. D. Chúc may mắn lần sau.
Question 17: “What a meaningful thing you've done for your less
fortunate friends, my son!” - “_______”
A. Thanks for taking care of me, Mom.
B. What a wonderful gift you've made for me.
C. I don’t really care. I must go now.
D. Thank you, Mom. That’s just a bit I can do for them.
HD: “Thật là một việc có ý nghĩa mà con đã làm cho những người
bạn kém may mắn của mình, con trai!”
A. Cảm ơn vì đã chăm sóc con, mẹ
B. Món quà mẹ tặng con thật tuyệt vời!
C. Con thật sự không quan tâm. Con phải đi bây giờ.
D. Cảm ơn, mẹ. Đó là chỉ một chút con có thể làm cho họ.
Kiến thức về trọng âm với từ hai âm tiết
A.teacher /‘ti:t∫ə(r)/: từ này trọng âm rơi vào âm tiết thứ nhất.
Vì theo quy tắc trọng âm không bao giờ rơi vào âm /ə/ và
trọng âm ưu tiên rơi vào nguyên âm dài.
B.lesson /'lesn/: Từ này trọng âm rơi vào âm tiết thứ nhất.
Vì theo quy tắc nếu các âm mà ngắn hết thì
trọng âm rơi vào âm tiết đầu.
C.action / 'æk∫n/ : Từ này trọng âm rơi vào âm tiết thứ
nhất. Vì theo quy tắc có đuôi -ion thì trọng âm rơi
vào trước âm đó.
D.police /pə'li:s/ : Từ này trọng âm rơi vào âm tiết thứ hai.
Vì theo quy tắc trọng âm không bao giờ rơi
vào âm /ə/ và trọng âm ưu tiên rơi vào nguyên âm dài.
=> Phần D trọng âm ớ âm tiết thứ 2, các từ còn lại trọng âm
rơi vào âm tiết thứ nhất
Question 18 A. teacher B. lesson C. action D. police
Question 19.
A. Importance B. happiness C. employment D. relation
Kiến thức trọng âm với từ 3 âm tiết và hậu tố
A.importance /ɪm’pɔ:tns/:Từ này trọng âm rơi vào âm tiết thứ hai.
Vì theo quy tắc trọng âm ưu tiên rơi
vao nguyên âm dài.
B.happiness /'hæpinəs/: Từ này trọng âm rơi vào âm tiết thứ nhất.
Vì theo quy tắc nếu các âm mà ngắn hết thì trọng âm rơi vào âm tiết
đầu.
C.employment /ɪm’pɔɪmənt/: Từ này trọng âm rơi vào âm tiết thứ
hai. Vì theo quy tắc trọng âm không
rơi vào đuôi -ment và ưu tiên rơi vào nguyên âm dài.
D.relation /rɪ'leɪ∫n/: Từ này trọng âm rơi vào âm tiết thứ hai. Vì
theo quy tắc có đuôi -ion thì trọng âm rơi
vào trước âm đó.
=> Phần B trọng âm rơi vào âm tiết thứ nhất, các từ còn lại trọng
âm ở âm tiết thứ hai.
Question 20. A. played B. planned C. cooked D. lived
Cách phát âm của đuôi -ed
Phần gạch chân của phần C phát âm là /t/, của các từ còn lại là /d/.
Kiến thức cách phát âm của nguyên âm
Phần gạch chân của phần D phát âm là /æ/, của các từ còn lại phát
âm là /ei/.
A. date /deɪt/ B. face /feɪs/ C. page /peɪdʒ/ D. map /mæp/
Question 21. A. date B. face C. page D. map
Question 22: Whenever problems come up, they discuss them
frankly and find solutions quickly.
A. honestly B. constantly C. loyally D. unselfishly
Từ đồng nghĩa
honestly /’ɔnistli/ (adv): chân thật, thẳng thắn
constantly /’kɔnstəntli/(adv): liên miên, kiên định
loyally /’lɔiəli/ (adv): trung thành
unselfishly /ʌn’selfi∫(ə)li/ (adv): không ích kỉ, không màng đến lợi
ích cá nhân
Tạm dịch: Bất cứ khi nào có vấn đề gì xảy ra họ đều trao đổi thẳng
thắn và nhanh chóng tìm ra
hướng giải quyết.
Þ Đáp án A (frankly = honestly)
Question 23: Unselfishnessis the very essence of friendship.
A. necessary part B. important part C. difficult part D.
interesting part
Câu 23: Đáp án B
Từ đồng nghĩa
A. necessary part: phần cần thiết B. important part: phần quan
trọng
C. difficult part: phần khó khăn D. interesting part: phần thú vị
Tạm dịch: Tính không ích kỉ là phần hết sức quan trọng của tình bạn.
Þ Đáp án B (essence = important part)
Question24: Peter is the black sleepof the family. He is currently serving 10
years in jail for stealing a car.
A.a member of a family who is regarded as a disgrace and an embarrassment.
B.a member of a family who supports family by raising sheep.
C. a member of a family who confers prestige on his family.
D. a breadwinner.
Từ trái nghĩa
a member of a family who is regardedas a disgrace and an embarrassment (một
thành viên trong
một gia đình, người mà được coi như một điều ô nhục và đáng hổ thẹn)
a member a family who supports family by raising sheep (một thành viên trong
một gia đình, người
mà nuôi sống gia đình bằng việc nuôi cừu)
a member of a family who confers prestigeon his family (một thành viên trong
một gia đình người
mà mang lại thanh thế cho gia đình mình)
a breadwinner (một trụ cột gia đình)
Tạm dịch: Peter là một đứa con phá gia chi tử trong gia đình. Nó hiện tại đang thụ
án 10 năm tù vì tội ăn trộm xe hơi.
Þ Đáp án C (the black sheep of the family >< a member of a family who confers
prestige on his family)
Question 25: There must be a mutual trust between friends.
A. reliance B. belief C. defendant D. suspicion
Từ trái nghĩa
reliance /ri’laiəns/ (n): sự tin cậy, sự tín nhiệm
belief /bi’li:f/ (n): lòng tin, sự tin tưởng
defendant /di’fendənt/ (n): bị cáo
suspicion /səs’pi∫n/ (n): sự nghi ngờ
Tạm dịch: Cần phải có sự tin tưởng lẫn nhau giữa những người bạn.
Þ Đáp án D (trust >< suspicion)
Question26: He got down to writing the letter as soon as he returned from his walk.
A. No sooner had he returned from his walk when he got down to writing the letter.
B. Not until he returned from his walk did he get down to writing the letter.
C. Only after he had returned from his walk did he get down to writing the letter.
D. Hardly had he returned from his walk when he got down to writing the letter.
Dịch nghĩa: Anh ta ngồi xuống viết thư ngay khi đi bộ về.
Xét các đáp án:
A. No sooner had he returned from his walk when he got down to writing the
letter. → Cấu trúc đảo ngữ với các cụm từ so sánh về thời gian: No sooner +
had + S + P2 + than + S + Ved (nếu dùng no sooner phải có than đằng sau).
B. Not until he returned from his walk did he get down to writing the letter.
→ Mãi cho đến khi anh trở về sau khi đi bộ, anh mới xuống viết thư, sai
nghĩa.
C. Only after he had returned from his walk did he get down to writing the
letter. Chỉ sau khi anh ấy trở về sau khi đi bộ, anh ấy mới viết thư, sai nghĩa
D. Hardly had he returned from his walk when he got down to writing the
letter. → Cấu trúc đảo ngữ với các cụm từ so sánh về thời gian:
Hardly/Scarcely/Barely + had + S + P2 + when + S + Ved: ngay khi/vừa
mới….thì….
Question 27. I can’t give up smoking but I’d like to.
A.I wish I can give up smoking.
B.I wish I could give up smoking.
C. I wish I would give up smoking.
D. I wish I could not give up smoking.
Đáp án. B
Giải thích: Vì tình huống ở thì hiện tại nên cần một điều ước cho hiện
tại có cấu trúc S + wish + V (past simple). Dạng quá khứ của động từ
can là could.
Dịch nghĩa: Tôi ước tôi có thể bỏ hút thuốc.
Question 28: The ocean probably distinguishes the earth from other
planets of the solar system, for scientistsbelieve that large bodies of
water are not existing on the other planets.
A. from B. probably C. for D. are not existing
Kiến thức về thì động từ
Do câu này ám chỉ tới một sự thật nên động từ "exist" không chia
dạng tiếp diễn (tobe V-ing) mà nên chia
về hiện tại đơn.
=> Đáp án D (are not existing => don’t exist)
Tạm dịch: Đại dương là dấu hiệu đặc trưng của Trái Đất khi so sánh
với những hành tinh khác trong hệ mặt trời, bởi các nhà khoa học tin
rằng các khối nước lớn không tồn tại ở những hành tinh khác.
Question 29: The school officials are considering a comprehensive
planning to alleviate the problem of overcrowding in the
dormitories.
A. are B. planning C. alleviate D. overcrowding
Câu 29: Đáp án B
Kiến thức về từ vựng
Vị trí này ta cần một danh từ, vì phía trước có mạo từ “a” và tính từ
“comprehensive”
Þ Đáp án B (planning ® plan)
Tạm dịch: Các cán bộ trong trường học đang xem xét một kế hoạch
toàn diện để làm giảm bớt vấn đề
quá tải trong ký túc xá.
Question 30. The dog stood up slowly, wagged it tail, blinked its
eyes, and barked. A B C
D
Kiến thức: sự hoà hợp giữa chủ ngữ và tính từ sở hữ cách
Giải thích: chủ ngữ là số ít chỉ vật → sở hữu cách cũng phải là số ít
chỉ vật
Sửa: it → its
Tạm dịch: Chú chó con đứng lên một cách chậm rãi, vẫy đuôi,
nheo mắt và sủa.
Question31: I really believe my letter came as a great surprise to John.
A. John maybe very surprised to receive my letter.
B. John mighthave been very surprised to receive my letter.
C. John must be very surprised to receive my letter.
D. John musthave been very surprised to receive my letter.
Đề bài: “Tôi tin chắc rằng bức thư của tôi là một sự bất ngờ lớn đối với John.”
A. John có thể rất bất ngờ khi nhận được thư của tôi. (May + V diễn tả một sự việc
có khả năng xảy ra ở hiện tại nhưng không chắc chắn. Câu này sai vì hành động
“came as a great surprise to John” là hành động đã xảy ra rồi.)
B. John có thể rất bất ngờ khi nhận được thư của tôi (Might have Vp2: diễn tả sự
việc có thể đã xảy ra trong quá khứ nhưng không chắc chắn. Câu này sai vì vế trước
đã có “really believe” nó đã là căn cứ để thể hiện rằng phán đoán này là chắc chắn
và có cơ sở)
C. John chắc chắn sẽ rất bất ngờ khi nhận được thư của tôi. (Must + V: diễn tả một
dự đoán chắc chắn sẽ xảy ra ở hiện tại. Câu này sai vì sự việc ở câu trên đã diễn ra
trong quá khứ rồi.)
D. John chắc chắn sẽ rất bất ngờ khi nhận được thư của tôi (Must have Vp2: diễn
tả sự việc chắc chắn xảy ra trong quá khứ.) => Đúng, vì “really believe” đã thể hiện
tính chắc chắn trong phán đoán của John.
Question 32: He said: “I haven’t met her since she left school.”
A. He said that he hadn’t met her since he had left school.
B. He said that he hadn’t met her since he left school.
C. He said that he hadn’t met her since she left school.
D. He said that he hadn’t met her since she had left school.
Đề bài: Anh ấy nói: “Tôi không gặp cô ấy kể từ khi cô ấy ra trường.”
A. Anh ấy nói rằng anh ấy đã không gặp cô ấy kể từ khi anh ấy ra trường.
(Không giống câu trên về nghĩa)
B. Anh ấy nói rằng anh ấy đã không gặp cô ấy kể từ khi anh ấy ra trường.
(Không giống câu trên về nghĩa)
C. Anh ấy nói rằng anh ấy đã không gặp cô ấy kể từ khi cô ấy ra trường.
(Đúng, theo quy tắc chuyển câu trực tiếp sang câu gián tiếp thì khi có
mệnh đề chỉ thời gian thì không cần lùi, do đó chỉ lùi vế trước của câu,
vế từ “since” trở đi không cần lùi)
D. Anh ấy nói rằng anh ấy đã không gặp cô ấy kể từ khi cô ấy ra trường.
(Sai vì ở mệnh đề với “since”, thì quá khứ ở câu trực tiếp vẫn giữ nguyên
khi chuyển sang câu gián tiếp).
Question 33. The last time he came back to his hometown was 5
years ago.
A. It's 5 years since he last lived in his hometown.
B. He didn't come back to his hometown 5 years ago.
C. He started coming back to his hometown 5 years ago.
D. He hasn’t come back to his hometown for 4 years.
Dùng cấu trúc:
S + have/ has + not + Ved/3………for/ since + time
= The last time + S + Ved/2……was + time (years, month,..) + ago
in + time (May, 1990,…)
S + last + Ved/2 …… time + ago/ in + time
Question 34. Most educational specialists believe that early
schooling should provide children with a(n) (34) _______ of their
own abilities and the self-confidence to use their abilities.
A. attitude B. awareness C. knowledge D. attention
Kiến thức : Đọc điền từ - câu hỏi từ vựng
Giải thích: awareness (n) sự nhận thức
Các đáp án còn lại không hợp về nghĩa:
Attitude (n): thái độ knowledge (n) kiến thức attention
(n) sự chú ý
Most educational specialists believe that early schooling should
provide children with a(n) awareness of their own abilities and the
self-confidence to use their abilities.
Tạm dịch: Hầu hết các chuyên gia giáo dục tin rằng việc đi học sớm
nên cung cấp cho trẻ em một sự nhận thức về khả năng của mình và
sự tự tin để sử dụng khả năng của mình.
Question 35.
One approach recognized by many (35) _______ as promoting
these qualities is the Montessori method, first practised by Maria
Montessori of Italy in the early 1900s.
A. experts B. researchers C. teachers D. tutors
Kiến thức : Đọc điền từ - câu hỏi từ vựng
Giải thích: experts (n): chuyên gia
Các đáp án còn lại không hợp về nghĩa.
reseachers (n) nhà nghiên cứu teachers (n): giáo viênD. tutors
(n): gia sư
One approach recognized by many experts as promoting these
qualities is the Montessori method, first practised by Maria
Montessori of Italy in the early 1900s.
Tạm dịch: Một phương pháp được nhiều chuyên gia công nhận để
thúc đẩy những phẩm chất này là phương pháp Montessori, được
thực hiện lần đầu iên bởi Maria Montessori của Ý vào đầu năm
1900.
Question 36.
Nancy McCormick Rambusch is credited with (36) _______ the
method in the United States, (37) _______ today there are over
400 Montessorischools.
A. popularly B. popularity C. popularizing D. popular
Kiến thức : Đọc điền từ - từ loại
Giải thích: Ở vị trí này cần một danh động từ (V-ing) vì sau giới từ
nên chọn popularizing
Popularly (adv): phổ biến popularity (n) sự phổ biến
Popularize (v) làm cho phổ biến popular (adj)
Tạm dịch: Nancy McComick Rambusch được tin tưởng trong việc
làm phổ biến phương pháp này tại Hoa Kì, nơi hiện nay có hơn 400
trường năng khiếu.
Question 37
Nancy McCormick Rambusch is credited with popularizing the
method in the United States, (37)………….. today there are over
400 Montessori schools.
A. which B. where C. x D. that
Kiến thức : Đọc điền từ - đại từ quan hệ
Giải thích: Ở vị trí này cần 1 đại từ quan hệ chỉ nơi chốn.
Tạm dịch: Nancy McComick Rambusch được tin tưởng trong việc
làm phổ biến phương pháp này tại Hoa Kì, nơi hiện nay có hơn 400
trường năng khiếu.
Question 38
Through such exploration, children develop their sense of touch and
learn how to do everyday tasks without adult assistance. (38) ______
benefits include improvement in language skills, and acquaintance
with elements of science, music, and art.
A. Other B. Others C. Another D. The others
Kiến thức : Đọc điền từ - từ chỉ số lượng
Giải thích:
Other: (khác) dùng với danh từ số nhiều
Others: = other + Noun (số nhiều)
Another: dùng với danh từ số ít
The others: những người kia/ những cái kia
Do vậy chọn A (other)
(29) Other benefits include improvement in language skills, and
acquaintance with elements of science, music, and art
Tạm dịch: Những lợi ích khác bao gồm cải thiện kĩ năng ngôn ngữ, và
làm quen với các yếu tố khoa học, âm nhạc và nghệ thuật.
Question39. what is the main idea of the passage?
A. Wings are the most important physical attribute of birds.
B. Different wing styles evolved for different types of flight.
C. Birds have many specialized features that aid in their survival.
D. Birds fly for many reasons.
Dịch đề bài: Ý chính của đoạn văn này là gì?
A. Cánh là đặc điểm thể chất quan trọng nhất của các loài chim.
B. Các loại cánh chim khác nhau tiến hóa cho các kiểu bay khác nhau.
C. Các loài chim có nhiều đặc điểm chuyên biệt để hỗ trợ sự sinh tồn của mình.
D. Các loài chim bay bởi nhiều lý do.
* Hướng dẫn làm bài:
Câu đầu tiên của bài này là: “Birds have evolved many physical attributes that
contribute to their flying ability.” Và câu cuối của bài này là: “Birds fly to find a prey,
escape predators, and attract mates-in other words, to survive.”
Như vậy, câu đầu tiên của bài nêu ra ý chính là về các đặc điểm thể chất được tiến
hóa để phù hợp với việc bay. Các đoạn sau của bài nói sâu hơn về các đặc điểm đó.
Câu cuối cùng của bài kết luận rằng đối với chim, bay là để tồn tại. Nghĩa là, các loài
chim đã tiến hóa các đặc điểm thể chất để phù hợp với việc bay, và để tồn tại.
Vậy nên, đáp án C đúng.
Các đáp án còn lại chỉ nói về nội dung của một phần trong bài chứ không bao quát
hết bài.
Question40. According to the passage, what causes birds to rise when they start
flying?
A. Long wings with hollow feathers B. Higher air pressure below than above the wings
C. Spreading out their tails D. Superior muscle strength
Dịch đề bài: Theo như bài đọc, điều gì giúp các loài chim nâng được độ cao
khi bắt đầu bay?
A. Cánh dài với những chiếc lông rỗng ruột
B. Áp suất không khí dưới cánh cao hơn ở trên cánh
C. Trải hết đuôi của chúng ra
D. Sức mạnh cơ bắp thượng đẳng
* Hướng dẫn làm bài:
Trích thông tin trong bài: “This creates higher pressure under the wings,
called lift, which pushes the bird up”
Tạm dịch: Điều này tạo ra áp suất cao hơn ở dưới cánh, được gọi là lực đẩy,
chính lực đẩy này đẩy cho chim nâng độ cao.
Như vậy, áp suất phía dưới cánh cao hơn trên cánh là thứ đẩy cho chim tăng
độ cao.
Question 41. The phrase “finches and sparrows” refers to_______.
A. wings B. maneuvers C. ways of flying D. birds
Dịch đề bài: Cụm từ “finches and sparrows” nói đến...
A. cánh
B. vận động/ diễn tập
C. cách bay
D. các loài chim
Câu hỏi về từ vựng. “Finches and sparrows”: Các loài chim họ sẻ Thông và
chim sẻ. Chọn đáp án D - birds: các loại chim - là gần nghĩa nhất với cụm từ
bài cho.
* Mở rộng: Đối với loại bài chọn từ gần nghĩa/ trái nghĩa, nếu không biết
nghĩa của từ, có thể dựa vào dữ liệu xung quanh để đoán nghĩa của từ/ cụm
từ bài cho.
Ví dụ như câu hỏi này, bài đọc cho: “Finches and sparrows have short, broad
wings”: Finches and sparrows có cánh ngắn và rộng. Vì bài đọc đang nói về
cánh chim, nên có thể suy luận được “Finches and sparrows” là tên của hai
loài chim (vì có cánh).
Question42. According to the passage, what benefit comes from having built-in
spoilers?
A. an ability to fly faster B. a steeper angle of diving for prey
C. prolonged flight D. superior maneuverability when climbing
Dịch đề bài: Theo như bài đọc, lợi ích của việc có sẵn các cấu cản dòng là gì?
(spoiler - cấu cản dòng - cấu cản không khí để phanh máy bay)
A. khả năng bay nhanh hơn
B. góc chúi xuống săn mồi đứng hơn
C. các chuyến bay dài hơn
D. tính cơ động cao hơn khi tăng độ cao
Câu hỏi tìm thông tin. Đọc hướng dẫn làm bài ở câu 39.
Từ khóa của câu hỏi là “spoiler”.
Trích thông tin trong bài: “Faster birds, like hawks have built - in spoilers that
reduce turbulence while flying. This allows a steeper angle of attack without
stalling.”
Như vậy, lợi ích của việc có sẵn cấu cản dòng là một góc đứng hơn khi tấn
công con mồi.
Vậy, chọn B là hợp lý.
DE PHAT TRIEN THEO CAU TRUC DE MINH HOA 2022 MON TIENG ANH DE 6 10.pdf
DE PHAT TRIEN THEO CAU TRUC DE MINH HOA 2022 MON TIENG ANH DE 6 10.pdf
DE PHAT TRIEN THEO CAU TRUC DE MINH HOA 2022 MON TIENG ANH DE 6 10.pdf
DE PHAT TRIEN THEO CAU TRUC DE MINH HOA 2022 MON TIENG ANH DE 6 10.pdf
DE PHAT TRIEN THEO CAU TRUC DE MINH HOA 2022 MON TIENG ANH DE 6 10.pdf
DE PHAT TRIEN THEO CAU TRUC DE MINH HOA 2022 MON TIENG ANH DE 6 10.pdf
DE PHAT TRIEN THEO CAU TRUC DE MINH HOA 2022 MON TIENG ANH DE 6 10.pdf
DE PHAT TRIEN THEO CAU TRUC DE MINH HOA 2022 MON TIENG ANH DE 6 10.pdf
DE PHAT TRIEN THEO CAU TRUC DE MINH HOA 2022 MON TIENG ANH DE 6 10.pdf
DE PHAT TRIEN THEO CAU TRUC DE MINH HOA 2022 MON TIENG ANH DE 6 10.pdf
DE PHAT TRIEN THEO CAU TRUC DE MINH HOA 2022 MON TIENG ANH DE 6 10.pdf
DE PHAT TRIEN THEO CAU TRUC DE MINH HOA 2022 MON TIENG ANH DE 6 10.pdf
DE PHAT TRIEN THEO CAU TRUC DE MINH HOA 2022 MON TIENG ANH DE 6 10.pdf
DE PHAT TRIEN THEO CAU TRUC DE MINH HOA 2022 MON TIENG ANH DE 6 10.pdf
DE PHAT TRIEN THEO CAU TRUC DE MINH HOA 2022 MON TIENG ANH DE 6 10.pdf
DE PHAT TRIEN THEO CAU TRUC DE MINH HOA 2022 MON TIENG ANH DE 6 10.pdf
DE PHAT TRIEN THEO CAU TRUC DE MINH HOA 2022 MON TIENG ANH DE 6 10.pdf
DE PHAT TRIEN THEO CAU TRUC DE MINH HOA 2022 MON TIENG ANH DE 6 10.pdf
DE PHAT TRIEN THEO CAU TRUC DE MINH HOA 2022 MON TIENG ANH DE 6 10.pdf
DE PHAT TRIEN THEO CAU TRUC DE MINH HOA 2022 MON TIENG ANH DE 6 10.pdf
DE PHAT TRIEN THEO CAU TRUC DE MINH HOA 2022 MON TIENG ANH DE 6 10.pdf
DE PHAT TRIEN THEO CAU TRUC DE MINH HOA 2022 MON TIENG ANH DE 6 10.pdf
DE PHAT TRIEN THEO CAU TRUC DE MINH HOA 2022 MON TIENG ANH DE 6 10.pdf
DE PHAT TRIEN THEO CAU TRUC DE MINH HOA 2022 MON TIENG ANH DE 6 10.pdf
DE PHAT TRIEN THEO CAU TRUC DE MINH HOA 2022 MON TIENG ANH DE 6 10.pdf
DE PHAT TRIEN THEO CAU TRUC DE MINH HOA 2022 MON TIENG ANH DE 6 10.pdf
DE PHAT TRIEN THEO CAU TRUC DE MINH HOA 2022 MON TIENG ANH DE 6 10.pdf
DE PHAT TRIEN THEO CAU TRUC DE MINH HOA 2022 MON TIENG ANH DE 6 10.pdf
DE PHAT TRIEN THEO CAU TRUC DE MINH HOA 2022 MON TIENG ANH DE 6 10.pdf
DE PHAT TRIEN THEO CAU TRUC DE MINH HOA 2022 MON TIENG ANH DE 6 10.pdf
DE PHAT TRIEN THEO CAU TRUC DE MINH HOA 2022 MON TIENG ANH DE 6 10.pdf
DE PHAT TRIEN THEO CAU TRUC DE MINH HOA 2022 MON TIENG ANH DE 6 10.pdf
DE PHAT TRIEN THEO CAU TRUC DE MINH HOA 2022 MON TIENG ANH DE 6 10.pdf
DE PHAT TRIEN THEO CAU TRUC DE MINH HOA 2022 MON TIENG ANH DE 6 10.pdf
DE PHAT TRIEN THEO CAU TRUC DE MINH HOA 2022 MON TIENG ANH DE 6 10.pdf
DE PHAT TRIEN THEO CAU TRUC DE MINH HOA 2022 MON TIENG ANH DE 6 10.pdf
DE PHAT TRIEN THEO CAU TRUC DE MINH HOA 2022 MON TIENG ANH DE 6 10.pdf
DE PHAT TRIEN THEO CAU TRUC DE MINH HOA 2022 MON TIENG ANH DE 6 10.pdf
DE PHAT TRIEN THEO CAU TRUC DE MINH HOA 2022 MON TIENG ANH DE 6 10.pdf
DE PHAT TRIEN THEO CAU TRUC DE MINH HOA 2022 MON TIENG ANH DE 6 10.pdf
DE PHAT TRIEN THEO CAU TRUC DE MINH HOA 2022 MON TIENG ANH DE 6 10.pdf
DE PHAT TRIEN THEO CAU TRUC DE MINH HOA 2022 MON TIENG ANH DE 6 10.pdf
DE PHAT TRIEN THEO CAU TRUC DE MINH HOA 2022 MON TIENG ANH DE 6 10.pdf
DE PHAT TRIEN THEO CAU TRUC DE MINH HOA 2022 MON TIENG ANH DE 6 10.pdf
DE PHAT TRIEN THEO CAU TRUC DE MINH HOA 2022 MON TIENG ANH DE 6 10.pdf
DE PHAT TRIEN THEO CAU TRUC DE MINH HOA 2022 MON TIENG ANH DE 6 10.pdf
DE PHAT TRIEN THEO CAU TRUC DE MINH HOA 2022 MON TIENG ANH DE 6 10.pdf
DE PHAT TRIEN THEO CAU TRUC DE MINH HOA 2022 MON TIENG ANH DE 6 10.pdf
DE PHAT TRIEN THEO CAU TRUC DE MINH HOA 2022 MON TIENG ANH DE 6 10.pdf
DE PHAT TRIEN THEO CAU TRUC DE MINH HOA 2022 MON TIENG ANH DE 6 10.pdf
DE PHAT TRIEN THEO CAU TRUC DE MINH HOA 2022 MON TIENG ANH DE 6 10.pdf
DE PHAT TRIEN THEO CAU TRUC DE MINH HOA 2022 MON TIENG ANH DE 6 10.pdf
DE PHAT TRIEN THEO CAU TRUC DE MINH HOA 2022 MON TIENG ANH DE 6 10.pdf
DE PHAT TRIEN THEO CAU TRUC DE MINH HOA 2022 MON TIENG ANH DE 6 10.pdf
DE PHAT TRIEN THEO CAU TRUC DE MINH HOA 2022 MON TIENG ANH DE 6 10.pdf
DE PHAT TRIEN THEO CAU TRUC DE MINH HOA 2022 MON TIENG ANH DE 6 10.pdf
DE PHAT TRIEN THEO CAU TRUC DE MINH HOA 2022 MON TIENG ANH DE 6 10.pdf
DE PHAT TRIEN THEO CAU TRUC DE MINH HOA 2022 MON TIENG ANH DE 6 10.pdf
DE PHAT TRIEN THEO CAU TRUC DE MINH HOA 2022 MON TIENG ANH DE 6 10.pdf
DE PHAT TRIEN THEO CAU TRUC DE MINH HOA 2022 MON TIENG ANH DE 6 10.pdf
DE PHAT TRIEN THEO CAU TRUC DE MINH HOA 2022 MON TIENG ANH DE 6 10.pdf
DE PHAT TRIEN THEO CAU TRUC DE MINH HOA 2022 MON TIENG ANH DE 6 10.pdf
DE PHAT TRIEN THEO CAU TRUC DE MINH HOA 2022 MON TIENG ANH DE 6 10.pdf
DE PHAT TRIEN THEO CAU TRUC DE MINH HOA 2022 MON TIENG ANH DE 6 10.pdf
DE PHAT TRIEN THEO CAU TRUC DE MINH HOA 2022 MON TIENG ANH DE 6 10.pdf
DE PHAT TRIEN THEO CAU TRUC DE MINH HOA 2022 MON TIENG ANH DE 6 10.pdf
DE PHAT TRIEN THEO CAU TRUC DE MINH HOA 2022 MON TIENG ANH DE 6 10.pdf
DE PHAT TRIEN THEO CAU TRUC DE MINH HOA 2022 MON TIENG ANH DE 6 10.pdf
DE PHAT TRIEN THEO CAU TRUC DE MINH HOA 2022 MON TIENG ANH DE 6 10.pdf
DE PHAT TRIEN THEO CAU TRUC DE MINH HOA 2022 MON TIENG ANH DE 6 10.pdf
DE PHAT TRIEN THEO CAU TRUC DE MINH HOA 2022 MON TIENG ANH DE 6 10.pdf
DE PHAT TRIEN THEO CAU TRUC DE MINH HOA 2022 MON TIENG ANH DE 6 10.pdf
DE PHAT TRIEN THEO CAU TRUC DE MINH HOA 2022 MON TIENG ANH DE 6 10.pdf
DE PHAT TRIEN THEO CAU TRUC DE MINH HOA 2022 MON TIENG ANH DE 6 10.pdf
DE PHAT TRIEN THEO CAU TRUC DE MINH HOA 2022 MON TIENG ANH DE 6 10.pdf
DE PHAT TRIEN THEO CAU TRUC DE MINH HOA 2022 MON TIENG ANH DE 6 10.pdf
DE PHAT TRIEN THEO CAU TRUC DE MINH HOA 2022 MON TIENG ANH DE 6 10.pdf
DE PHAT TRIEN THEO CAU TRUC DE MINH HOA 2022 MON TIENG ANH DE 6 10.pdf
DE PHAT TRIEN THEO CAU TRUC DE MINH HOA 2022 MON TIENG ANH DE 6 10.pdf
DE PHAT TRIEN THEO CAU TRUC DE MINH HOA 2022 MON TIENG ANH DE 6 10.pdf
DE PHAT TRIEN THEO CAU TRUC DE MINH HOA 2022 MON TIENG ANH DE 6 10.pdf
DE PHAT TRIEN THEO CAU TRUC DE MINH HOA 2022 MON TIENG ANH DE 6 10.pdf
DE PHAT TRIEN THEO CAU TRUC DE MINH HOA 2022 MON TIENG ANH DE 6 10.pdf
DE PHAT TRIEN THEO CAU TRUC DE MINH HOA 2022 MON TIENG ANH DE 6 10.pdf
DE PHAT TRIEN THEO CAU TRUC DE MINH HOA 2022 MON TIENG ANH DE 6 10.pdf
DE PHAT TRIEN THEO CAU TRUC DE MINH HOA 2022 MON TIENG ANH DE 6 10.pdf
DE PHAT TRIEN THEO CAU TRUC DE MINH HOA 2022 MON TIENG ANH DE 6 10.pdf
DE PHAT TRIEN THEO CAU TRUC DE MINH HOA 2022 MON TIENG ANH DE 6 10.pdf
DE PHAT TRIEN THEO CAU TRUC DE MINH HOA 2022 MON TIENG ANH DE 6 10.pdf
DE PHAT TRIEN THEO CAU TRUC DE MINH HOA 2022 MON TIENG ANH DE 6 10.pdf
DE PHAT TRIEN THEO CAU TRUC DE MINH HOA 2022 MON TIENG ANH DE 6 10.pdf
DE PHAT TRIEN THEO CAU TRUC DE MINH HOA 2022 MON TIENG ANH DE 6 10.pdf
DE PHAT TRIEN THEO CAU TRUC DE MINH HOA 2022 MON TIENG ANH DE 6 10.pdf
DE PHAT TRIEN THEO CAU TRUC DE MINH HOA 2022 MON TIENG ANH DE 6 10.pdf
DE PHAT TRIEN THEO CAU TRUC DE MINH HOA 2022 MON TIENG ANH DE 6 10.pdf
DE PHAT TRIEN THEO CAU TRUC DE MINH HOA 2022 MON TIENG ANH DE 6 10.pdf
DE PHAT TRIEN THEO CAU TRUC DE MINH HOA 2022 MON TIENG ANH DE 6 10.pdf
DE PHAT TRIEN THEO CAU TRUC DE MINH HOA 2022 MON TIENG ANH DE 6 10.pdf
DE PHAT TRIEN THEO CAU TRUC DE MINH HOA 2022 MON TIENG ANH DE 6 10.pdf
DE PHAT TRIEN THEO CAU TRUC DE MINH HOA 2022 MON TIENG ANH DE 6 10.pdf
DE PHAT TRIEN THEO CAU TRUC DE MINH HOA 2022 MON TIENG ANH DE 6 10.pdf
DE PHAT TRIEN THEO CAU TRUC DE MINH HOA 2022 MON TIENG ANH DE 6 10.pdf
DE PHAT TRIEN THEO CAU TRUC DE MINH HOA 2022 MON TIENG ANH DE 6 10.pdf
DE PHAT TRIEN THEO CAU TRUC DE MINH HOA 2022 MON TIENG ANH DE 6 10.pdf

Weitere ähnliche Inhalte

Was ist angesagt?

Đề thi đại học 2008 môn Tiếng Anh
Đề thi đại học 2008 môn Tiếng AnhĐề thi đại học 2008 môn Tiếng Anh
Đề thi đại học 2008 môn Tiếng Anhtuituhoc
 
Dịch nghĩa và giải thích chi tiết starter toeic unit 2
Dịch nghĩa và giải thích chi tiết starter toeic unit 2Dịch nghĩa và giải thích chi tiết starter toeic unit 2
Dịch nghĩa và giải thích chi tiết starter toeic unit 2my nguyễn
 
Đề Tiếng Anh 12 cơ bản unit 10 có đáp án - VipLam.Net
Đề Tiếng Anh 12 cơ bản unit 10 có đáp án - VipLam.NetĐề Tiếng Anh 12 cơ bản unit 10 có đáp án - VipLam.Net
Đề Tiếng Anh 12 cơ bản unit 10 có đáp án - VipLam.NetThùy Linh
 
Bai tap-tieng-anh-12unit-10
Bai tap-tieng-anh-12unit-10Bai tap-tieng-anh-12unit-10
Bai tap-tieng-anh-12unit-10Huyen Tran
 
De tieng-anh-part5-toeic-co-dap-an-loi-giai-chi-tiet
De tieng-anh-part5-toeic-co-dap-an-loi-giai-chi-tietDe tieng-anh-part5-toeic-co-dap-an-loi-giai-chi-tiet
De tieng-anh-part5-toeic-co-dap-an-loi-giai-chi-tietCharlie Cúc Cu
 
đáP án và giải thích đề 2
đáP án và giải thích đề 2đáP án và giải thích đề 2
đáP án và giải thích đề 2Huynh ICT
 
Dịch nghĩa và giải thích chi tiết starter toeic unit 4
Dịch nghĩa và giải thích chi tiết starter toeic unit 4Dịch nghĩa và giải thích chi tiết starter toeic unit 4
Dịch nghĩa và giải thích chi tiết starter toeic unit 4my nguyễn
 
Bài Giải Chi Tiết Môn Anh Hệ CĐ Khối A1&D 2014
Bài Giải Chi Tiết Môn Anh Hệ CĐ Khối A1&D 2014Bài Giải Chi Tiết Môn Anh Hệ CĐ Khối A1&D 2014
Bài Giải Chi Tiết Môn Anh Hệ CĐ Khối A1&D 2014Tommy Bảo
 
Đề Tiếng Anh 12 cơ bản unit 12 có đáp án - VipLam.Net
Đề Tiếng Anh 12 cơ bản unit 12 có đáp án - VipLam.NetĐề Tiếng Anh 12 cơ bản unit 12 có đáp án - VipLam.Net
Đề Tiếng Anh 12 cơ bản unit 12 có đáp án - VipLam.NetThùy Linh
 
Dịch nghĩa và giải thích chi tiết starter toeic unit 3
Dịch nghĩa và giải thích chi tiết starter toeic unit 3Dịch nghĩa và giải thích chi tiết starter toeic unit 3
Dịch nghĩa và giải thích chi tiết starter toeic unit 3my nguyễn
 
BÀI TẬP BỔ TRỢ TIẾNG ANH LỚP 10 GLOBAL SUCCESS NĂM 2023 CÓ FILE NGHE - 2 HỌC ...
BÀI TẬP BỔ TRỢ TIẾNG ANH LỚP 10 GLOBAL SUCCESS NĂM 2023 CÓ FILE NGHE - 2 HỌC ...BÀI TẬP BỔ TRỢ TIẾNG ANH LỚP 10 GLOBAL SUCCESS NĂM 2023 CÓ FILE NGHE - 2 HỌC ...
BÀI TẬP BỔ TRỢ TIẾNG ANH LỚP 10 GLOBAL SUCCESS NĂM 2023 CÓ FILE NGHE - 2 HỌC ...Nguyen Thanh Tu Collection
 
Đề Tiếng Anh 12 cơ bản unit 15 có đáp án - VipLam.Net
Đề Tiếng Anh 12 cơ bản unit 15 có đáp án - VipLam.NetĐề Tiếng Anh 12 cơ bản unit 15 có đáp án - VipLam.Net
Đề Tiếng Anh 12 cơ bản unit 15 có đáp án - VipLam.NetThùy Linh
 
40 ĐỀ THI THỬ TỐT NGHIỆP THPT - MÔN TIẾNG ANH - NĂM 2023 - SOẠN CHUẨN CẤU TR...
40 ĐỀ THI THỬ TỐT NGHIỆP THPT - MÔN TIẾNG ANH - NĂM 2023 - SOẠN CHUẨN CẤU TR...40 ĐỀ THI THỬ TỐT NGHIỆP THPT - MÔN TIẾNG ANH - NĂM 2023 - SOẠN CHUẨN CẤU TR...
40 ĐỀ THI THỬ TỐT NGHIỆP THPT - MÔN TIẾNG ANH - NĂM 2023 - SOẠN CHUẨN CẤU TR...Nguyen Thanh Tu Collection
 
Đề Tiếng Anh 12 cơ bản unit 3 có đáp án - VipLam.Net
Đề Tiếng Anh 12 cơ bản unit 3 có đáp án - VipLam.NetĐề Tiếng Anh 12 cơ bản unit 3 có đáp án - VipLam.Net
Đề Tiếng Anh 12 cơ bản unit 3 có đáp án - VipLam.NetThùy Linh
 
Bài tập ôn thi THPT Quốc gia: Từ đồng nghĩa - trái nghĩa
Bài tập ôn thi THPT Quốc gia: Từ đồng nghĩa - trái nghĩaBài tập ôn thi THPT Quốc gia: Từ đồng nghĩa - trái nghĩa
Bài tập ôn thi THPT Quốc gia: Từ đồng nghĩa - trái nghĩaschoolantoreecom
 
Từ vựng TOEIC Part 7 thông dụng nhất - ORI TOEIC
Từ vựng TOEIC Part 7 thông dụng nhất - ORI TOEICTừ vựng TOEIC Part 7 thông dụng nhất - ORI TOEIC
Từ vựng TOEIC Part 7 thông dụng nhất - ORI TOEICTran Diem
 
Đề Tiếng Anh 12 cơ bản unit 4 có đáp án - VipLam.Net
Đề Tiếng Anh 12 cơ bản unit 4 có đáp án - VipLam.NetĐề Tiếng Anh 12 cơ bản unit 4 có đáp án - VipLam.Net
Đề Tiếng Anh 12 cơ bản unit 4 có đáp án - VipLam.NetThùy Linh
 
Đề Tiếng Anh 12 cơ bản unit 6 có đáp án - VipLam.Net
Đề Tiếng Anh 12 cơ bản unit 6 có đáp án - VipLam.NetĐề Tiếng Anh 12 cơ bản unit 6 có đáp án - VipLam.Net
Đề Tiếng Anh 12 cơ bản unit 6 có đáp án - VipLam.NetThùy Linh
 
Đề thi thử và đáp án chi tiết môn Tiếng Anh số 3 - Megabook.vn
Đề thi thử và đáp án chi tiết môn Tiếng Anh số 3 - Megabook.vnĐề thi thử và đáp án chi tiết môn Tiếng Anh số 3 - Megabook.vn
Đề thi thử và đáp án chi tiết môn Tiếng Anh số 3 - Megabook.vnMegabook
 
đáP án và giải thích đề 20
đáP án và giải thích đề 20đáP án và giải thích đề 20
đáP án và giải thích đề 20Huynh ICT
 

Was ist angesagt? (20)

Đề thi đại học 2008 môn Tiếng Anh
Đề thi đại học 2008 môn Tiếng AnhĐề thi đại học 2008 môn Tiếng Anh
Đề thi đại học 2008 môn Tiếng Anh
 
Dịch nghĩa và giải thích chi tiết starter toeic unit 2
Dịch nghĩa và giải thích chi tiết starter toeic unit 2Dịch nghĩa và giải thích chi tiết starter toeic unit 2
Dịch nghĩa và giải thích chi tiết starter toeic unit 2
 
Đề Tiếng Anh 12 cơ bản unit 10 có đáp án - VipLam.Net
Đề Tiếng Anh 12 cơ bản unit 10 có đáp án - VipLam.NetĐề Tiếng Anh 12 cơ bản unit 10 có đáp án - VipLam.Net
Đề Tiếng Anh 12 cơ bản unit 10 có đáp án - VipLam.Net
 
Bai tap-tieng-anh-12unit-10
Bai tap-tieng-anh-12unit-10Bai tap-tieng-anh-12unit-10
Bai tap-tieng-anh-12unit-10
 
De tieng-anh-part5-toeic-co-dap-an-loi-giai-chi-tiet
De tieng-anh-part5-toeic-co-dap-an-loi-giai-chi-tietDe tieng-anh-part5-toeic-co-dap-an-loi-giai-chi-tiet
De tieng-anh-part5-toeic-co-dap-an-loi-giai-chi-tiet
 
đáP án và giải thích đề 2
đáP án và giải thích đề 2đáP án và giải thích đề 2
đáP án và giải thích đề 2
 
Dịch nghĩa và giải thích chi tiết starter toeic unit 4
Dịch nghĩa và giải thích chi tiết starter toeic unit 4Dịch nghĩa và giải thích chi tiết starter toeic unit 4
Dịch nghĩa và giải thích chi tiết starter toeic unit 4
 
Bài Giải Chi Tiết Môn Anh Hệ CĐ Khối A1&D 2014
Bài Giải Chi Tiết Môn Anh Hệ CĐ Khối A1&D 2014Bài Giải Chi Tiết Môn Anh Hệ CĐ Khối A1&D 2014
Bài Giải Chi Tiết Môn Anh Hệ CĐ Khối A1&D 2014
 
Đề Tiếng Anh 12 cơ bản unit 12 có đáp án - VipLam.Net
Đề Tiếng Anh 12 cơ bản unit 12 có đáp án - VipLam.NetĐề Tiếng Anh 12 cơ bản unit 12 có đáp án - VipLam.Net
Đề Tiếng Anh 12 cơ bản unit 12 có đáp án - VipLam.Net
 
Dịch nghĩa và giải thích chi tiết starter toeic unit 3
Dịch nghĩa và giải thích chi tiết starter toeic unit 3Dịch nghĩa và giải thích chi tiết starter toeic unit 3
Dịch nghĩa và giải thích chi tiết starter toeic unit 3
 
BÀI TẬP BỔ TRỢ TIẾNG ANH LỚP 10 GLOBAL SUCCESS NĂM 2023 CÓ FILE NGHE - 2 HỌC ...
BÀI TẬP BỔ TRỢ TIẾNG ANH LỚP 10 GLOBAL SUCCESS NĂM 2023 CÓ FILE NGHE - 2 HỌC ...BÀI TẬP BỔ TRỢ TIẾNG ANH LỚP 10 GLOBAL SUCCESS NĂM 2023 CÓ FILE NGHE - 2 HỌC ...
BÀI TẬP BỔ TRỢ TIẾNG ANH LỚP 10 GLOBAL SUCCESS NĂM 2023 CÓ FILE NGHE - 2 HỌC ...
 
Đề Tiếng Anh 12 cơ bản unit 15 có đáp án - VipLam.Net
Đề Tiếng Anh 12 cơ bản unit 15 có đáp án - VipLam.NetĐề Tiếng Anh 12 cơ bản unit 15 có đáp án - VipLam.Net
Đề Tiếng Anh 12 cơ bản unit 15 có đáp án - VipLam.Net
 
40 ĐỀ THI THỬ TỐT NGHIỆP THPT - MÔN TIẾNG ANH - NĂM 2023 - SOẠN CHUẨN CẤU TR...
40 ĐỀ THI THỬ TỐT NGHIỆP THPT - MÔN TIẾNG ANH - NĂM 2023 - SOẠN CHUẨN CẤU TR...40 ĐỀ THI THỬ TỐT NGHIỆP THPT - MÔN TIẾNG ANH - NĂM 2023 - SOẠN CHUẨN CẤU TR...
40 ĐỀ THI THỬ TỐT NGHIỆP THPT - MÔN TIẾNG ANH - NĂM 2023 - SOẠN CHUẨN CẤU TR...
 
Đề Tiếng Anh 12 cơ bản unit 3 có đáp án - VipLam.Net
Đề Tiếng Anh 12 cơ bản unit 3 có đáp án - VipLam.NetĐề Tiếng Anh 12 cơ bản unit 3 có đáp án - VipLam.Net
Đề Tiếng Anh 12 cơ bản unit 3 có đáp án - VipLam.Net
 
Bài tập ôn thi THPT Quốc gia: Từ đồng nghĩa - trái nghĩa
Bài tập ôn thi THPT Quốc gia: Từ đồng nghĩa - trái nghĩaBài tập ôn thi THPT Quốc gia: Từ đồng nghĩa - trái nghĩa
Bài tập ôn thi THPT Quốc gia: Từ đồng nghĩa - trái nghĩa
 
Từ vựng TOEIC Part 7 thông dụng nhất - ORI TOEIC
Từ vựng TOEIC Part 7 thông dụng nhất - ORI TOEICTừ vựng TOEIC Part 7 thông dụng nhất - ORI TOEIC
Từ vựng TOEIC Part 7 thông dụng nhất - ORI TOEIC
 
Đề Tiếng Anh 12 cơ bản unit 4 có đáp án - VipLam.Net
Đề Tiếng Anh 12 cơ bản unit 4 có đáp án - VipLam.NetĐề Tiếng Anh 12 cơ bản unit 4 có đáp án - VipLam.Net
Đề Tiếng Anh 12 cơ bản unit 4 có đáp án - VipLam.Net
 
Đề Tiếng Anh 12 cơ bản unit 6 có đáp án - VipLam.Net
Đề Tiếng Anh 12 cơ bản unit 6 có đáp án - VipLam.NetĐề Tiếng Anh 12 cơ bản unit 6 có đáp án - VipLam.Net
Đề Tiếng Anh 12 cơ bản unit 6 có đáp án - VipLam.Net
 
Đề thi thử và đáp án chi tiết môn Tiếng Anh số 3 - Megabook.vn
Đề thi thử và đáp án chi tiết môn Tiếng Anh số 3 - Megabook.vnĐề thi thử và đáp án chi tiết môn Tiếng Anh số 3 - Megabook.vn
Đề thi thử và đáp án chi tiết môn Tiếng Anh số 3 - Megabook.vn
 
đáP án và giải thích đề 20
đáP án và giải thích đề 20đáP án và giải thích đề 20
đáP án và giải thích đề 20
 

Ähnlich wie DE PHAT TRIEN THEO CAU TRUC DE MINH HOA 2022 MON TIENG ANH DE 6 10.pdf

đáP án và giải thích đề 10
đáP án và giải thích đề 10đáP án và giải thích đề 10
đáP án và giải thích đề 10Huynh ICT
 
đáP án và giải thích đề 25
đáP án và giải thích đề 25đáP án và giải thích đề 25
đáP án và giải thích đề 25Huynh ICT
 
đáP án và giải thích đề 24
đáP án và giải thích đề 24đáP án và giải thích đề 24
đáP án và giải thích đề 24Huynh ICT
 
đáP án và giải thích đề 8
đáP án và giải thích đề 8đáP án và giải thích đề 8
đáP án và giải thích đề 8Huynh ICT
 
đáP án và giải thích đề 31
đáP án và giải thích đề 31đáP án và giải thích đề 31
đáP án và giải thích đề 31Huynh ICT
 
đáP án và giải thích đề 19
đáP án và giải thích đề 19đáP án và giải thích đề 19
đáP án và giải thích đề 19Huynh ICT
 
đáP án và giải thích đề 23
đáP án và giải thích đề 23đáP án và giải thích đề 23
đáP án và giải thích đề 23Huynh ICT
 
đáP án và giải thích đề 16
đáP án và giải thích đề 16đáP án và giải thích đề 16
đáP án và giải thích đề 16Huynh ICT
 
Đề thi thử Ôn thi Tiếng Anh vào Cao Đẳng , Đại học năm 2013 - Đề 110
Đề thi thử  Ôn thi Tiếng Anh vào Cao Đẳng , Đại học năm 2013 - Đề 110Đề thi thử  Ôn thi Tiếng Anh vào Cao Đẳng , Đại học năm 2013 - Đề 110
Đề thi thử Ôn thi Tiếng Anh vào Cao Đẳng , Đại học năm 2013 - Đề 110phamnhakb
 
đáP án và giải thích đề 6
đáP án và giải thích đề 6đáP án và giải thích đề 6
đáP án và giải thích đề 6Huynh ICT
 
đáP án và giải thích đề 14
đáP án và giải thích đề 14đáP án và giải thích đề 14
đáP án và giải thích đề 14Huynh ICT
 
đáP án và giải thích đề 9
đáP án và giải thích đề 9đáP án và giải thích đề 9
đáP án và giải thích đề 9Huynh ICT
 
đáP án và giải thích đề 4
đáP án và giải thích đề 4đáP án và giải thích đề 4
đáP án và giải thích đề 4Huynh ICT
 
Giai chi tiet de thi dh khoi d t anh 2011
Giai chi tiet de thi dh khoi d t anh 2011Giai chi tiet de thi dh khoi d t anh 2011
Giai chi tiet de thi dh khoi d t anh 2011Hua Tran Phuong Thao
 
đáP án và giải thích đề 17
đáP án và giải thích đề 17đáP án và giải thích đề 17
đáP án và giải thích đề 17Huynh ICT
 
đáP án và giải thích đề 18
đáP án và giải thích đề 18đáP án và giải thích đề 18
đáP án và giải thích đề 18Huynh ICT
 
đáP án và giải thích đề 32
đáP án và giải thích đề 32đáP án và giải thích đề 32
đáP án và giải thích đề 32Huynh ICT
 
đáP án và giải thích đề 13
đáP án và giải thích đề 13đáP án và giải thích đề 13
đáP án và giải thích đề 13Huynh ICT
 
đáP án và giải thích đề 26
đáP án và giải thích đề 26đáP án và giải thích đề 26
đáP án và giải thích đề 26Huynh ICT
 
đáP án và giải thích đề 11
đáP án và giải thích đề 11đáP án và giải thích đề 11
đáP án và giải thích đề 11Huynh ICT
 

Ähnlich wie DE PHAT TRIEN THEO CAU TRUC DE MINH HOA 2022 MON TIENG ANH DE 6 10.pdf (20)

đáP án và giải thích đề 10
đáP án và giải thích đề 10đáP án và giải thích đề 10
đáP án và giải thích đề 10
 
đáP án và giải thích đề 25
đáP án và giải thích đề 25đáP án và giải thích đề 25
đáP án và giải thích đề 25
 
đáP án và giải thích đề 24
đáP án và giải thích đề 24đáP án và giải thích đề 24
đáP án và giải thích đề 24
 
đáP án và giải thích đề 8
đáP án và giải thích đề 8đáP án và giải thích đề 8
đáP án và giải thích đề 8
 
đáP án và giải thích đề 31
đáP án và giải thích đề 31đáP án và giải thích đề 31
đáP án và giải thích đề 31
 
đáP án và giải thích đề 19
đáP án và giải thích đề 19đáP án và giải thích đề 19
đáP án và giải thích đề 19
 
đáP án và giải thích đề 23
đáP án và giải thích đề 23đáP án và giải thích đề 23
đáP án và giải thích đề 23
 
đáP án và giải thích đề 16
đáP án và giải thích đề 16đáP án và giải thích đề 16
đáP án và giải thích đề 16
 
Đề thi thử Ôn thi Tiếng Anh vào Cao Đẳng , Đại học năm 2013 - Đề 110
Đề thi thử  Ôn thi Tiếng Anh vào Cao Đẳng , Đại học năm 2013 - Đề 110Đề thi thử  Ôn thi Tiếng Anh vào Cao Đẳng , Đại học năm 2013 - Đề 110
Đề thi thử Ôn thi Tiếng Anh vào Cao Đẳng , Đại học năm 2013 - Đề 110
 
đáP án và giải thích đề 6
đáP án và giải thích đề 6đáP án và giải thích đề 6
đáP án và giải thích đề 6
 
đáP án và giải thích đề 14
đáP án và giải thích đề 14đáP án và giải thích đề 14
đáP án và giải thích đề 14
 
đáP án và giải thích đề 9
đáP án và giải thích đề 9đáP án và giải thích đề 9
đáP án và giải thích đề 9
 
đáP án và giải thích đề 4
đáP án và giải thích đề 4đáP án và giải thích đề 4
đáP án và giải thích đề 4
 
Giai chi tiet de thi dh khoi d t anh 2011
Giai chi tiet de thi dh khoi d t anh 2011Giai chi tiet de thi dh khoi d t anh 2011
Giai chi tiet de thi dh khoi d t anh 2011
 
đáP án và giải thích đề 17
đáP án và giải thích đề 17đáP án và giải thích đề 17
đáP án và giải thích đề 17
 
đáP án và giải thích đề 18
đáP án và giải thích đề 18đáP án và giải thích đề 18
đáP án và giải thích đề 18
 
đáP án và giải thích đề 32
đáP án và giải thích đề 32đáP án và giải thích đề 32
đáP án và giải thích đề 32
 
đáP án và giải thích đề 13
đáP án và giải thích đề 13đáP án và giải thích đề 13
đáP án và giải thích đề 13
 
đáP án và giải thích đề 26
đáP án và giải thích đề 26đáP án và giải thích đề 26
đáP án và giải thích đề 26
 
đáP án và giải thích đề 11
đáP án và giải thích đề 11đáP án và giải thích đề 11
đáP án và giải thích đề 11
 

Mehr von Nguyen Thanh Tu Collection

ĐỀ CƯƠNG + TEST ÔN TẬP CUỐI KÌ 2 TIẾNG ANH 11 - GLOBAL SUCCESS (THEO CHUẨN MI...
ĐỀ CƯƠNG + TEST ÔN TẬP CUỐI KÌ 2 TIẾNG ANH 11 - GLOBAL SUCCESS (THEO CHUẨN MI...ĐỀ CƯƠNG + TEST ÔN TẬP CUỐI KÌ 2 TIẾNG ANH 11 - GLOBAL SUCCESS (THEO CHUẨN MI...
ĐỀ CƯƠNG + TEST ÔN TẬP CUỐI KÌ 2 TIẾNG ANH 11 - GLOBAL SUCCESS (THEO CHUẨN MI...Nguyen Thanh Tu Collection
 
VẬN DỤNG KIẾN THỨC LIÊN MÔN TRONG GIẢI BÀI TẬP ÔN THI THPTQG MÔN SINH HỌC - H...
VẬN DỤNG KIẾN THỨC LIÊN MÔN TRONG GIẢI BÀI TẬP ÔN THI THPTQG MÔN SINH HỌC - H...VẬN DỤNG KIẾN THỨC LIÊN MÔN TRONG GIẢI BÀI TẬP ÔN THI THPTQG MÔN SINH HỌC - H...
VẬN DỤNG KIẾN THỨC LIÊN MÔN TRONG GIẢI BÀI TẬP ÔN THI THPTQG MÔN SINH HỌC - H...Nguyen Thanh Tu Collection
 
ĐỀ THAM KHẢO THEO HƯỚNG MINH HỌA 2025 KIỂM TRA CUỐI HỌC KÌ 2 NĂM HỌC 2023-202...
ĐỀ THAM KHẢO THEO HƯỚNG MINH HỌA 2025 KIỂM TRA CUỐI HỌC KÌ 2 NĂM HỌC 2023-202...ĐỀ THAM KHẢO THEO HƯỚNG MINH HỌA 2025 KIỂM TRA CUỐI HỌC KÌ 2 NĂM HỌC 2023-202...
ĐỀ THAM KHẢO THEO HƯỚNG MINH HỌA 2025 KIỂM TRA CUỐI HỌC KÌ 2 NĂM HỌC 2023-202...Nguyen Thanh Tu Collection
 
50 ĐỀ PHÁT TRIỂN THEO CẤU TRÚC ĐỀ MINH HỌA BGD NGÀY 22-3-2024 KỲ THI TỐT NGHI...
50 ĐỀ PHÁT TRIỂN THEO CẤU TRÚC ĐỀ MINH HỌA BGD NGÀY 22-3-2024 KỲ THI TỐT NGHI...50 ĐỀ PHÁT TRIỂN THEO CẤU TRÚC ĐỀ MINH HỌA BGD NGÀY 22-3-2024 KỲ THI TỐT NGHI...
50 ĐỀ PHÁT TRIỂN THEO CẤU TRÚC ĐỀ MINH HỌA BGD NGÀY 22-3-2024 KỲ THI TỐT NGHI...Nguyen Thanh Tu Collection
 
30 ĐỀ PHÁT TRIỂN THEO CẤU TRÚC ĐỀ MINH HỌA BGD NGÀY 22-3-2024 KỲ THI TỐT NGHI...
30 ĐỀ PHÁT TRIỂN THEO CẤU TRÚC ĐỀ MINH HỌA BGD NGÀY 22-3-2024 KỲ THI TỐT NGHI...30 ĐỀ PHÁT TRIỂN THEO CẤU TRÚC ĐỀ MINH HỌA BGD NGÀY 22-3-2024 KỲ THI TỐT NGHI...
30 ĐỀ PHÁT TRIỂN THEO CẤU TRÚC ĐỀ MINH HỌA BGD NGÀY 22-3-2024 KỲ THI TỐT NGHI...Nguyen Thanh Tu Collection
 
BỘ ĐỀ CHÍNH THỨC + TÁCH ĐỀ + ĐỀ LUYỆN THI VÀO LỚP 10 CHUYÊN TOÁN CÁC TỈNH NĂM...
BỘ ĐỀ CHÍNH THỨC + TÁCH ĐỀ + ĐỀ LUYỆN THI VÀO LỚP 10 CHUYÊN TOÁN CÁC TỈNH NĂM...BỘ ĐỀ CHÍNH THỨC + TÁCH ĐỀ + ĐỀ LUYỆN THI VÀO LỚP 10 CHUYÊN TOÁN CÁC TỈNH NĂM...
BỘ ĐỀ CHÍNH THỨC + TÁCH ĐỀ + ĐỀ LUYỆN THI VÀO LỚP 10 CHUYÊN TOÁN CÁC TỈNH NĂM...Nguyen Thanh Tu Collection
 
GIÁO TRÌNH BỒI DƯỠNG HỌC SINH GIỎI THCS VÀ THI VÀO 10 THPT CHUYÊN MÔN TIẾNG A...
GIÁO TRÌNH BỒI DƯỠNG HỌC SINH GIỎI THCS VÀ THI VÀO 10 THPT CHUYÊN MÔN TIẾNG A...GIÁO TRÌNH BỒI DƯỠNG HỌC SINH GIỎI THCS VÀ THI VÀO 10 THPT CHUYÊN MÔN TIẾNG A...
GIÁO TRÌNH BỒI DƯỠNG HỌC SINH GIỎI THCS VÀ THI VÀO 10 THPT CHUYÊN MÔN TIẾNG A...Nguyen Thanh Tu Collection
 
30 ĐỀ PHÁT TRIỂN THEO CẤU TRÚC ĐỀ MINH HỌA BGD NGÀY 22-3-2024 KỲ THI TỐT NGHI...
30 ĐỀ PHÁT TRIỂN THEO CẤU TRÚC ĐỀ MINH HỌA BGD NGÀY 22-3-2024 KỲ THI TỐT NGHI...30 ĐỀ PHÁT TRIỂN THEO CẤU TRÚC ĐỀ MINH HỌA BGD NGÀY 22-3-2024 KỲ THI TỐT NGHI...
30 ĐỀ PHÁT TRIỂN THEO CẤU TRÚC ĐỀ MINH HỌA BGD NGÀY 22-3-2024 KỲ THI TỐT NGHI...Nguyen Thanh Tu Collection
 
31 ĐỀ THI THỬ VÀO LỚP 10 - TIẾNG ANH - FORM MỚI 2025 - 40 CÂU HỎI - BÙI VĂN V...
31 ĐỀ THI THỬ VÀO LỚP 10 - TIẾNG ANH - FORM MỚI 2025 - 40 CÂU HỎI - BÙI VĂN V...31 ĐỀ THI THỬ VÀO LỚP 10 - TIẾNG ANH - FORM MỚI 2025 - 40 CÂU HỎI - BÙI VĂN V...
31 ĐỀ THI THỬ VÀO LỚP 10 - TIẾNG ANH - FORM MỚI 2025 - 40 CÂU HỎI - BÙI VĂN V...Nguyen Thanh Tu Collection
 
ĐỀ CƯƠNG + TEST ÔN TẬP CUỐI KÌ 2 TIẾNG ANH 11 - GLOBAL SUCCESS (THEO CHUẨN MI...
ĐỀ CƯƠNG + TEST ÔN TẬP CUỐI KÌ 2 TIẾNG ANH 11 - GLOBAL SUCCESS (THEO CHUẨN MI...ĐỀ CƯƠNG + TEST ÔN TẬP CUỐI KÌ 2 TIẾNG ANH 11 - GLOBAL SUCCESS (THEO CHUẨN MI...
ĐỀ CƯƠNG + TEST ÔN TẬP CUỐI KÌ 2 TIẾNG ANH 11 - GLOBAL SUCCESS (THEO CHUẨN MI...Nguyen Thanh Tu Collection
 
GIÁO ÁN KẾ HOẠCH BÀI DẠY CÔNG NGHỆ 8 KẾT NỐI TRI THỨC - CẢ NĂM THEO CÔNG VĂN ...
GIÁO ÁN KẾ HOẠCH BÀI DẠY CÔNG NGHỆ 8 KẾT NỐI TRI THỨC - CẢ NĂM THEO CÔNG VĂN ...GIÁO ÁN KẾ HOẠCH BÀI DẠY CÔNG NGHỆ 8 KẾT NỐI TRI THỨC - CẢ NĂM THEO CÔNG VĂN ...
GIÁO ÁN KẾ HOẠCH BÀI DẠY CÔNG NGHỆ 8 KẾT NỐI TRI THỨC - CẢ NĂM THEO CÔNG VĂN ...Nguyen Thanh Tu Collection
 
BỘ ĐỀ KIỂM TRA CUỐI KÌ 2 TIẾNG ANH I-LEARN SMART START LỚP 3, 4 NĂM HỌC 2023-...
BỘ ĐỀ KIỂM TRA CUỐI KÌ 2 TIẾNG ANH I-LEARN SMART START LỚP 3, 4 NĂM HỌC 2023-...BỘ ĐỀ KIỂM TRA CUỐI KÌ 2 TIẾNG ANH I-LEARN SMART START LỚP 3, 4 NĂM HỌC 2023-...
BỘ ĐỀ KIỂM TRA CUỐI KÌ 2 TIẾNG ANH I-LEARN SMART START LỚP 3, 4 NĂM HỌC 2023-...Nguyen Thanh Tu Collection
 
BÀI TẬP BỔ TRỢ TIẾNG ANH 11 THEO ĐƠN VỊ BÀI HỌC - CẢ NĂM - CÓ FILE NGHE (GLOB...
BÀI TẬP BỔ TRỢ TIẾNG ANH 11 THEO ĐƠN VỊ BÀI HỌC - CẢ NĂM - CÓ FILE NGHE (GLOB...BÀI TẬP BỔ TRỢ TIẾNG ANH 11 THEO ĐƠN VỊ BÀI HỌC - CẢ NĂM - CÓ FILE NGHE (GLOB...
BÀI TẬP BỔ TRỢ TIẾNG ANH 11 THEO ĐƠN VỊ BÀI HỌC - CẢ NĂM - CÓ FILE NGHE (GLOB...Nguyen Thanh Tu Collection
 
BÀI TẬP BỔ TRỢ TIẾNG ANH 8 - I-LEARN SMART WORLD - CẢ NĂM - CÓ FILE NGHE (BẢN...
BÀI TẬP BỔ TRỢ TIẾNG ANH 8 - I-LEARN SMART WORLD - CẢ NĂM - CÓ FILE NGHE (BẢN...BÀI TẬP BỔ TRỢ TIẾNG ANH 8 - I-LEARN SMART WORLD - CẢ NĂM - CÓ FILE NGHE (BẢN...
BÀI TẬP BỔ TRỢ TIẾNG ANH 8 - I-LEARN SMART WORLD - CẢ NĂM - CÓ FILE NGHE (BẢN...Nguyen Thanh Tu Collection
 
GIÁO ÁN KẾ HOẠCH BÀI DẠY TOÁN 8 CÁNH DIỀU - CẢ NĂM THEO CÔNG VĂN 5512 (2 CỘT)...
GIÁO ÁN KẾ HOẠCH BÀI DẠY TOÁN 8 CÁNH DIỀU - CẢ NĂM THEO CÔNG VĂN 5512 (2 CỘT)...GIÁO ÁN KẾ HOẠCH BÀI DẠY TOÁN 8 CÁNH DIỀU - CẢ NĂM THEO CÔNG VĂN 5512 (2 CỘT)...
GIÁO ÁN KẾ HOẠCH BÀI DẠY TOÁN 8 CÁNH DIỀU - CẢ NĂM THEO CÔNG VĂN 5512 (2 CỘT)...Nguyen Thanh Tu Collection
 
GIÁO ÁN KẾ HOẠCH BÀI DẠY TOÁN 8 KẾT NỐI TRI THỨC - CẢ NĂM THEO CÔNG VĂN 5512 ...
GIÁO ÁN KẾ HOẠCH BÀI DẠY TOÁN 8 KẾT NỐI TRI THỨC - CẢ NĂM THEO CÔNG VĂN 5512 ...GIÁO ÁN KẾ HOẠCH BÀI DẠY TOÁN 8 KẾT NỐI TRI THỨC - CẢ NĂM THEO CÔNG VĂN 5512 ...
GIÁO ÁN KẾ HOẠCH BÀI DẠY TOÁN 8 KẾT NỐI TRI THỨC - CẢ NĂM THEO CÔNG VĂN 5512 ...Nguyen Thanh Tu Collection
 
CHUYÊN ĐỀ ÔN THEO CÂU CHO HỌC SINH LỚP 12 ĐỂ ĐẠT ĐIỂM 5+ THI TỐT NGHIỆP THPT ...
CHUYÊN ĐỀ ÔN THEO CÂU CHO HỌC SINH LỚP 12 ĐỂ ĐẠT ĐIỂM 5+ THI TỐT NGHIỆP THPT ...CHUYÊN ĐỀ ÔN THEO CÂU CHO HỌC SINH LỚP 12 ĐỂ ĐẠT ĐIỂM 5+ THI TỐT NGHIỆP THPT ...
CHUYÊN ĐỀ ÔN THEO CÂU CHO HỌC SINH LỚP 12 ĐỂ ĐẠT ĐIỂM 5+ THI TỐT NGHIỆP THPT ...Nguyen Thanh Tu Collection
 
TỔNG HỢP HƠN 100 ĐỀ THI THỬ TỐT NGHIỆP THPT HÓA HỌC 2024 - TỪ CÁC TRƯỜNG, TRƯ...
TỔNG HỢP HƠN 100 ĐỀ THI THỬ TỐT NGHIỆP THPT HÓA HỌC 2024 - TỪ CÁC TRƯỜNG, TRƯ...TỔNG HỢP HƠN 100 ĐỀ THI THỬ TỐT NGHIỆP THPT HÓA HỌC 2024 - TỪ CÁC TRƯỜNG, TRƯ...
TỔNG HỢP HƠN 100 ĐỀ THI THỬ TỐT NGHIỆP THPT HÓA HỌC 2024 - TỪ CÁC TRƯỜNG, TRƯ...Nguyen Thanh Tu Collection
 
GIÁO ÁN KẾ HOẠCH BÀI DẠY ĐỊA LÍ 8 KẾT NỐI TRI THỨC - CẢ NĂM THEO CÔNG VĂN 551...
GIÁO ÁN KẾ HOẠCH BÀI DẠY ĐỊA LÍ 8 KẾT NỐI TRI THỨC - CẢ NĂM THEO CÔNG VĂN 551...GIÁO ÁN KẾ HOẠCH BÀI DẠY ĐỊA LÍ 8 KẾT NỐI TRI THỨC - CẢ NĂM THEO CÔNG VĂN 551...
GIÁO ÁN KẾ HOẠCH BÀI DẠY ĐỊA LÍ 8 KẾT NỐI TRI THỨC - CẢ NĂM THEO CÔNG VĂN 551...Nguyen Thanh Tu Collection
 
BÀI TẬP BỔ TRỢ TIẾNG ANH LỚP 8 CẢ NĂM - FRIENDS PLUS - BẢN ĐẸP - CÓ FILE NGH...
BÀI TẬP BỔ TRỢ TIẾNG ANH LỚP 8 CẢ NĂM - FRIENDS PLUS - BẢN ĐẸP - CÓ FILE NGH...BÀI TẬP BỔ TRỢ TIẾNG ANH LỚP 8 CẢ NĂM - FRIENDS PLUS - BẢN ĐẸP - CÓ FILE NGH...
BÀI TẬP BỔ TRỢ TIẾNG ANH LỚP 8 CẢ NĂM - FRIENDS PLUS - BẢN ĐẸP - CÓ FILE NGH...Nguyen Thanh Tu Collection
 

Mehr von Nguyen Thanh Tu Collection (20)

ĐỀ CƯƠNG + TEST ÔN TẬP CUỐI KÌ 2 TIẾNG ANH 11 - GLOBAL SUCCESS (THEO CHUẨN MI...
ĐỀ CƯƠNG + TEST ÔN TẬP CUỐI KÌ 2 TIẾNG ANH 11 - GLOBAL SUCCESS (THEO CHUẨN MI...ĐỀ CƯƠNG + TEST ÔN TẬP CUỐI KÌ 2 TIẾNG ANH 11 - GLOBAL SUCCESS (THEO CHUẨN MI...
ĐỀ CƯƠNG + TEST ÔN TẬP CUỐI KÌ 2 TIẾNG ANH 11 - GLOBAL SUCCESS (THEO CHUẨN MI...
 
VẬN DỤNG KIẾN THỨC LIÊN MÔN TRONG GIẢI BÀI TẬP ÔN THI THPTQG MÔN SINH HỌC - H...
VẬN DỤNG KIẾN THỨC LIÊN MÔN TRONG GIẢI BÀI TẬP ÔN THI THPTQG MÔN SINH HỌC - H...VẬN DỤNG KIẾN THỨC LIÊN MÔN TRONG GIẢI BÀI TẬP ÔN THI THPTQG MÔN SINH HỌC - H...
VẬN DỤNG KIẾN THỨC LIÊN MÔN TRONG GIẢI BÀI TẬP ÔN THI THPTQG MÔN SINH HỌC - H...
 
ĐỀ THAM KHẢO THEO HƯỚNG MINH HỌA 2025 KIỂM TRA CUỐI HỌC KÌ 2 NĂM HỌC 2023-202...
ĐỀ THAM KHẢO THEO HƯỚNG MINH HỌA 2025 KIỂM TRA CUỐI HỌC KÌ 2 NĂM HỌC 2023-202...ĐỀ THAM KHẢO THEO HƯỚNG MINH HỌA 2025 KIỂM TRA CUỐI HỌC KÌ 2 NĂM HỌC 2023-202...
ĐỀ THAM KHẢO THEO HƯỚNG MINH HỌA 2025 KIỂM TRA CUỐI HỌC KÌ 2 NĂM HỌC 2023-202...
 
50 ĐỀ PHÁT TRIỂN THEO CẤU TRÚC ĐỀ MINH HỌA BGD NGÀY 22-3-2024 KỲ THI TỐT NGHI...
50 ĐỀ PHÁT TRIỂN THEO CẤU TRÚC ĐỀ MINH HỌA BGD NGÀY 22-3-2024 KỲ THI TỐT NGHI...50 ĐỀ PHÁT TRIỂN THEO CẤU TRÚC ĐỀ MINH HỌA BGD NGÀY 22-3-2024 KỲ THI TỐT NGHI...
50 ĐỀ PHÁT TRIỂN THEO CẤU TRÚC ĐỀ MINH HỌA BGD NGÀY 22-3-2024 KỲ THI TỐT NGHI...
 
30 ĐỀ PHÁT TRIỂN THEO CẤU TRÚC ĐỀ MINH HỌA BGD NGÀY 22-3-2024 KỲ THI TỐT NGHI...
30 ĐỀ PHÁT TRIỂN THEO CẤU TRÚC ĐỀ MINH HỌA BGD NGÀY 22-3-2024 KỲ THI TỐT NGHI...30 ĐỀ PHÁT TRIỂN THEO CẤU TRÚC ĐỀ MINH HỌA BGD NGÀY 22-3-2024 KỲ THI TỐT NGHI...
30 ĐỀ PHÁT TRIỂN THEO CẤU TRÚC ĐỀ MINH HỌA BGD NGÀY 22-3-2024 KỲ THI TỐT NGHI...
 
BỘ ĐỀ CHÍNH THỨC + TÁCH ĐỀ + ĐỀ LUYỆN THI VÀO LỚP 10 CHUYÊN TOÁN CÁC TỈNH NĂM...
BỘ ĐỀ CHÍNH THỨC + TÁCH ĐỀ + ĐỀ LUYỆN THI VÀO LỚP 10 CHUYÊN TOÁN CÁC TỈNH NĂM...BỘ ĐỀ CHÍNH THỨC + TÁCH ĐỀ + ĐỀ LUYỆN THI VÀO LỚP 10 CHUYÊN TOÁN CÁC TỈNH NĂM...
BỘ ĐỀ CHÍNH THỨC + TÁCH ĐỀ + ĐỀ LUYỆN THI VÀO LỚP 10 CHUYÊN TOÁN CÁC TỈNH NĂM...
 
GIÁO TRÌNH BỒI DƯỠNG HỌC SINH GIỎI THCS VÀ THI VÀO 10 THPT CHUYÊN MÔN TIẾNG A...
GIÁO TRÌNH BỒI DƯỠNG HỌC SINH GIỎI THCS VÀ THI VÀO 10 THPT CHUYÊN MÔN TIẾNG A...GIÁO TRÌNH BỒI DƯỠNG HỌC SINH GIỎI THCS VÀ THI VÀO 10 THPT CHUYÊN MÔN TIẾNG A...
GIÁO TRÌNH BỒI DƯỠNG HỌC SINH GIỎI THCS VÀ THI VÀO 10 THPT CHUYÊN MÔN TIẾNG A...
 
30 ĐỀ PHÁT TRIỂN THEO CẤU TRÚC ĐỀ MINH HỌA BGD NGÀY 22-3-2024 KỲ THI TỐT NGHI...
30 ĐỀ PHÁT TRIỂN THEO CẤU TRÚC ĐỀ MINH HỌA BGD NGÀY 22-3-2024 KỲ THI TỐT NGHI...30 ĐỀ PHÁT TRIỂN THEO CẤU TRÚC ĐỀ MINH HỌA BGD NGÀY 22-3-2024 KỲ THI TỐT NGHI...
30 ĐỀ PHÁT TRIỂN THEO CẤU TRÚC ĐỀ MINH HỌA BGD NGÀY 22-3-2024 KỲ THI TỐT NGHI...
 
31 ĐỀ THI THỬ VÀO LỚP 10 - TIẾNG ANH - FORM MỚI 2025 - 40 CÂU HỎI - BÙI VĂN V...
31 ĐỀ THI THỬ VÀO LỚP 10 - TIẾNG ANH - FORM MỚI 2025 - 40 CÂU HỎI - BÙI VĂN V...31 ĐỀ THI THỬ VÀO LỚP 10 - TIẾNG ANH - FORM MỚI 2025 - 40 CÂU HỎI - BÙI VĂN V...
31 ĐỀ THI THỬ VÀO LỚP 10 - TIẾNG ANH - FORM MỚI 2025 - 40 CÂU HỎI - BÙI VĂN V...
 
ĐỀ CƯƠNG + TEST ÔN TẬP CUỐI KÌ 2 TIẾNG ANH 11 - GLOBAL SUCCESS (THEO CHUẨN MI...
ĐỀ CƯƠNG + TEST ÔN TẬP CUỐI KÌ 2 TIẾNG ANH 11 - GLOBAL SUCCESS (THEO CHUẨN MI...ĐỀ CƯƠNG + TEST ÔN TẬP CUỐI KÌ 2 TIẾNG ANH 11 - GLOBAL SUCCESS (THEO CHUẨN MI...
ĐỀ CƯƠNG + TEST ÔN TẬP CUỐI KÌ 2 TIẾNG ANH 11 - GLOBAL SUCCESS (THEO CHUẨN MI...
 
GIÁO ÁN KẾ HOẠCH BÀI DẠY CÔNG NGHỆ 8 KẾT NỐI TRI THỨC - CẢ NĂM THEO CÔNG VĂN ...
GIÁO ÁN KẾ HOẠCH BÀI DẠY CÔNG NGHỆ 8 KẾT NỐI TRI THỨC - CẢ NĂM THEO CÔNG VĂN ...GIÁO ÁN KẾ HOẠCH BÀI DẠY CÔNG NGHỆ 8 KẾT NỐI TRI THỨC - CẢ NĂM THEO CÔNG VĂN ...
GIÁO ÁN KẾ HOẠCH BÀI DẠY CÔNG NGHỆ 8 KẾT NỐI TRI THỨC - CẢ NĂM THEO CÔNG VĂN ...
 
BỘ ĐỀ KIỂM TRA CUỐI KÌ 2 TIẾNG ANH I-LEARN SMART START LỚP 3, 4 NĂM HỌC 2023-...
BỘ ĐỀ KIỂM TRA CUỐI KÌ 2 TIẾNG ANH I-LEARN SMART START LỚP 3, 4 NĂM HỌC 2023-...BỘ ĐỀ KIỂM TRA CUỐI KÌ 2 TIẾNG ANH I-LEARN SMART START LỚP 3, 4 NĂM HỌC 2023-...
BỘ ĐỀ KIỂM TRA CUỐI KÌ 2 TIẾNG ANH I-LEARN SMART START LỚP 3, 4 NĂM HỌC 2023-...
 
BÀI TẬP BỔ TRỢ TIẾNG ANH 11 THEO ĐƠN VỊ BÀI HỌC - CẢ NĂM - CÓ FILE NGHE (GLOB...
BÀI TẬP BỔ TRỢ TIẾNG ANH 11 THEO ĐƠN VỊ BÀI HỌC - CẢ NĂM - CÓ FILE NGHE (GLOB...BÀI TẬP BỔ TRỢ TIẾNG ANH 11 THEO ĐƠN VỊ BÀI HỌC - CẢ NĂM - CÓ FILE NGHE (GLOB...
BÀI TẬP BỔ TRỢ TIẾNG ANH 11 THEO ĐƠN VỊ BÀI HỌC - CẢ NĂM - CÓ FILE NGHE (GLOB...
 
BÀI TẬP BỔ TRỢ TIẾNG ANH 8 - I-LEARN SMART WORLD - CẢ NĂM - CÓ FILE NGHE (BẢN...
BÀI TẬP BỔ TRỢ TIẾNG ANH 8 - I-LEARN SMART WORLD - CẢ NĂM - CÓ FILE NGHE (BẢN...BÀI TẬP BỔ TRỢ TIẾNG ANH 8 - I-LEARN SMART WORLD - CẢ NĂM - CÓ FILE NGHE (BẢN...
BÀI TẬP BỔ TRỢ TIẾNG ANH 8 - I-LEARN SMART WORLD - CẢ NĂM - CÓ FILE NGHE (BẢN...
 
GIÁO ÁN KẾ HOẠCH BÀI DẠY TOÁN 8 CÁNH DIỀU - CẢ NĂM THEO CÔNG VĂN 5512 (2 CỘT)...
GIÁO ÁN KẾ HOẠCH BÀI DẠY TOÁN 8 CÁNH DIỀU - CẢ NĂM THEO CÔNG VĂN 5512 (2 CỘT)...GIÁO ÁN KẾ HOẠCH BÀI DẠY TOÁN 8 CÁNH DIỀU - CẢ NĂM THEO CÔNG VĂN 5512 (2 CỘT)...
GIÁO ÁN KẾ HOẠCH BÀI DẠY TOÁN 8 CÁNH DIỀU - CẢ NĂM THEO CÔNG VĂN 5512 (2 CỘT)...
 
GIÁO ÁN KẾ HOẠCH BÀI DẠY TOÁN 8 KẾT NỐI TRI THỨC - CẢ NĂM THEO CÔNG VĂN 5512 ...
GIÁO ÁN KẾ HOẠCH BÀI DẠY TOÁN 8 KẾT NỐI TRI THỨC - CẢ NĂM THEO CÔNG VĂN 5512 ...GIÁO ÁN KẾ HOẠCH BÀI DẠY TOÁN 8 KẾT NỐI TRI THỨC - CẢ NĂM THEO CÔNG VĂN 5512 ...
GIÁO ÁN KẾ HOẠCH BÀI DẠY TOÁN 8 KẾT NỐI TRI THỨC - CẢ NĂM THEO CÔNG VĂN 5512 ...
 
CHUYÊN ĐỀ ÔN THEO CÂU CHO HỌC SINH LỚP 12 ĐỂ ĐẠT ĐIỂM 5+ THI TỐT NGHIỆP THPT ...
CHUYÊN ĐỀ ÔN THEO CÂU CHO HỌC SINH LỚP 12 ĐỂ ĐẠT ĐIỂM 5+ THI TỐT NGHIỆP THPT ...CHUYÊN ĐỀ ÔN THEO CÂU CHO HỌC SINH LỚP 12 ĐỂ ĐẠT ĐIỂM 5+ THI TỐT NGHIỆP THPT ...
CHUYÊN ĐỀ ÔN THEO CÂU CHO HỌC SINH LỚP 12 ĐỂ ĐẠT ĐIỂM 5+ THI TỐT NGHIỆP THPT ...
 
TỔNG HỢP HƠN 100 ĐỀ THI THỬ TỐT NGHIỆP THPT HÓA HỌC 2024 - TỪ CÁC TRƯỜNG, TRƯ...
TỔNG HỢP HƠN 100 ĐỀ THI THỬ TỐT NGHIỆP THPT HÓA HỌC 2024 - TỪ CÁC TRƯỜNG, TRƯ...TỔNG HỢP HƠN 100 ĐỀ THI THỬ TỐT NGHIỆP THPT HÓA HỌC 2024 - TỪ CÁC TRƯỜNG, TRƯ...
TỔNG HỢP HƠN 100 ĐỀ THI THỬ TỐT NGHIỆP THPT HÓA HỌC 2024 - TỪ CÁC TRƯỜNG, TRƯ...
 
GIÁO ÁN KẾ HOẠCH BÀI DẠY ĐỊA LÍ 8 KẾT NỐI TRI THỨC - CẢ NĂM THEO CÔNG VĂN 551...
GIÁO ÁN KẾ HOẠCH BÀI DẠY ĐỊA LÍ 8 KẾT NỐI TRI THỨC - CẢ NĂM THEO CÔNG VĂN 551...GIÁO ÁN KẾ HOẠCH BÀI DẠY ĐỊA LÍ 8 KẾT NỐI TRI THỨC - CẢ NĂM THEO CÔNG VĂN 551...
GIÁO ÁN KẾ HOẠCH BÀI DẠY ĐỊA LÍ 8 KẾT NỐI TRI THỨC - CẢ NĂM THEO CÔNG VĂN 551...
 
BÀI TẬP BỔ TRỢ TIẾNG ANH LỚP 8 CẢ NĂM - FRIENDS PLUS - BẢN ĐẸP - CÓ FILE NGH...
BÀI TẬP BỔ TRỢ TIẾNG ANH LỚP 8 CẢ NĂM - FRIENDS PLUS - BẢN ĐẸP - CÓ FILE NGH...BÀI TẬP BỔ TRỢ TIẾNG ANH LỚP 8 CẢ NĂM - FRIENDS PLUS - BẢN ĐẸP - CÓ FILE NGH...
BÀI TẬP BỔ TRỢ TIẾNG ANH LỚP 8 CẢ NĂM - FRIENDS PLUS - BẢN ĐẸP - CÓ FILE NGH...
 

Kürzlich hochgeladen

Tổng hợp Ngữ pháp Tiếng Anh 11 cho học sinh.docx
Tổng hợp Ngữ pháp Tiếng Anh 11 cho học sinh.docxTổng hợp Ngữ pháp Tiếng Anh 11 cho học sinh.docx
Tổng hợp Ngữ pháp Tiếng Anh 11 cho học sinh.docxTrangL188166
 
.................KHTN 9....................................Viet Nam.......
.................KHTN 9....................................Viet Nam........................KHTN 9....................................Viet Nam.......
.................KHTN 9....................................Viet Nam.......thoa051989
 
Giáo trình xã hội học Thể dục Thể thao hay
Giáo trình xã hội học Thể dục Thể thao hayGiáo trình xã hội học Thể dục Thể thao hay
Giáo trình xã hội học Thể dục Thể thao hayLcTh15
 
trò chơi về môn học tư tưởng hồ chí minh
trò chơi về môn học tư tưởng hồ chí minhtrò chơi về môn học tư tưởng hồ chí minh
trò chơi về môn học tư tưởng hồ chí minhNguynHuTh6
 
CH glucid university of Pham Ngoc Thach- v7.pdf
CH glucid university of Pham Ngoc Thach- v7.pdfCH glucid university of Pham Ngoc Thach- v7.pdf
CH glucid university of Pham Ngoc Thach- v7.pdfSuperJudy1
 
Báo cáo thực hành Quản lý kinh tế dược.pptx
Báo cáo thực hành Quản lý kinh tế dược.pptxBáo cáo thực hành Quản lý kinh tế dược.pptx
Báo cáo thực hành Quản lý kinh tế dược.pptxhoangvubaongoc112011
 
GIẢI-ĐỀ-CƯƠNG-NHẬP-MÔN-KHOA-HỌC-XÃ-HỘI-VÀ-NHÂN-VĂN-KHIÊM-BK69.pdf
GIẢI-ĐỀ-CƯƠNG-NHẬP-MÔN-KHOA-HỌC-XÃ-HỘI-VÀ-NHÂN-VĂN-KHIÊM-BK69.pdfGIẢI-ĐỀ-CƯƠNG-NHẬP-MÔN-KHOA-HỌC-XÃ-HỘI-VÀ-NHÂN-VĂN-KHIÊM-BK69.pdf
GIẢI-ĐỀ-CƯƠNG-NHẬP-MÔN-KHOA-HỌC-XÃ-HỘI-VÀ-NHÂN-VĂN-KHIÊM-BK69.pdfHngNguyn271079
 
NỘI DUNG HỌC THI ôn thi môn LỊCH SỬ ĐẢNG.docx
NỘI DUNG HỌC THI ôn thi môn LỊCH SỬ ĐẢNG.docxNỘI DUNG HỌC THI ôn thi môn LỊCH SỬ ĐẢNG.docx
NỘI DUNG HỌC THI ôn thi môn LỊCH SỬ ĐẢNG.docx7E26NguynThThyLinh
 
Tài liệu kỹ thuật điều hòa Panasonic - Điện lạnh Bách Khoa
Tài liệu kỹ thuật điều hòa Panasonic - Điện lạnh Bách KhoaTài liệu kỹ thuật điều hòa Panasonic - Điện lạnh Bách Khoa
Tài liệu kỹ thuật điều hòa Panasonic - Điện lạnh Bách KhoaKhiNguynCngtyTNHH
 

Kürzlich hochgeladen (9)

Tổng hợp Ngữ pháp Tiếng Anh 11 cho học sinh.docx
Tổng hợp Ngữ pháp Tiếng Anh 11 cho học sinh.docxTổng hợp Ngữ pháp Tiếng Anh 11 cho học sinh.docx
Tổng hợp Ngữ pháp Tiếng Anh 11 cho học sinh.docx
 
.................KHTN 9....................................Viet Nam.......
.................KHTN 9....................................Viet Nam........................KHTN 9....................................Viet Nam.......
.................KHTN 9....................................Viet Nam.......
 
Giáo trình xã hội học Thể dục Thể thao hay
Giáo trình xã hội học Thể dục Thể thao hayGiáo trình xã hội học Thể dục Thể thao hay
Giáo trình xã hội học Thể dục Thể thao hay
 
trò chơi về môn học tư tưởng hồ chí minh
trò chơi về môn học tư tưởng hồ chí minhtrò chơi về môn học tư tưởng hồ chí minh
trò chơi về môn học tư tưởng hồ chí minh
 
CH glucid university of Pham Ngoc Thach- v7.pdf
CH glucid university of Pham Ngoc Thach- v7.pdfCH glucid university of Pham Ngoc Thach- v7.pdf
CH glucid university of Pham Ngoc Thach- v7.pdf
 
Báo cáo thực hành Quản lý kinh tế dược.pptx
Báo cáo thực hành Quản lý kinh tế dược.pptxBáo cáo thực hành Quản lý kinh tế dược.pptx
Báo cáo thực hành Quản lý kinh tế dược.pptx
 
GIẢI-ĐỀ-CƯƠNG-NHẬP-MÔN-KHOA-HỌC-XÃ-HỘI-VÀ-NHÂN-VĂN-KHIÊM-BK69.pdf
GIẢI-ĐỀ-CƯƠNG-NHẬP-MÔN-KHOA-HỌC-XÃ-HỘI-VÀ-NHÂN-VĂN-KHIÊM-BK69.pdfGIẢI-ĐỀ-CƯƠNG-NHẬP-MÔN-KHOA-HỌC-XÃ-HỘI-VÀ-NHÂN-VĂN-KHIÊM-BK69.pdf
GIẢI-ĐỀ-CƯƠNG-NHẬP-MÔN-KHOA-HỌC-XÃ-HỘI-VÀ-NHÂN-VĂN-KHIÊM-BK69.pdf
 
NỘI DUNG HỌC THI ôn thi môn LỊCH SỬ ĐẢNG.docx
NỘI DUNG HỌC THI ôn thi môn LỊCH SỬ ĐẢNG.docxNỘI DUNG HỌC THI ôn thi môn LỊCH SỬ ĐẢNG.docx
NỘI DUNG HỌC THI ôn thi môn LỊCH SỬ ĐẢNG.docx
 
Tài liệu kỹ thuật điều hòa Panasonic - Điện lạnh Bách Khoa
Tài liệu kỹ thuật điều hòa Panasonic - Điện lạnh Bách KhoaTài liệu kỹ thuật điều hòa Panasonic - Điện lạnh Bách Khoa
Tài liệu kỹ thuật điều hòa Panasonic - Điện lạnh Bách Khoa
 

DE PHAT TRIEN THEO CAU TRUC DE MINH HOA 2022 MON TIENG ANH DE 6 10.pdf

  • 1. Question 1: The Botanical Garden was a very _________ place where we made a short tour round the garden and took some pictures of the whole class. A. beautify B. beautifully C. beautiful D. beautification A. beautify /’bju:tifai/ (v): làm đẹp B. beautifully /’bju:tifli/ (a): đẹp, hay C. beautiful /’bju:tifl/ (a): đẹp, hay D. beautification /bju:tifi’kei∫n/ (n): sự làm đẹp Căn cứ vào cấu trúc câu vì vậy vị trí ô trống cần điền là một tính từ để tạo thành một cụm danh từ. Adj + N Dịch nghĩa: Botanical Garden đã là một nơi rất đẹp, nơi mà chúng ta thực hiện những tua ngắn quanh vườn và chụp nhiều bức ảnh của toàn bộ lớp.
  • 2. Question 2 : She doesn’t imagine that I have much money, _______? A. does she B. do I C. haven’t I D. doesn’t she Dịch nghĩa: “Cô ấy không nghĩ rằng tôi có nhiều tiền phải không?” Câu bắt đầu với she + doesn’t thì thành lập hỏi đuôi sẽ dùng does she.
  • 3. Question 3: After feeling off for days, Tom finally went to see his doctor. A. Food B. color C. fitness D. balance Kiến thức về thành ngữ feel off color = under the weather: không được khỏe Tạm dịch: Sau vài ngày cảm thấy không khỏe, Tom cuối cùng mới chịu đi bác sĩ.
  • 4. . Question 4: This policewoman is famous ____ her courage and strong will. A. with B. for C. on D. about Dịch nghĩa: Nữ cảnh sát này nổi tiếng vì lòng can đảm và ý chí mạnh mẽ. => Theo cấu trúc: be famous for: nổi tiếng vì cái gì
  • 5. Question 5: There were 79 killed and 230 ________in a bomb explosion at the embassy. A. injured B. wounded C. hurt D. ached Đáp án B Kiến thức: về từ cùng trường nghĩa A. injured: bị thương thể chất do tai nạn giao thông, làm tổn thương đến cảm xúc cà tinh thân B. wounded: bị thương ngoài da do vũ khí trong chiến tranh hay bom đạn, bị xúc phạm C. hurt : bị tổn thương về thể xác D. ached: bị đau đau một bộ phận trên cơ thể do sử dụng quá nhiều đến nó) Tạm dịch: Có 79 người tử vọng và 230 người bị thương trong vụ nổ bom tại đại sứ quán.
  • 6. Question 6: ____ many international agreements affirming their human rights, women are still much more likely than men to be poor and illiterate. A. Although B. Despite C. Because D. Since Dịch nghĩa: Mặc dù nhiều xác nhận các thỏa thuận quốc tế quyền con người của họ, nữ giới vẫn nhiều hơn nam giới về số lượng nghèo và mù chữ. Xét các đáp án: A. Although + clause: mặc dù B. Despite + N/Ving: mặc dù C. Because + clause: bởi vì D. Since = because + clause: bởi vì *Note: “affirming” là danh động từ, có chức năng là một danh từ chứ không phải Ving, cùng với “agreements” tạo thành cụm danh từ, và cả cụm “many….rights” là một cụm danh từ hoàn thiện, do đó ta chọn B.
  • 7. Question 7: Lara goes to a (n) _____________ temple every week. A. great big ancient Buddhist B. Buddhist great big ancient C. ancient great big Buddhist D. big great ancient Buddhist *Theo quy tắc trật tự tính từ trong câu: OSASCOMP : great - Opinion; big - Size; ancient -A *Note: Buddhist temple (phr.n): ngôi đền tín đồ đạo Phật Tạm dịch: Lara tới ngôi đền tín đồ Phật cổ rất to và đẹp mỗi tuần.
  • 8. Question 8: The meeting appeared to be going well until Sarah ______________ the issue of holiday pay. A. brought about B. brought up C. brought off D. brought on Đáp án B A. brought about: đem lại, gây ra B. brought up: nuôi dưỡng, đề cập đến C. brought off: giải quyết việc khó D. brought on: dẫn đến, gây ra + bring sb on = help someone to improve, especially through training or practice + bring on something = to cause something to happen, esp. something bad Dịch nghĩa: Cuộc họp dường như đã diễn ra tốt đẹp cho đến khi Sarah đề cập đến vấn đề trợ cấp ngày nghỉ.
  • 9. Question 9: When I came to her house last night, she _____ dinner. A. is having B. was having C. has had D. had had Đáp án B – Căn cứ vào đây là mệnh đề trạng ngữ chỉ thời gian: When S + V (quá khứ đơn), S + V (quá khứ tiếp diễn). ( Diễn tả một hành động đang xảy ra thì có hành động khác xen vào). Vế chứa “when” chia thì quá khứ đơn => Vế còn lại chia thì quá khứ tiếp diễn. – Dịch: Khi tôi đến nhà cô ấy tối qua, cô ấy đang ăn tối.
  • 10. Question 10. _______, I will give her a gift. A. When he will come B. When she comes here C. Until he will come D. No sooner she cames 10. Đáp án B Kiến thức: Mệnh đề chỉ thời gian / Sự hòa hợp thì trong mệnh đề Giải thích: Mệnh đề chính chia thì tương lai => động từ trong mệnh đề chỉ thời gian chia thì hiện tại (hiện tại đơn, hiện tại hoàn thành). When +S+Vo/s/es +…, S+ will + Vo. Loại đáp án A,C, sai thì, và đáp D chưa đúng nghĩa .
  • 11. Question 11: We have been working hard. Let’s a break. A. make B. find C. do D. take Question 11. Chọn đáp án D Giải thích: take a break: nghỉ ngơi. Dịch nghĩa: Chúng ta đã làm việc thật chăm chỉ. Nghỉ ngơi thôi!
  • 12. Question 12: It is parents' duty and responsibility to ______ hands to take care of their children and give them a happy home. A. give B. hold C. join D. shake Câu 12: Đáp án C Kiến thức về cụm từ cố định give/ lend sb a hand = do sb a favor = help sb: giúp ai hold hands: nắm tay join hands = work together: chung tay, cùng nhau shake hands: bắt tay
  • 13. Question 13. Harry Potter and the Goblet of Fire ________ by J K Rowling. A. written B. was written C. wrote D. were written 13 (TH) Kiến thức: Bị động thì quá khứ đơn Giải thích: “Harry Potter and the Goblet of Fire”: chủ ngữ chỉ sự vật – nên câu ở trạng thái bị động của động từ “write” Cấu trúc câu bị động thì quá khứ đơn: S + was/ were + Vp2 (by + O). Tạm dịch: Harry Potter và the Goblet of Fire được viết bởi J K Rowling
  • 14. Question 14. ________ his composition, Louie handed it to his teacher. A. Having written B. Having been written C. Being written D. Writing Kiến thức: Mệnh đề phân tử / Rút gọn mệnh đề đồng ngữ Giải thích: Khi 2 mệnh đề có cùng chủ ngữ (she) thì có thể rút gọn 1 trong 2 mệnh đề về dạng: - V-ing / Having P2: nếu mệnh đề được rút gọn mang nghĩa chủ động - P2 (quá khứ phân từ): nếu mệnh đề được rút gọn mang nghĩa bị động Chủ ngữ “He” có thể làm chủ (tự thực hiện hành động “write” (viết) => nghĩa chủ động. Tạm dịch: Sau khi viết xong bài của mình, Louie giao nó cho giáo viên của mình.
  • 15. Question 15: The faster we walk, ____________ we will get there. A. the soonestB. the soon C. the more soon D. the sooner 15.Đáp án D HD: Dịch nghĩa: Chúng ta đi bộ càng nhanh thì chúng ta sẽ đến đó càng sớm. Xét các đáp án: A. the soonest→ Đây là cấu trúc so sánh kép nên chỗ trống cần điền từ có dạng so sánh hơn B. the soon → Đây là cấu trúc so sánh kép nên chỗ trống cần điền từ có dạng so sánh hơn C. the more soon → soon có 1 âm tiết nên ta sử dụng cấu trúc: adj/adv + đuôi -er + than D. the sooner → Cấu trúc so sánh kép khi nói về 2 người hoặc sự vật: The more/-er + S + V, the more/-er + S + V
  • 16. Question 16: "Why do you like pop music?” - “_______.” A. Pop music originated in the Western world. B. Because it is soft and beautiful. C. Westlife is a pop band. D. Since the 1950s, pop has had some new elements. Câu 16 Đáp án B “Tại sao bạn thích nhạc pop?” -"____________’’ A. Nhạc POP có nguồn gốc từ những nước phương Tây. B. Bởi vì nó nhẹ nhàng và hay. C. Westlife là một nhóm nhạc pop. D. Từ những năm 1950, nhạc pop đã có nhiều yếu tố mới.
  • 17. Question 17: “What’s your neighbourhood like?” - “_______." A. I live in Ha Noi. B. Sounds good. I like living there. C. It’s good. I love it. D. No, there is no hospital there. Câu 17 Đáp án C Tạm dịch: “Khu phố của bạn thế nào?” A. Tôi sống ở Hà Nội. B. Nghe tốt đấy. Tôi thích sống ở đây. C. Tốt lắm. Tôi yêu nó. D. Không, không có bệnh viện ở đây
  • 18. Question 18: A. occurB. prefer C. apply D. surface Từ “surface” có trọng âm rơi vào âm tiết thứ nhất Các từ còn lại có trọng âm rơi vào âm tiết thứ hai
  • 19. Question 19: A. domination B. adventure C. grammatical D. emergency Từ “domination” có trọng âm rơi vào âm tiết thứ 3 Các từ còn lại có trọng âm rơi vào âm tiết thứ 2
  • 20. Question 20: A. mended B. faced C. objected D. waited Từ “faced” có phần gạch chân phát âm là /t/ Các từ còn lại có phần gạch chân phát âm là /id/
  • 21. Question 21: A. family B. famine C. famous D. fabricate Từ “famous” có phần gạch chân phát âm là /ei/ Các từ còn lại có phần gạch chân phát âm là /æ
  • 22. Question 22: Life expectancy for both men and women has improved greatly in the past twenty years. A. Living standard B. Longevity C. Life Skills D. Lifeline Từ đồng nghĩa - Kiến thức về từ vựng Tạm dịch: Tuổi thọ của cả nam lẫn nữ đều đã tăng đáng kể trong vòng 20 năm qua. => Life expactancy: tuổi thọ Xét các đáp án: A. Living standard: tiêu chuẩn sống B. Longevity: tuổi thọ C. Life skills: kĩ năng sống D. Lifeline: dây an toàn, dây cứu đắm
  • 23. Question 23: Young college and university students participate in helping the poor who have suffered badly in wars or natural disasters. A. get involved in B. interfere with C. join hands D. come into Từ đồng nghĩa Tạm dịch: Những sinh viên cao đẳng, đại học tham gia giúp đỡ những người nghèo, người chịu ảnh hưởng nặng nề từ chiến tranh và thiên tai. => Participate in: tham gia Xét các đáp án: A. get involved in: tham gia B. interfere with: cản trở C. join hands: giúp đỡ D. come into: thừa kế
  • 24. Question 24: As a brilliant and mature studentMarie harboured the dream of a scientific career which was impossible for a woman at that time. A. Kept B. had C. abandoned D. deleted Câu 24: Đáp án C Từ trái nghĩa Tạm dịch: Là một sinh xuất sắc và chín chắn, Marie ấp ủ giấc mơ sự nghiệp khoa học, cái mà là không thể cho một người phụ nữ vào thời điểm đó. => harbour the dream of st: ấp ủ/nuôii dưỡng giấc mơ gì => Đáp án C (harbor >< abandon: từ bỏ)
  • 25. Question 25: After the tragic death of Pierre Curie in 1906, she not only took charge of educating her two children but also took the position which her husband had finally obtained at the Sorbonne. A. brilliant B. lovely C. fascinating D. happy Từ trái nghĩa Tạm dịch: Sau cái chết bi thương của ông Pierre Currie vào năm 1906, bà không chỉ đảm nhận trách nhiệm dạy dỗ hai con mà còn kế tục sự nghiệp của chồng ở Sorbonne. => tragic /'trædʒɪk/ (a): bi kịch, bi thương brilliant /’briljənt/ (a): tài giỏi, lỗi lạc lovely /'lʌvli/ (a): đẹp đẽ, xinh xắn fascinating /'fæsineitiη/ (a): hấp dẫn, lôi cuốn, quyến rũ happy (a): hạnh phúc => Đáp án D (tragic >< happy
  • 26. Question26: It was not until the sun was shining brightly that the little girl woke up. A. No sooner was the sun shining brightly than the little girl woke up. B. Not until the little girl woke up was the sun shining brightly. C. Not until the sun was shining brightly did the little girl wake up. D. As soon as the little girl woke up, the sun hasn’t shone brightly yet. Đảo ngữ Not until +s+ V2/ed did+S+V0 Dịch nghĩa: Mãi đến khi mặt trời chiếu sáng rực rỡ, cô bé mới tỉnh dậy. Xét các đáp án: A. No sooner was the sun shining brightly than the little girl woke up. → Mặt trời vừa chiếu sáng rực rỡ thì cô bé tỉnh dậy, sai nghĩa. B. Not until the little girl woke up was the sun shining brightly. → Mãi đến khi cô bé tỉnh dậy thì mặt trời mới chiếu sáng rực rỡ, sai nghĩa. C. Not until the sun was shining brightly did the little girl wake up. → Mãi đến khi mặt trời chiếu sáng rực rỡ, cô bé mới tỉnh dậy, đúng nghĩa. D. As soon as the little girl woke up, the sun hasn’t shone brightly yet. → Ngaykhi cô bé tỉnh dậy, mặt trời đã tỏa sáng rực rỡ, sai nghĩa.
  • 27. Question27: You must read the instructions. You won’t know how to use this machine without reading them. A. Unless you read the instructions, you won’t know how to use this machine. B. You will know how to use this machine unless you read the instructions. C. Without reading the instructions, the use of this machine won’t be known. D. Reading the instructions, so you will know how to use this machine. Kiến th ức: Câu điều kiệ n loại 1 Giải thích: Câu điều kiện loại 1 diễn tả sự việc có thể x ả y ra ở hiện tại hoặc tương lai. Cấu trúc: If/ Unless + S + V (thì hiện tại đơn), S + will/ can + V Tạm dị ch: Bạn phải đọc hướng dẫn. Bạn sẽ không biết làm thế nào để sử d ụng máy này mà không đọc chúng. A. Nếu bạn không đọc các hướng dẫn, b ạn sẽ không biết làm thế nào để sử dụng máy này. => đúng B. Bạn sẽ biết cách sử dụ ng máy này nếu bạn khô ng đọc hướng dẫn. => sai nghĩa C. Nếu không đọc hướng dẫn, việc sử dụng máy này s ẽ không được biết. => sai nghĩa D. Đọc các hướng d ẫn, vì vậ y b ạn sẽ biết cách sử dụng máy này. => sai nghĩa
  • 28. Question 28. I visit Mexico and the United States last year. A. visit B. Mexico C. United States D. and Trong câu có trạng ngữ last year là thì quá khứ đơn, do vậy động từ phải chia “ visited” Tạm dịch. Tôi đã đến thăm Mexico và Hoa Kỳ vào năm ngoái. Tạm dịch. Mẹ của Minh thích gặp trực tiếp bạn bè của bà ấy hơn là gọi điện thoại.
  • 29. Question 29: We are having her house painted by a construction company. A B C D Question 29: Đáp án B Nhận biết lỗi sai : Kiến thức về sự hoà hợp giữa chủ ngữ và tính từ sở hữu cách Phải sửa: her → our
  • 30. Question 30: She is becoming increasingly forgetable. Last night she locked herself out of the house twice. Nhận biết lỗi sai : Kiến thức về dùng từ đúng ngữ cảnh Forgetable ( a) có thể quên thay bằng Forgetful (a) :Hay quên
  • 31. Question 31: It started to rain at 4 o’clock and it is still raining. A. It has been raining at 4 o’clock. B. It has been raining since 4 o’clock C. It has been raining for 4 o’clock. D. It has been raining in 4 o’clock. Question 31: B Đề: Trời đã bắt đầu mưa lúc 2 giờ và vẫn đang mưa. Viết lại câu dùng thì HTHTTD. Dùng “since” trước một mốc thời gian → chọn B Dịch: Trời mưa suốt từ 4 giờ.
  • 32. Question 32: “Mum, please don’t tell Dad my mistake!” the girl said. A. The girl insisted her mother not tell her father his mistake. B. The girl told her mother not to mention her mistake any more. C. The girl asked her mother not to tell her father her mistake. D. The girl wanted her mother to keep her mistake in her heart. Question 32: C Kiến thức về cảu trực tiếp Ask Sb to + V/ not + to +V +st: đề nghị ai đó làm gì
  • 33. Question 33: You are able to go out with your friend this evening A. You musn’t go out with your friend this evening. B. You should go out with your friend this evening. C. You needn’t go out with your friend this evening D. You can go out with your friend this evening. Question 33: D Kiến thức modal verb : To be able to + V = Can + V + St Musn’t + V + St = are not allowed to + V Should + V + St: Needn’t + V + St = don’t/ doesn’t have + V +St
  • 34. Question 34. Not (34) _____________ body recognises the benefits of new developments in communications technology. A. nobody B. others C. every D. all Everybody
  • 35. Question 35. ...So widespread has texting become, however, that even pigeons have started doing it.____________, in this case, it’s difficult to view the results as anything but positive: A. Therefore B. What’s more C.whereas D. That is Xét về nghĩa thì “What’s more” phù hợp. A. Therefore (adv): vì vậy,do đó B. What’s more (adv): Quan trọng hơn C. Whereas (conj): trong khi đó D. That is - mệnh đề này thiếu, không có nghĩa. Tuy nhiên nhắn tin trở nên quá phổ biến đến nỗi thậm chí những con bồ câu đang bắt đầu nhắn tin. ____________, trong trường hợp này, rất khó để xem xét các kết quả không có gì khác là tích cực. Cần dùng một trạng từ nối câu vì chỗ trống nằm giữa một dấu chấm và một dấu phấy.
  • 36. Question 36 ...each____________with sensor equipment and a mobile phone A. armed: được trang bị (to be armed with sth) B. loaded: được nạp, được chất lên (to be loaded with sth) C. granted: được trợ cấp,được ban cho D. stocked:dự trữ, chất kho mỗi____________với thiết bị cảm ứng và một chiếc điện thoại di động. Dùng mệnh đề phân từ nối hai câu khác chủ ngữ. Trong trường hợp này cả 4 phương án đều là phân từ 2 nên chỉ cần xét nghĩa. Đáp án A phù hợp.
  • 37. Question 37 ….The readings made by the sensors will be automatically converted into text messages and beamed to the Internet____________ - they will appear on a dedicated “pigeon blog A. when (adv): khi mà B. which (pron): cái mà C. where (adv): nơi mà D. what (pron): cái mà ”: Những số liệu thu từ máy cảm ứng sẽ tự động đổi thành tin nhắn và phát vào Internet ______________chúng sẽ xuất hiện ở mục nhật kí bồ câu. Câu sau chỗ trống không thiếu thành phần câu nào nên chỉ có thể thêm một trạng ngữ bắt đầu mệnh đề quan hệ. Chọn where vì Internet là không gian mạng, không dùng để chỉ thời gian.
  • 38. Question 38 Each bird will carry these a miniature backpack, (37) _____________ , that is, from the camera, which will hang around its neck. A. instead B. except C. apart D. besides A. instead of: thay vào đó B. except for: ngoại trừ C. apart from: ngoài ra/ bên cạnh D. besides: bên cạnh đó Giới từ đề bài sẵn cho là “from” → chọn C và tạm dịch câu thành: Mỗi chú chim sẽ mang những dụng cụ này trong một chiếc balo siêu nhỏ được đeo ở cổ bên cạnh chiếc camera.
  • 39. Question39. what is the main topic of this passage? A. The benefits of solar and wind power over conventional energy sources. B. Two types of alternative energy sources that should be further utilized. C. How energy resources are tapped from nature. D. Examples of the use of energy sources worldwide. Câu hỏi: Nội dung/ Chủ để chính của bài là gì? A. Những ích lợi vượt trội hơn của năng lượng mặttrời và năng lượng gió so với các nguồn năng lượng truyền thống. B. Hai nguồn năng lượng sạch nên được khai thác nhiều hơn nữa. C. Cách mà những nguồn năng lượng được lấy ra từ tự nhiên. D. Các ví dụ về cách sử dụng các nguồn năng lượng trên khắp thế giới. Nhìn vào cấu trúc bài viết, nội dung bài xoay quanh việc phân tích ích lợi khi sử dụng 2 nguồn năng lượng sạch và cách thức khai thác chúng. Đáp án B. Không chọn A do trong bài không nêu rõ việc so sánh này. Không chọn C do đây chỉ là một ý của bài mà thôi. Không chọn D do bài chỉ nêu ra ít ví dụ về khai thác năng lượng mặt trời và địa nhiệt. Chú ý: Với câu hỏi chủ đề chính của bài, đáp án đúng phải nếu được nội dung xuyên suốt của toàn bài, các phương án chỉ nêu một ý nhỏ hay một ý nào đó xuất hiện ở từng đoạn đơn lẻ không phải là đáp án đúng. Có thể để lại câu hỏi này làm sau cùng, khi đã đọc bài nhiều lần và hiểu rõ nội dung của bài.
  • 40. Question 40. which of the following words could best replace the word “abundant”? A. a lot B. scarce C. little D. enough Câu hỏi: Từ nào đồng nghĩa/ có thể thay thế tốt nhất từ “abundant”? A. a lot: nhiều B. scarce: hiếm C. little: quá it D. enough: đủ Đáp án A. abundant: dồi dào, phong phú = a lot: nhiều. Chú ý: đây là câu hỏi từ vựng, nếu biết nghĩa của từ đang được hỏi sẽ rất thuận lợi, nhưng dù có biết hay không thì luôn phải đặt từ cần tìm vào câu và ngữ cảnh của nó để dùng thông tin luận ra nghĩa. “Solar energy, which reaches the earth through sunlight, is so abundant that it could meet the needs of worldwide energy consumption 6,000 times over.” Năng lượng mặt trời, nguồn năng lượng mà tới trái đất thông qua ánh nắng, là nguồn năng lượng cực dồi dào đến mức mà nó có thể đáp ứng được nhu cầu sử dụng năng lượng của toàn thế giới gấp tận 6000 lần. Tips: Cũng có thể thay lần lượt từng phương án lên thay cho từ cần tìm nghĩa để phán đoán.
  • 41. Question41. The word “it” in the second paragraph refers to_____________ A. solar energy B. the earth C. sunlight D. energy consumption Question41. Đáp án A Câu hỏi. Từ “it” nằm ở đoạn 2 thay thế cho từ nào? Các phương án khác. A. năng lượng mặt trời B. trái đất C. ánh nắng D. mức tiêu thụ năng lượng. Đây là câu hỏi suy luận về đại từ dùng để thay thế cho danh từ đã xuất hiện phía trước nhưng không muốn nhắc lại danh từ đó. Ta xem lại câu chứa từ “it”: Solar energy, which reaches the earth through sunlight, is so abundant that it could meet the needs of worldwide energy consumption 6,000 times over. Năng lượng mặt trời, nguồn năng lượng mà tới trái đất thông qua ánh nắng, là nguồn năng lượng cực dồi dào đến mức mà nó có thể đáp ứng được nhu cầu sử dụng năng lượng của toàn thế giới gấp tận 6000 lần. “Nó” ở đây là năng lượng mặt trời,
  • 42. Question 42. According to the passage, how is solar energy production similar to geothermal energy production? A. They both require the use of a generator. B. They both use heat from the earth’s surface. C. They both require fairly simple technology. D. They are both conventional and costly. Câu hỏi: Theo như bài viết, cách tạo ra năng lượng mặt trời giống với cách tạo ra năng lượng địa nhiệt như thế nào? A. Chúng đều cần sử dụng máy phát điện. B. Chúng đều sử dụng nhiệt từ bề mặt trái đất. C. Chúng đều cần công nghệ khá đơn giản D. Chúng đều là năng lượng thông thường và khá đắt đỏ. Thông tin chúng ta tìm ở đoạn 3 dòng 5 và 6. “... and as with solar power, the technology needed to utilize geothermal energy is fairly simple.”: và cũng như năng lượng mặt trời, công nghệ cần có để khai thác năng lượng địa nhiệt là khá đơn giản, phù hợp nhất với đáp án C. Các phương án khác không đúng theo như ý của bài.
  • 43. Question43. what best describes the author’s purpose in writing the passage? A. To warn people about the hazards of fossil fuel use. B. To convince people of the benefits of developing alternative energy sources. C. To describe the advantages and disadvantages of alternative energy use. D. To outline the problems and solutions connected with global warming. Câu hỏi. Câu nào miêu tả mục đích của tác giả trong bài viếtnày? A. Để cảnh báo con người về mối nguy hại của việc sử dụng năng lượng hóa thạch. B. Để thuyết phục con người về những lợi ích của các nguồn năng lượng thay thế đang phát triển. C. Để nói vể ưu điểm và nhược điểm của việc sử dụng nguồn năng lượng thay thế. D. Để kể ra các vấn đề và các giải pháp liên quan đến việc nóng lên của toàn cầu. Rõ ràng ta thấy bài văn tập trung nói về ích lợi của 2 nguồn năng lượng sạch vừa dồi dào, bất tận vừa thân thiện môi trường. Phần kết của bài tác giả cũng khẳng định lại những ưu điểm của năng lượng sạch vượt trội hơn so với năng lượng thông thường và nhấn mạnh chúng ta cần đầu tư hơn nữa vào sự phát triển những nguồn năng lượng này trên toàn cầu → Đáp án B.
  • 44. Question 44. what does the passage mainly discuss? A. How leadership differs in small and large groups B. The role of leaders in social groups C. The problems faced by leaders D. How social groups determine who will lead them Câu hỏi: Bài viết tập trung thảo luận vấn đề gì? A. Cách thức mà sự lãnh đạo khác nhau trong các nhóm nhỏ và nhóm lớn. B. Vai trò của người lãnh đạo trong các nhóm xã hội. C. Các vấn đề mà các nhà lãnh đạo phải đối mặt. D. Cách thức mà các nhóm xã hội tìm ra ai là người lãnh đạo họ. Nội dung cả bài xoay quanh thảo luận về người lãnh đạo trong nhóm, vai trò và cách thức lãnh đạo. Đáp án đúng là B. Loại A và D vì chỉ là ý nhỏ của đoạn 1; loại C do không đề cập nhiều trong bài
  • 45. Question45. The passage mentions all of the following ways by which people can become leaders EXCEPT___________ A. recruitment B. specific leadership training C. traditional cultural patterns D. formal election process Câu hỏi: Bài viết đề cập tất cả các cách sau để người ta có thể trở thành người lãnh đạo TRỪ____________ A. tuyển dụng B. sự huấn luyện lãnh đạo riêng biệt C. kiểu mẫu văn hóa truyền thống D. tiến trình bầu cử trang trọng Đây là câu hỏi có chứa từ phủ định. Thông tin tìm ở đoạn 1, dùng phương pháp loại trừ, cách thức nào không được nhắc trong bài sẽ là đáp án chọn. Loại các phương án A, C, D do chúng lần lượt xuất hiện trong các câu: In the family, traditional cultural patterns (phương án C) confer leadership on one or both of the parents. In larger groups, leaders are usually chosen formally through election (phương án D) or recruitment (phương án A). Trong gia đình, cách thức lãnh đạo truyền thống chọn lãnh đạo có thể là bố, mẹ hoặc cả hai. Trong các nhóm lớn hơn, những người lãnh đạo thường được chọn một cách chính thống thông qua bầu cử hoặc tuyển dụng.
  • 46. Question 46. which of the followingstatements about leadershipcan be inferredfrom paragraph 2? A. person can best learn how to be an effective leader by studying research on leadership. B. Most people desire to be leadersbut can produce little evidence of their qualifications. C. A person who is an effective leader of a particulargroup may not be an effective leader in another group. D. Few people succeed in sharing a leadershiprole with another person. Câu hỏi: Phát biểu nào dưới đây về sự lãnh đạo mà có thể được suy ra từ đoạn 2: A. Một người có thể học làm người lãnh đạo hiệu quả tốt nhất bằng việc học các nghiên cứu về lĩnh vực lãnh đạo. B. Hầu hết mọi người đều khát khao trở thành lãnh đạo nhưng có quá ít bằng chứng về năng lực. C. Một người có thể là lãnh đạo tốt của nhóm này nhưng chưa chắc đã là lãnh đạo hiệu quả ở một nhóm khác. D. Có quá ít người thành công trong việc chia sẻ sự lãnh đạo với một người khác. Đoạn 2 nhấn mạnh không có những phẩm chất chung cho tất cả các nhà lãnhđạo của các nhóm “there is no set of personal qualitiesthat all leaders have in common”; Mỗi nhóm khác nhau lại có yêu cầu riêng và người lãnh đạo của nhóm ấy phải hội tụ đủ các phẩm chất riêng biệt ấy để dẫn dắt nhóm của mình “any person may be recognized as a leader if the person has qualitiesthat meet the needs of that particular group” Xâu chuỗi các ý lại ta có thể suy ra phương án C là phù hợp nhất với thông tin ở đoạn 2.
  • 47. Question47. The passage indicates that instrumental leaders generally focus on___________ A. sharing responsibility with group members B. achieving a goal C. ensuring harmonious relationships D. identifying new leaders Câu hỏi: Bài viết chỉ ra rằng người lãnh đạo “instrumental leaders” thường tập trung vào_____________ A. Chia sẻ trách nhiệm với các thành viên khác trong nhóm. B. Đạt được mục tiêu. C. Đảm bảo một mối quan hệ hòa thuận giữa các thành viên trong nhóm. D. Xác định ra những người lãnh đạo mới. Thông tin có thể được tìm thấy rải rác ở đoạn 3 và 4 nhưng có thể thấy rõ ở đoạn 4 những dòng đầu “Instrumental leaders are likely to have a rather secondary relationship to other group members. They give orders and maydiscipline group members who inhibit attainment of the group’s goals.” Người lãnh đạo “instrumental leaders” có tình cảm xa cách hơn với các thành viên khác trong nhóm. Họ đưa ra mệnh lệnh và nhiều kỉ luật với các thành viên để có thể đạt được mục tiêu chung của nhóm. → Chọn đáp án B
  • 48. Question 48. The word “collective” in paragraph 3 is closest in meaning to___________ A. necessary B. group C. particular D. typical Câu hỏi: Từ “collective” ở đoạn 3 gần nghĩa nhất với từ nào? A. cần thiết B. nhóm C. cụ thể D. điển hình Câu hỏi từ vựng: đoán nghĩa của từ, ta đặt từ vào văn cảnh và câu chứa nó để đoán nghĩa. Expressive leadership, on the other hand, is leadership that emphasizes the collective well-being of a social group’s member. Người lãnh đạo kiểu “Expressive” mặt khác lại tập trung vào sự hạnh phúc của mỗi thành viên trong cả nhóm → Chọn đáp án B
  • 49. Question49. It can be understoodthat___________ A. There is lots of tension and conflict in an election of a leader in the family. B. There is usually an election to choose leaders in a family as well as in larger groups. C. It has been said that there must be a set of personal qualities that all leaders have in common. D. Leaders are sometimes chosen formally or informally. Câu hỏi: Có thế hiếu rằng_____________ A. Có rất nhiều căng thẳng và mâu thuẫn trong sự lựa chọn người lãnh đạo trong gia đình. B. Thường có sự tuyển chọn người lãnh đạo trong gia đình cũng như trong các nhóm lớn hơn. C. Người ta thường nói rằng phải có một tiêu chuẩn các phẩm chất năng lực chung của những người lãnh đạo. D. Người lãnh đạo được chọn ra một cách trang trọng hoặc không trang trọng. Đáp án D là đúng vì chúng ta có 2 cách để chọn ra người lãnh đạo, trang trọng ở các nhóm, tổ chức lớn; không trang trọng ở gia đình và nhóm bạn bè. A. Bài không nhắc tới. B. Sai, do gia đình không cần thông qua “election: tuyển chọn” người lãnh đạo. C. Đoạn 2 khẳng định không có tiêu chuẩn chung cho mọi nhà lãnh đạo. Dùng phương pháp loại trừ ta chọn được đáp án D.
  • 50. Question 50. The word “resolve” in paragraph 4 is closest in meaning to___________ A. talk about B. find a solution for C. avoid repeating D. avoid thinking about Câu hỏi: Từ “resolve” ở đoạn 4 đồng nghĩa với từ nào? A. Nói về B. Tìm ra cách giải quyết cho C. Tránh sự lặp lại D. Tránh nghĩ về Câu hỏi về từ gần nghĩa nhất, đặt vào câu văn chứa từ resolve ta có. They offer sympathywhen someone experiences difficulties or is subjected to discipline, are quick to lighten a serious moment with humor, and try toresolve issues that threaten to divide the group. Họ thường có sự thông cảm với những thành viên đang gặp khó khăn hoặc những người đang phải kỉ luật, họ nhanh chóng làm giảm nhẹ đi những tình huống căng thẳng với sự hài hước và cố gắng tìm cách giải quyết cho những vấn đề đe dọa gây chia rẽ nhóm. → Chọn đáp án B
  • 51. Question 1: As a ______ and mature student, Marie Curie earned a Physics degree with flying colors. A. brilliant B. brilliantly C. brilliantnessD. brilliance A. brilliant /’briliənt/ (a): xuất sắc, tài ba B. brilliantly /’briliəntli/ (a): một cách xuất sắc, một cách lỗi lạc D. brilliance /’briliəns/ (n): sự xuất sắc, sự tài ba => Căn cứ vào mạo từ “a” vào liên từ “and” vì vậy dựa vào cấu trúc song song thì vị trí còn trống cần sử dụng một tính từ. Ta loại được phương án B, C, D. Dịch nghĩa: Là một học sinh xuất sắc và chín chắn, Marie Curie đã nhận được tấm bằng vật lý với thành tích tuyệt vời.
  • 52. Question 2: How beautiful the queen is, ________? A. isn’t she B. is she C. is it D. isn’t it Dịch nghĩa: “Nữ hoàng thật lộng lẫy, phải không?” Với câu cảm thán, lấy danh từ trong câu đổi thành đại từ, phần hỏi đuôi dùng is/ am/ are tùy vào chủ ngữ.
  • 53. Question 3. We know that we are at fault for our third consecutive defeat, so there is no need to________salt into the wound. A. spread B. rub C. apply D. put Chúng ta có thành ngữ cố định rub salt into the wound có nghĩa là “xát muối vào vết thương” nên phương án phù hợp nhất là B. Dịch nghĩa: Chúng tôi biết rằng chúng tôi có lỗi trong thất bại thứ ba liên tiếp của chúng ta, vì vậy không cần phải xát muối vào vết thương đâu.
  • 54. Question 4 .Students are________ less pressure as a result of changes in testing procedures. A. under B. above C. upon D. out of Sau vị trí cần điền là cụm danh từ less pressure (ít áp lực hơn) nên ta cần một giới từ với nghĩa ở dưới để diễn tả ý chịu áp lực. Trong bốn phương án, phương án C là giới từ upon nghĩa là ở trên và phương án D là giới từ out of có nghĩa là ra khỏi nên không phù hợp. Phương án A và B đều mang nghĩa là ở dưới, tuy nhiên under được dùng với nghĩa rộng hơn below và đặc biệt ià đi với các từ ngữ trừu tượng.
  • 55. Question 5: It is reported that the building was completely________by fire. A. spoilt B. ruined C. damaged D. destroyed Đáp án D A. spoilt: làm hỏng phá hỏng ( dùng như ruin) B. ruined : làm hỏng, phá hủy giá trị ( giá trị, cơ hội thú vị…) C. damaged: làm hư hỏng ( thường nhà của do thiên tai…) D. destroyed : phá hủy, tàn phá ( làm hư hại phá hủy đến không dùng được nữa…) Trong câu có completely:hoàn toàn nên ta dùng đáp án D.
  • 56. Question6: ____ ASEAN is becoming more integrated, investors should be aware of local preferences and cultural sensitivities. A. Because B. However C. Despite D. Although Dịch nghĩa: Mặc dù ASEAN đang trở nên hội nhập hơn, các nhà đầu tư nên nhận thức được các ưu tiên địa phương và sự nhạy cảm về văn hóa. Xét các đáp án: A. Because → Bởi vì ASEAN đang trở nên hội nhập hơn, các nhà đầu tư nên nhận thức được các ưu tiên địa phương và sự nhạy cảm về văn hóa. (không hợp lý) B. However → Tuy nhiên, ASEAN đang trở nên hội nhập hơn, các nhà đầu tư nên nhận thức được các ưu tiên địa phương và sự nhạy cảm về văn hóa. (không hợp lý) C. Despite → Liên từ chỉ sự đối lập, sau Despite (mặc dù) = In spite of + noun/noun phrase/V_ing (DT/cụm DT/V_ing), không phải clause (mệnh đề) D. Although → Mệnh đề chỉ sự nhượng bộ, các liên từ được dịch là
  • 57. Question 7: It is really a (n)_____________ which is suitable for my daughter. A. undergraduate interesting economicscourse B. economicscourse interesting undergraduate C. interesting economicsundergraduate course D. interesting undergraduate economics course *Theo quy tắt trật tự tính từ trong câu: OSASCOMP interesting - O; economics - P Và tính từ luôn đứng trước danh từ. *Note: undergraduate course (phr.n): khóa học đại học Tạm dịch: Đó thực sự là một khóa học đại học phù hợp cho con gái tôi.
  • 58. Question 8: A man was arrested for _________________ the White House in November 2017. A. backing down B. breaking into C. bringing off D. bargaining for A. backing down: rút lại ý kiến, thừa nhận mình sai/thất bại B. breaking into: đột nhập C. bringing off = succeed in doing something difficult: giải quyết việc khó D. bargaining for/on st = expect or be prepared for something: mong đợi, chờ đón Dịch nghĩa: Một người đàn ông đã bị bắt vì đột nhập vào Nhà Trắng vào tháng 11 năm 2017.
  • 59. Question 9: While my father ________________ a film on TV, my mother was cooking dinner. A. watched B. was watching C. had watched D. watches Đáp án B – Căn cứ vào đây là mệnh đề trạng ngữ chỉ thời gian: While S + V (quá khứ tiếp diễn), S + V (quá khứ tiếp diễn). (Diễn tả hai hành động xảy ra song song cùng lúc trong quá khứ). Vế không chứa “while” chia thì quá khứ tiếp diễn => Vế còn lại chia thì quá khứ tiếp diễn. – Dịch: Trong khi bố của tôi đang xem phim trên TV, mẹ của tôi đang nấu ăn.
  • 60. Question 10: Mary will have worked at this school for 25 years ________. A. by the time she retires B. when she retired C. as soon as she had retired D. after she had retired Kiến thức: Mệnh đề chỉ thời gian / Sự hòa hợp thì trong mệnh đề Giải thích: Mệnh đề chính chia thì tương lai => động từ trong mệnh đề chỉ thời gian chia thì hiện tại (hiện tại đơn, hiện tại hoàn thành). Loại luôn được các phương án B, C, D do sai thì. by the time: cho đến khi when: khi as soon as: ngay khi after: sau khi Tạm dịch: Mary sẽ đã làm việc được ở ngôi trường này được 25 năm cho đến khi bà ấy nghỉ hưu. Chọn A.
  • 61. Question 11: After the visit to that special school, we friends with some studentswith reading disabilities. A. acquainted B. had C. made D. realised Kiến thức về cụm từ cố định Make friends with sb: kết bạn với ai đó Tạm dịch: Sau khi tới thăm ngôi trường dành cho học sinh khuyết tật, tôi đã kết bạn được với một số bạn có vấn đề về kĩ năng đọc. Các cấu trúc cần lưu ý khác: Be acquainted with sb: quen với ai Make acquaintance: 1àm quen
  • 62. Question 12. In a formal interview, it is essential to maintain good eye___________with the interviewers. A. link B. connection C. touch D. contact Ta có cụm từ “eye contact”: sự giao tiếp bằng mắt A. link (n): mối liên lạc B. connection (n): sự liên quan, mối quan hệ C. touch (n): sự chạm, sờ D. contact (n): sự giao tiếp, sự tiếp xúc Tạm dịch: Trong một cuộc phỏng vấn trang trọng, cần duy trì giao tiếp tốt bằng mắt với người phỏng vấn.
  • 63. Question 13. Only half of the exercises ________ so far, but the rest will have been finished by Saturday. A. are being done B. were done C. have been done D. have done Đáp án C. dựa vào so far là dấu hiện dật biết thì hiện tại hoàn thành loiaj đáp án A,B Xét đến chủ ngữ “only half of the exercises”( sự việc )- trạng thái bị động Tạm dịch : Chỉ một nửa số bài tập đã được thực hiện cho đến nay, nhưng phần còn lại sẽ được hoàn thành vào thứ Bảy
  • 64. Question 14. ____________the homework, he was allowed to go out with his friends. A. Finishing B. Finish C. To finish D. Having finished Question 14. Đáp án D Kiến thức được hỏi: Rút gọn mệnh đề cùng chủ ngữ dùng Having P2. Khi hai mệnh đề cùng chủ ngữ, và câu muốn nhấn mạnh hành động phía trước được hoàn thành xong trước rồi hành động phía sau mới xảy ra thì chúng ta dùng công thức: Having +P2, S+Ved. Đáp án D. Tạm dịch: Sau khi hoàn thành xong bài tập về nhà, anh ấy được phép đi chơi với những người bạn của mình.
  • 65. Question 15. The country is rapidly losing its workers as____________people are emigrating. A. the most and the most B. the more and the more C. more and more D. most and most Question 15. Đáp án C Kiến thức được hỏi: so sánh kép More and more + N: càng ngày càng nhiều cái gì More and more + adj: càng ngày càng thế nào Trong các phương án duy nhất C đúng. Tạm dịch: Đất nước đang dần dần bị sụt giảm số lượng người lao động vì ngày càng nhiều người di cư.
  • 66. Question 16. “Good luck with the exam, Minh!” - “_______, Dad.” A. By no means B. I wish so C. Never mind D. Thank you HD: “Chúc may mắn với bài thi, Minh!”. “______________,Ba” A. Không có gì. B. Con hy vọng như vậy. C. Đừng bận tâm. D. Cảm ơn.
  • 67. Question 17. “Dad. I’ve passed the English test.” - “_______.” A. Good job! B. Good test!C. Good luck! D. Good way! HD: “Bố. Con vừa qua bài kiểm tra tiếng Anh.” A. Làm tốt lắm! B. Bài kiểm tra rất tốt! C. Chúc may mắn! D. Một cách tốt!
  • 68. Question 18. A. sleepy B. trophy C. facial D. exact Trọng âm đáp án D rơi vào âm tiết số hai, còn các phương án còn lại trọng âm rơi vào âm thứ nhất. A. sleepy /ˈsliːpi/ (adj): buồn ngủ B. trophy /ˈtrəʊfi/ /'troʊfi/ (n): chiếc cúp C. facial /ˈfeɪʃl/ (adj): thuộc về khuôn mặt D. exact /ɪɡˈzækt/ (adj): chính xác Lưu ý: Thường thì danh từ và tính từ có hai âm tiết trọng âm rơi hầu hết vào số một, tuy nhiên ở đây “exact” là trường hợp ngoại lệ, cần làm nhiều và tìm tòi bổ sung các trường hợp ngoại lệ khác nữa như các tính từ: polite, alone, alive danh từ: machine, mistake, police, idea...
  • 69. Question 19. A. authority B. necessity C. academic D. commercially Trọng âm đáp án C rơi vào âm tiết số 3, còn các phương án còn lại trọng âm rơi số hai. A. authority /ɔːˈθɒrəti/ (n): quyền lực, có thẩm quyền B. necessity /nəˈsesəti/ (n): sự cần thiết C. academic /ˌækəˈdemɪk/ (adj): có tính chất học thuật, (thuộc) học viện, (thuộc) trường đại học D. commercially /kəˈmɜːʃəli/ (adv): về phương diện thương mại Lưu ý: Đuôi ty, gy, cy, phy, fy thường là các duỗi trọng âm sẽ rơi âm tiết số ba tính từ dưới lên. Ta có đuôi “ate” cũng vậy. Đuôi “ic”,“cial”,“tial”,“tion”,“sion”, “cian” trọng âm rơi âm tiết ngay trước.
  • 70. A. workshop /ˈwɜːkʃɒp/ (n): hội thảo B. worm /wɜːm/ (n): sâu/ giun C. worry /ˈwʌri/ (n)/ (v): mối lo/ lo lắng D. wordless /ˈwɜːdləs/ (a): không nói nên lời * Mở rộng: worry có hai phiên âm, là / 'wʌri/ và /'wɜ:ri/. Tuy nhiên, trong trường hợp này, ba đáp án còn lại đều có phần gạch chân là âm /ɜ:/ nên từ worry cần có phần gạch chân là âm /ʌ/ để đảm bảo đáp án. Question 20. A. borrowed B. helped C. dismissed D. booked Question 21. A. workshop B. worm C. worry D. wordless A. borrowed /ˈbɒrəʊ/: mượn B. helped /helpt/: giúp đỡ C. dismissed /dɪsˈmɪst/: giải tán D. booked /bʊk/: đặt vé/ đặt trước
  • 71. Question 22. The football final has been postponed until next Sunday due to the heavy snowstorm. A. cancelled B. changed C. delayed D. continued Đáp án C đúng vì ta có postpone (v) có nghĩa là hoãn lại = delay (v). Đây là câu hỏi tìm từ đồng nghĩa nên chọn đáp án C A. cancelled: hủy bỏ B. changed: thay đổi C. delayed: hoãn, làm chậm trễ D. continued: tiếp tục Tạm dịch: Trận chung kết bóng đá bị hoãn đến chủ nhật tuần tới do bão tuyết lớn
  • 72. Question 23. The discovery of the new planet was regarded as a major breakthrough in astronomy. A. promised B. doubted C. considered D. refused Đáp án C đúng vì ta có: (to) regard st1 as st2 = (to) consider st1 st2: coi cái gì là cái gì. Đây là câu hỏi tìm từ đồng nghĩa nên ta chọn phương án C A. promised: hứa hẹn B. doubted: nghi ngờ C. considered: cân nhắc D. refused: từ chối Tạm dịch: Sự phát hiện ra hành tinh mới đó được coi như một bước đột phá lớn trong ngành thiên văn học.
  • 73. Question 24. The loss of his journals had caused him even more sorrow than his retirement from the military six years earlier. A. Grief B. joy C. comfort D. sympathy Dữ liệu của câu: Việc đánh mất nhật kí của mình khiến ông ấy sorrow hơn cả việc xuất ngũ 6 năm trước. sorrow (n): nỗi buồn (danh từ của từ sorry). grief (n): nỗi đau buồn joy (n): niềm vui comfort (n): sự thoải mái sympathy (n): sự đồng cảm sorrow (nỗi buồn) trái nghĩa với joy (niềm vui). Đáp án B. Tạm dịch: Việc đánh mất nhật kí của mình khiến ông ấy buồn hơn cả việc xuất ngũ 6 năm trước.
  • 74. Question 25. As a newspaper reporter, she always wanted to get information at first hand A. indirectly B. directly C. easily D. slowly Dữ liệu của bài: Là một phóng viên báo chí, cô ấy luôn muốn nhận được thống tin at first hand. Logic: Phóng viên thì muốn tiếp cận thông tin như thế nào? “At first hand”: trực tiếp • indirectly (adv): một cách gián tiếp • directly (adv): trực tiếp • easily (adv): dễ dàng • slowly (adv): chậm rãi → at first hand (trực tiếp) trái nghĩa với indirectly (một cách gián tiếp) Tạm dịch: Là một phóng viên báo chí, cô ấy luôn muốn nhận được thông tin một cách trực tiếp.
  • 75. Câu 26: I had just come home. Then I heard the phone ringing. A.No sooner had I come home then I heard the phone ringing. B.Scarcely had I come home when I heard the phone ringing. C.I had barely come home than I heard the phone ringing. D.Hardly barely come home did I hear the phone ringing. Câu 26. Chọn đáp án B Giải thích: Đáp án B: Tôi vừa vào nhà thì tôi nghe thấy tiếng chuông điện thoại reo. Đáp án A sai ở chữ “then” Đáp án C sai ở chữ “than” Đáp án D sai cấu trúc. *Cấu trúc câu đảo ngữ: No sooner + QKHT + than + QKĐ: vừa mới.... thì...... = Hardly/Barely/Scarcely + QKHT + when/before +QKĐ Dịch nghĩa: Ngay khi tôi vào nhà thì tôi nghe thấy tiếng chuông điện thoại reo.
  • 76. Question 27: You don’t try to work hard. You will fail in the exam. A. Unless you don’t try to work hard, you will fail in the exam. B. Unless you try to work hard, you won’t fail in the exam. C. Unless you try to work hard, you will fail in the exam. D. Unless do you try to work hard, you will fail in the exam. Dịch nghĩa. Bạn không cố gắng học chăm chỉ. Bạn sẽ trượt kỳ thi. Trừ phi bạn không cố gắng học chăm chỉ, bạn sẽ trượt kì thi. Trừ phi bạn cố gắng học chăm chỉ, bạn sẽ không trượt kì thi. Trừ phi bạn cố gắng học chăm chỉ, bạn sẽ trượt kì thi. Sai cấu trúc.
  • 77. Question 28: Exceeding speed limits and fail to wear safety belts are two common causes of automobile death. A. Exceeding B. fail C. safety belts D. causes Ta dùng danh động từ (V-ing) như 1 danh từ để làm chủ ngữ của câu Chữa lỗi: fall → falling Dịch: Vượt quá giới hạn tốc độ và không đeo dây an toàn là hai nguyên nhân phổ biến gây tử vong cho người lái ô tô.
  • 78. Question 29: It was suggested that John studies the material more thoroughly before attempting to pass the exam. A. studies B. more C. attempting D. to pass Cấu trúc: It was suggested that + S + V (nguyên thể): ai đó được gợi ý nên làm gì Chữa lỗi: studies → study Dịch: John được gợi ý nen học tài liệu ký lưỡng hơn trước khi cố gắng để đỗ được kỳ thi.
  • 79. Question 30: They are the shocking news in newspapers that is what people are talking about this morning. A. They are B. is C. are D. about 30: A Danh từ được đề cập đến trong câu là “news” – danh từ không đếm được → dùng như danh từ số ít Chữa lỗi: They are → It is Dịch: Đó là một tin gây sốc trên báo chí, là những gì mà mọi người đã bàn tán sáng nay
  • 80. Question 31. I haven't met my grandparents for five years. A.I often met my grandparents five years ago. B.I last met my grandparents five years ago. C.I have met my grandparents for five years. D.I didn't meet my grandparents five years ago. Tôi chưa gặp ông bà tôi các đây 5 năm rồi A.Tôi thường gặp ông bà của tôi 5 năm trước ( sai nghĩa) B.Lần cuối tôi gặp ông bà của tôi là vào 5 năm trước ( đúng) C.Tôi gặp ông bà của tôi trong 5 năm( sai nghĩa) D.Tôi đã không gặp ông bà của tôi 5 năm trước đây( sai nghĩa)
  • 81. Question 32. “I will help you with the housework”, Mai said. A. Mai promised to help me with the housework. B. Mai asked me to help her with the housework. C. Mai begged to help me with the housework. D. Mai insisted on helping me with the housework. Mai “ Tôi sẽ giúp bạn làm công việc nhà” A.Mai hứa giúp tôi làm công việc nhà B.May yêu cầu tôi giúp cô ấy làm công việc nhà C.Mai cầu xin giúp tôi cùng làm công việc nhà D.Mai khăng khăng giúp tôi làm công việc nhà
  • 82. Question 33. There was no need for him to have left the house in such weather. A.He shouldn’t leave the house in such weather. B.He didn’t have to leave the house in such weather. C.He didn’t need to leave the house in such weather. D.He needn’t have left the house in such weather. There’s no need for somebody to do something:không cần thiết phải làm gì. There was no need for somebody to have done something = Needn’t have done something: đáng lẽ ra không cần thiết phải làm gì, nhưmg thực tế đã làm điều đó trong quả khứ. Đáp án D Tạm dịch: Lẽ ra bạn đã không cần phải rời nhà trong thời tiết như vậy
  • 83. Question 34: The first thing that is included in the "living together” (34) ______ is the expected good relations with your family. This also involves sharing equally the housework. A. custom B. tradition C. notion D. trend Kiến thức về từ vựng A. custom/ˈkʌs.təm/ (n): phong tục, tập quán B. tradition /trəˈdɪʃn/ (n): truyền thống C. notion /ˈnoʊʃn/ (n): quan niệm, niềm tin, ý tưởng hoặc sự hiểu biết về cái gì D. trend /trend/ (n): xu hướng, phương hướng Tạm dịch: “The first thing that is included in the “living together” (1)___________ is the expected good relations with your family.” (Điều đầu tiên được bao gồm trong quan niệm “sống chung” là niềm mong đợi về các mối quan hệ tốt đẹp trong gia đình bạn.)
  • 84. Question 35: . This also involves sharing equally the housework. (35) _______ of people think that everyone should share the housework equally, but in many homes parents do most of it. A. lots B. few C. little D. a lot of Kiến thức về lượng từ Tạm dịch: “Lots of people think that everyone should share the housework (2)________, but in many homes parents do most of it.” (Nhiều người nghĩ rằng mỗi thành viên nên chia sẻ việc nhà một cách đồng đều cho nhau, nhưng trong nhiều gia đình thì cha mẹ làm hầu hết mọi việc.)
  • 85. Question 36: (36)_________, housework'scontributions of the teenager make him more responsible. A. In addition B. However C. In contrast D. In case Kiến thức về liên từ A. In addition: thêm vào đó B. However: tuy nhiên C. In contrast: trái lại, trái với D. In case: phòng khi; trong trường hợp Tạm dịch: (Theo nhiều quan điểm nhất định, nhiều gia đình không thể chia sẻ việc nhà cho nhau trong khi lẽ ra họ nên làm vậy. Trên thực tế, chia sẻ việc nhà một cách đồng đều giữa các thành viên là điều dường như không thể bởi vì thời gian biểu của các gia đình. Do đó, theo cách nào đó người ta tin rằng con cái và cha mẹ phải cùng nhau làm mọi việc. Để làm được điều này họ có thể thiết lập một kế hoạch cho công việc nhà cửa. Thêm vào đó, những đóng góp trong việc nhà của thanh thiếu niên cũng làm cho họ trở nên có trách nhiệm hơn.) Vì hai đoạn văn nối với nhau có ý tưởng trợ, bổ sung lẫn nhau nên ta
  • 86. Question 37: According to researchers, teenagers should share the housework because (37) ______ will help them when they have to establish their own family in the future. A. which B. what C. that D. who Kiến thức về đại từ chỉ định Xét cấu trúc câu: (Theo các nhà nghiên cứu, thanh thiếu niên nên sẻ chia việc nhà cho nhau bởi vì điều đó sẽ giúp ích cho họ khi họ phải lập gia đình riêng cho mình trong tương lai.) Nhận thấy câu có ba vế có chủ vị rõ ràng được nối với nhau bởi hai liên từ là “because” và “when”. Do đó đây không phải dạng đại từ quan hệ nên loại A, D. Vị trí cần điền là một đại từ chỉ định để thay thế cho mệnh đề phía trước “teenagers should share the housework”. Người ta dùng đại từ chỉ định “that” để thay thế cho điều đã nói phía trước khi họ không muốn lặp lại câu đó bởi sẽ làm câu nói lủng củng, rườm rà.
  • 87. Question 38: It can be (38) ________ concluded that many parents don't really prepare their children for future, because they don't stimulate them to learn how to run a house. A. likely B. probably C. auspiciously D. possibly Kiến thức về từ vựng Xét các đáp án: A. likely /ˈlaɪkli/ (adv) = probably: có thể, có khả năng xảy ra hoặc trở thành hiện thực B. probably /ˈprɑːbəbli/ (adv): có thể, có khả năng xảyra hoặc trở thành hiện thực C. auspiciously /ɔːˈspɪʃəsli/ (adv): có khả năng thành công trong tương lai D. possibly /ˈpɒsəbli/ (adv): có thể (ở mức độ không chắc chắn) Người ta dùng “possibly” đi với “can (’t)/could(n’t)” trong câu để nhấn mạnh khả năng cái gì có khả năng/không có khả năng làm gì. (Có thể kết luận rằng nhiều cha mẹ không thực sự chuẩn bị cho tương lai con cái họ, bởi vì họ không khích lệ con cái mình học cách làm như nào để quán xuyến việc nhà.)
  • 88. Question 39: Which of the following is the best title for the passage? A. Common Complaints About Work B. Accomplishing Trivial Matters C. Achieving Job Satisfaction D. Learning to Manage Time Câu 39 : Đáp án D Learning to Manage Time = học cách quản lý thời gian
  • 89. Question 40: According to the passage, why do many people never seem to have enough time to accomplish things? A.They do not prioritize tasks. B. They get tied down by one difficult problem C. They fail to deal with trivial matters D. They do not seek the advice of time management experts A. They do not prioritise tasks. Thông tin ở đoạn 1: “People commonly complain that they never have enough time to accomplish tasks. The hours and minutes seem to slip away before many planned chores get done. According to time management experts, the main reason for this is that most people fail to set priorities about what to do first.” (Mọi người thường than phiền rằng họ chẳng bao giờ có đủ thời gian để hoàn thành các công việc. Hàng giờ và hàng phút dường như trôi qua đi trước khi các công việc đã được lên kế hoạch hoàn thiện. Theo các chuyên gia quản lý thời gian, nguyên nhân chính cho việc này chính là hầu hết mọi người không thể ưu tiên làm việc gì trước.)
  • 90. Question 41: In paragraph 2, the word those refers to . A. daily lists B. trivial matters C. priorities D. people Thông tin ở đoạn sau: “One simple solution... accomplished daily.” (Một giải pháp thường được những người đó sử dụng đầu tiên chính là để cho danh sách các công việc được hoàn thành hàng ngày.
  • 91. Question 42: The passage states that one solution to time management problems is to . A. consult a time management expert B. accomplish time - consuming matters first C. keep daily lists of priorities and check them regularly D. spend only a short time on each task C. keep daily lists of priorities and check them regularly Thông tin ở đoạn 2: “One simple solution... progress.” (Một giải pháp thường được những người đó sử dụng đầu tiên chính là để cho danh sách các công việc được hoàn thành hàng ngày. Những danh sách này sắp xếp các công việc từ cần thiết nhất đến ít cần thiêtết nhất và được kiểm tra thường xuyên qua từng ngày để tiến hành quá trình.)
  • 92. Question 43. The paragraph following the passage most probably discusses . A. mental and physical health problems B. another solution to time management problems C. ways to achieve a sense of fulfilment D. different types of lists Câu 43: Đáp án B another solution to time management problems = Một giải pháp khác để quản lý thời gian
  • 93. Question 44: What topic does the passage mainly discuss? A. The way how to write the resume for job application. B. The mistakes people make when applying for a job. C. The common way to make impression in a job interview. D. The necessary skills for job application. Đoạn văn chủ yếu thảo luận về chủ đề gì? A. Cách viết bản sơ yếu lý lịch để xin việc. B. Những lỗi mọi người thường gặp phải khi nộp đơn xin việc. C. Cách thông thường để tạo ấn tượng tốt trong một cuộc phỏng vấn xin việc. D. Những kĩ năng cần thiết để xin việc. Căn cứ vào thông tin đoạn 1: There are many mistakes that people make when writing their resume (CV) or completing a job application. Here are some of the most common and most serious. (Có nhiều lỗi mà mọi người thường gặp khi viết bản sơ yếu lí lịch hay hoàn thành hồ sơ xin việc. Dưới đây là một vài lỗi thường gặp và nghiêm trọng nhất.)
  • 94. Question 45: The word “executing” in paragraph 2 is closest in meaning to ________________. A. Enumerating B. determining C. completing D. implementing Từ “executing” trong đoạn 2 gần nghĩa nhất với từ __________. A. liệt kê B. xác định, tìm ra C. hoàn thành D. thực hiện, thi hành Từ đồng nghĩa: executing (thực hiện) = implementing They do not necessarily know the specific skills you used in executing them, nor do they know what results you achieved - both of which are essential. (Họ không cần thiết phải biết những kĩ năng cụ thể bạn đã sử dụng để thực hiện các nhiệm vụ đó, họ cũng không cần biết kết quả bạn đã đạt được là gì – cả hai thứ đó đều rất quan trọng).
  • 95. Question 46: The word “concrete” in paragraph 3 could be best replaced by ______________. A. indeterminate B. specific C. substantial D. important Từ “concrete” trong đoạn 3 được thay thế tốt nhất bởi từ _________. A. mơ hồ, không rõ B. cụ thể, rõ ràng C. chủ yếu, thiết yếu D. quan trọng Từ đồng nghĩa: concrete(cụ thể) = specific The more concrete information you can include, the better. (Thông tin bạn cung cấp càng cụ thể càng tốt).
  • 96. Question 47: What does the word “it” in paragraph 3 refer to? A. organization money B. Information C. productivity D. percentage Từ “it” trong đoạn 3 đề cập đến từ nào? A. tiền của tổ chức B. thông tin C. năng suất D. tỉ lệ phần tram Căn cứ thông tin đoạn 3: If any innovations you introduced saved the organization money, how much did they save? If you found a way of increasing productivity, by what percentage did you increase it? (Nếu bạn đã đưa ra được ý tưởng đổi mới nào giúp tiết kiệm được tiền cho tổ chức, thì họ đã tiết kiệm được bao nhiêu tiền? Nếu bạn đề xuất được 1 cách tăng năng suất, vậy bạn đã làm tăng nó lên bao nhiêu phần trăm?).
  • 97. Question 48: According to the passage, what information should candidates include in their resume? A. specific skills for previous jobs B. the past achievements C. previous positions D. future objective Theo đoạn văn, thông tin nào những người xin việc nên bao hàm trong bản sơ yếu lí lịch của họ? A. các kĩ năng cụ thể cho các công việc trước đây B. các thành tựu đạt được trong quá khứ C. các chức vụ, vị trí công việc trước đây D. mục tiêu trong tương lai Từ khóa: information/ includein their resume Căn cứ các thông tin trong đoạn văn: The biggest problem is perhapslisting the duties for which you were responsible in a past position:all this tells yourpotentialemployersis what you were supposed to do. They do not necessarily know the specific skillsyou used in executing them, nor do they know what results you achieved - both of which are essential. (Vấn đề lớn nhất có lẽ là liệt kê các công việc mà bạn đã làm trong chức vụ trước đây: tất cả những điều này nói cho nhà tuyển dụng tiềm năng của bạn biết những gì bạn có thể làm được. Họ không cần thiết phải biết những kĩ năng cụ thể bạn đã sử dụng để thực hiện các nhiệm vụ đó, họ cũng không cần biết kết quả bạn đã đạt được là gì – cả hai thứ đó đều rất quan trọng). Writing what you are trying to achieve in life - your objective - is a waste of space. (Viết về những điều bạn đang cố gắng đạt được trong cuộc sống – mục tiêu của bạn – là một sự lãng
  • 98. Question 49: Why did the author mention that applicants should write a good brief career summary? A. To make the employers interested in what they want. B. To make the interviewers more curious about you. C. Because the employers do not care for what you want to achieve. D. Because it can provide their specific skills in their positions. Câu 49: Đáp án B Tại sao tác giả lại đề cập trong đoạn văn rằng những người nộp đơn xin việc nên viết một bản tóm tắt sự nghiệp hay mà ngắn gọn? A. Để khiến nhà tuyển dụng quan tâm đến những gì họ muốn. B. Để khiến những người phỏng vấn tò mò hơn về bạn. C. Bởi vì những người tuyển dụng không quan tâm bạn đã đạt được thành công gì. D. Bởi vì nó có thể cung cấp các kĩ năng cụ thể ở vị trí của họ. Từ khóa: applicants/ write a good brief career summary Căn cứ thông tin đoạn 4: Instead, use that space for a career summary. A good one is brief - three to four sentences long. A good one will make the person reviewing your application want to read further. (Thay vào đó, hãy sử dụng chỗ giấy đó để viết bản tóm tắt về sự nghiệp. Một bản tóm tắt phù hợp là tầm 3 đến 4 câu ngắn gọn. Một bản tóm tắt hay sẽ khiến cho người đọc hồ sơ xin việc của bạn muốn đọc nhiều hơn nữa).
  • 99. Question 50: According to the passage, which of the following is NOT true? A. The abilityto negotiateeffectively is as significantas technicalskills. B. Candidatesmust study the job they are applying carefullybefore writing the CV. C. Applicantsshould not apply for a distinct job from what they are doing. D. The information interviewees present should be relatedto the job they are applying. Theo đoạn văn, câu nào sau đây là không đúng? A. Khả năng đàm phán hiệu quả cũng quan trọng như các kĩ năng thực hành. B. Người xin việc cần phải nghiên cứu công việc họ đang nộp đơn xin một cách kĩ càng trước khi viết bản sơ yếu lí lịch. C. Người xin việc không nên nộp đơn xin một công việc khác với công việc mà họ đang làm. D. Những thông tin mà người đi phỏng vấn trình bày nên có liên quan đến công việc mà họ đang xin. Từ khóa: not true Căn cứ vào các thông tin trong đoạn văn: However, yourabilityto negotiate effectively, for example, can be just as important as your technicalskills. (Tuy nhiên, khả năng đàm phán hiệu quả của bạn cũng quan trọng như các kĩ năng thực hành.) All information you give should be relevant, so carefully consider the job for which you are applying.If you are applying for a job that is somewhat different than your current job, it is up to you to draw a connectionfor the resume reviewer, so that they will understandhow your skills will fit in their organization.(Tất cả các thông tin bạn cung cấp nên có mối liên quan với nhau, vì vậy hãy nghiên cứu công việc bạn định xin một cách kĩ càng. Nếu bạn đang xin một công việc, về mặt nào đó, khác với công việc hiện tại của bạn, bạn nên tạo sự liên kết giữa 2 công việc để người đọc bản sơ yếu lý lịch hiểu được các kĩ năng của bạn phù hợp với tổ chức của họ).
  • 100. Question 1. Susan has achieved great in her career thanks to her hard work. A. successfully. B. successful. C. succeed. D. success Ta thấy phía trước vị trí trống là tân ngữ của câu và câu trước nó là một tính từ nên chỉ có thể điền một danh từ. A. successfully.(adv) B. successful.(adj) C. succeed.(v) D. success (n) Tạm dịch: Susan đã đạt được thành công lớn trong sự nghiệp của cô ấy nhờ sự chăm chỉ làm việc của mình.
  • 101. Question 2: You’ve been punished of bad marks, _________? A. have you B. haven’t you C. do you D. don’t you Dịch nghĩa: “Bạn chưa bao giờ bị phạt vì điểm thấp phải không?” Câu bắt đầu với you have thì thành lập hỏi đuôi sẽ dùng haven’t you.
  • 102. Question 3: Hearing about people who mistreat animals makes me go hot under the . A. shoes B. collar C. T-shirt D. vest Kiến thức về thành ngữ Ta có thành ngữ: “to be hot under the collar”: điên tiết, cáu tiết Tạm dịch: Nghe về những người đối xử không tốt với động vật khiến tôi điên tiết.
  • 103. Question 4: The sign warns people ______ the dangers of swimming in this river. A. about B. from C. with D. to Dịch nghĩa: Biển báo cảnh báo mọi người về sự nguy hiểm của việc bơi lội trên dòng sông này. * Căn cứ vào cấu trúc: - warn sb about sth: cảnh báo ai về cái gì.
  • 104. Question 5: Can the sales team meet its financial________? A. purposes B. aimsC. goals D. objectives A. purposes: Lý do làm gì hay cái gì tồn tại B. aims: mục tiêu, đích nhắm (cái đã lên kế hoạch, hạnh động với hy là đạt được) C. goals: mục tiêu ( “dùng aim” và thường dùng với “ achieve goals” D. objectives: mục tiêu, chủ đích cái đề ra để đạt được hay làm được) Tạm dịch: đội ngũ bán hàng có thể đáp ứng các mục tiêu tài chính đã đề ra hay không?
  • 105. Question 6: The residents of the village are living a happy life _______ they lack modern facilities. A. despite B. although C. therefore D. because of Dịch nghĩa: Dân làng đang sống một cuộc sống hạnh phúc mặc dù họ thiếu các trang thiết bị tiện nghi hiện đại. Xét các đáp án: A. despite = in spite of + N/Ving: mặc dù B. although + clause: mặc dù C. therefore: do đó, do vậy D. because of + N/Ving: bởi vì => Theo nghĩa cần một liên từ mang nghĩa “mặc dù” và sau nó là một mệnh đề, do đó chỉ có B thỏa mãn
  • 106. Question 7: – “Can I help you, sir?” – “I’m looking for a ___________ desk for my son.” A. wooden round fashionable B. round fashionable wooden C. wooden fashionable round D. fashionable round wooden *Theo quy tắc trật tự tính từ trong câu: OSASCOMP : fashionable - O; round - Shape; wooden - M Dịch: -‘’Tôi có thể giúp gì, thưa ông?’‘ – ’‘Tôi đang tìm một chiếc bàn tròn hợp thời trang bằng gỗ cho con trai,’’
  • 107. Question 8: The United States __________ some 150,000 military reservists when the war broke out. A. came about B. caught on C. called up D. carried out A. came about: xảy đến, xảy ra B. caught on: trở nên phố biến C. called up: gọi điện, gọi đi lính/nghĩa vụ quân sự D. carried out: tiến hành Dịch nghĩa: Mỹ đã gọi đi lính khoảng 150.000 người lính dự bị khi chiến tranh nổ ra.
  • 108. Question 9: We______for going home when it began to rain. A. were preparing B. are preparing C. will be preparing D. have prepared – Căn cứ vào đây là mệnh đề trạng ngữ chỉ thời gian: When S + V (quá khứ đơn), S + V (quá khứ tiếp diễn). Vế chứa “when” chia thì quá khứ đơn => Vế còn lại chia thì quá khứ tiếp diễn. – Dịch: Chúng tôi đang chuẩn bị về nhà thì trời mưa.
  • 109. Question 10. _______, he will get ready to go to work. A. When he will have breakfast tomorrow B. Until he would have breakfast tomorrow C. After he has breakfast tomorrow D. As soon as he had breakfast tomorrow -Căn cứ mệnh đề chính S+ will+Vo và các đáp án bên dưới ta sẽ kết hợp với câu có S+V( hiện tại đơn) Vậy ta loại đáp án A,B, D sai thì. Tạm dịch : Sau khi anh ấy ăn sáng vào ngày mai anh ấy sẽ đi làm
  • 110. Question 11: You should more attention to what your teacher explains. A. make B. pay C. get D. set Kiến thức về cụm từ cố định Pay attention to = take notice of: chú ý Tạm dịch: Bạn nên chú ý tới điều giáo viên giải thích.
  • 111. Question 12: I'm not going to go ice skating! I'd only fall over and a fool of myself. A. create B. show C.do D. make Kiến thức về cụm từ cố định make a fool (out) of sb/yourself: khiến ai đó trông như kẻ ngốc Tạm dịch: Tôi sẽ không đi trượt băng đâu! Tôi chỉ bị ngã và khiến mình trông như kẻ ngốc.
  • 112. Question 13. French ________ at their schools as a compulsory lesson for 20 years until last year. A. is being taught B. was being taught C. had been taught D. has been taught Kiến thức: Câu bị động Giải thích: Dấu hiệu : last year, thì quá khứ =>loại A, D, for 20 years (thì quá khứ hoàn thành) => loại B. Công thức: S + had been + Ved/V3. Tạm dịch: Cho đến năm ngoái, Tieesng Pháp đã được dạy như là môn học bắt buộc ở các trường của họ được 20 năm .
  • 113. Question 14. ________ all the papers already, Sarah put them back in the file. A. To have photocopied B. To photocopy C. Photocopying D. Having photocopied Kiến thức: Mệnh đề phân tử / Rút gọn mệnh đề đồng ngữ Giải thích: Khi 2 mệnh đề có cùng chủ ngữ (Sarah) thì có thể rút gọn 1 trong 2 mệnh đề về dạng: - V-ing / Having P2: nếu mệnh đề được rút gọn mang nghĩa chủ động - P2 (quá khứ phân từ): nếu mệnh đề được rút gọn mang nghĩa bị động Chủ ngữ “she” có thể làm chủ (tự thực hiện hành động “photocopied=> nghĩa chủ động. đã xảy ra hành động nên ta dùng Having +V.P2
  • 114. Question 15: The more she practices, _____________ she becomes. A. the greater confidence B. more confidently C. the more confident D. the most confident Dịch nghĩa: Cô càng luyện tập, cô càng trở nên tự tin. Xét các đáp án: A. the greater confidence → Vì cấu trúc become + adj nên confidence sai B. more confidently → thiếu mạo từ the và sai từ confidently (sau become + adj) C. the more confident → Cấu trúc so sánh kép khi nói về 2 người hoặc sự vật: The more/-er + S + V, the more/-er + S + V D. the most confident → Sai cấu trúc so sánh kép
  • 115. Question 16: “Take care! Have a safe trip back!” - “_______.” A. Thanks for coming. B. Sounds good. C. Thanks, bye. D. Good luck next time. Tình huống: “Giữ gìn sức khỏe nhé! Chúc đi an toàn nhé!” A. Cảm ơn vì đã tới. B. Nghe hay đấy. C. Cảm ơn, tạm biệt. D. Chúc may mắn lần sau. Question 17: “What a meaningful thing you've done for your less fortunate friends, my son!” - “_______” A. Thanks for taking care of me, Mom. B. What a wonderful gift you've made for me. C. I don’t really care. I must go now. D. Thank you, Mom. That’s just a bit I can do for them. HD: “Thật là một việc có ý nghĩa mà con đã làm cho những người bạn kém may mắn của mình, con trai!” A. Cảm ơn vì đã chăm sóc con, mẹ B. Món quà mẹ tặng con thật tuyệt vời! C. Con thật sự không quan tâm. Con phải đi bây giờ. D. Cảm ơn, mẹ. Đó là chỉ một chút con có thể làm cho họ.
  • 116. Kiến thức về trọng âm với từ hai âm tiết A.teacher /‘ti:t∫ə(r)/: từ này trọng âm rơi vào âm tiết thứ nhất. Vì theo quy tắc trọng âm không bao giờ rơi vào âm /ə/ và trọng âm ưu tiên rơi vào nguyên âm dài. B.lesson /'lesn/: Từ này trọng âm rơi vào âm tiết thứ nhất. Vì theo quy tắc nếu các âm mà ngắn hết thì trọng âm rơi vào âm tiết đầu. C.action / 'æk∫n/ : Từ này trọng âm rơi vào âm tiết thứ nhất. Vì theo quy tắc có đuôi -ion thì trọng âm rơi vào trước âm đó. D.police /pə'li:s/ : Từ này trọng âm rơi vào âm tiết thứ hai. Vì theo quy tắc trọng âm không bao giờ rơi vào âm /ə/ và trọng âm ưu tiên rơi vào nguyên âm dài. => Phần D trọng âm ớ âm tiết thứ 2, các từ còn lại trọng âm rơi vào âm tiết thứ nhất Question 18 A. teacher B. lesson C. action D. police
  • 117. Question 19. A. Importance B. happiness C. employment D. relation Kiến thức trọng âm với từ 3 âm tiết và hậu tố A.importance /ɪm’pɔ:tns/:Từ này trọng âm rơi vào âm tiết thứ hai. Vì theo quy tắc trọng âm ưu tiên rơi vao nguyên âm dài. B.happiness /'hæpinəs/: Từ này trọng âm rơi vào âm tiết thứ nhất. Vì theo quy tắc nếu các âm mà ngắn hết thì trọng âm rơi vào âm tiết đầu. C.employment /ɪm’pɔɪmənt/: Từ này trọng âm rơi vào âm tiết thứ hai. Vì theo quy tắc trọng âm không rơi vào đuôi -ment và ưu tiên rơi vào nguyên âm dài. D.relation /rɪ'leɪ∫n/: Từ này trọng âm rơi vào âm tiết thứ hai. Vì theo quy tắc có đuôi -ion thì trọng âm rơi vào trước âm đó. => Phần B trọng âm rơi vào âm tiết thứ nhất, các từ còn lại trọng âm ở âm tiết thứ hai.
  • 118. Question 20. A. played B. planned C. cooked D. lived Cách phát âm của đuôi -ed Phần gạch chân của phần C phát âm là /t/, của các từ còn lại là /d/. Kiến thức cách phát âm của nguyên âm Phần gạch chân của phần D phát âm là /æ/, của các từ còn lại phát âm là /ei/. A. date /deɪt/ B. face /feɪs/ C. page /peɪdʒ/ D. map /mæp/ Question 21. A. date B. face C. page D. map
  • 119. Question 22: Whenever problems come up, they discuss them frankly and find solutions quickly. A. honestly B. constantly C. loyally D. unselfishly Từ đồng nghĩa honestly /’ɔnistli/ (adv): chân thật, thẳng thắn constantly /’kɔnstəntli/(adv): liên miên, kiên định loyally /’lɔiəli/ (adv): trung thành unselfishly /ʌn’selfi∫(ə)li/ (adv): không ích kỉ, không màng đến lợi ích cá nhân Tạm dịch: Bất cứ khi nào có vấn đề gì xảy ra họ đều trao đổi thẳng thắn và nhanh chóng tìm ra hướng giải quyết. Þ Đáp án A (frankly = honestly)
  • 120. Question 23: Unselfishnessis the very essence of friendship. A. necessary part B. important part C. difficult part D. interesting part Câu 23: Đáp án B Từ đồng nghĩa A. necessary part: phần cần thiết B. important part: phần quan trọng C. difficult part: phần khó khăn D. interesting part: phần thú vị Tạm dịch: Tính không ích kỉ là phần hết sức quan trọng của tình bạn. Þ Đáp án B (essence = important part)
  • 121. Question24: Peter is the black sleepof the family. He is currently serving 10 years in jail for stealing a car. A.a member of a family who is regarded as a disgrace and an embarrassment. B.a member of a family who supports family by raising sheep. C. a member of a family who confers prestige on his family. D. a breadwinner. Từ trái nghĩa a member of a family who is regardedas a disgrace and an embarrassment (một thành viên trong một gia đình, người mà được coi như một điều ô nhục và đáng hổ thẹn) a member a family who supports family by raising sheep (một thành viên trong một gia đình, người mà nuôi sống gia đình bằng việc nuôi cừu) a member of a family who confers prestigeon his family (một thành viên trong một gia đình người mà mang lại thanh thế cho gia đình mình) a breadwinner (một trụ cột gia đình) Tạm dịch: Peter là một đứa con phá gia chi tử trong gia đình. Nó hiện tại đang thụ án 10 năm tù vì tội ăn trộm xe hơi. Þ Đáp án C (the black sheep of the family >< a member of a family who confers prestige on his family)
  • 122. Question 25: There must be a mutual trust between friends. A. reliance B. belief C. defendant D. suspicion Từ trái nghĩa reliance /ri’laiəns/ (n): sự tin cậy, sự tín nhiệm belief /bi’li:f/ (n): lòng tin, sự tin tưởng defendant /di’fendənt/ (n): bị cáo suspicion /səs’pi∫n/ (n): sự nghi ngờ Tạm dịch: Cần phải có sự tin tưởng lẫn nhau giữa những người bạn. Þ Đáp án D (trust >< suspicion)
  • 123. Question26: He got down to writing the letter as soon as he returned from his walk. A. No sooner had he returned from his walk when he got down to writing the letter. B. Not until he returned from his walk did he get down to writing the letter. C. Only after he had returned from his walk did he get down to writing the letter. D. Hardly had he returned from his walk when he got down to writing the letter. Dịch nghĩa: Anh ta ngồi xuống viết thư ngay khi đi bộ về. Xét các đáp án: A. No sooner had he returned from his walk when he got down to writing the letter. → Cấu trúc đảo ngữ với các cụm từ so sánh về thời gian: No sooner + had + S + P2 + than + S + Ved (nếu dùng no sooner phải có than đằng sau). B. Not until he returned from his walk did he get down to writing the letter. → Mãi cho đến khi anh trở về sau khi đi bộ, anh mới xuống viết thư, sai nghĩa. C. Only after he had returned from his walk did he get down to writing the letter. Chỉ sau khi anh ấy trở về sau khi đi bộ, anh ấy mới viết thư, sai nghĩa D. Hardly had he returned from his walk when he got down to writing the letter. → Cấu trúc đảo ngữ với các cụm từ so sánh về thời gian: Hardly/Scarcely/Barely + had + S + P2 + when + S + Ved: ngay khi/vừa mới….thì….
  • 124. Question 27. I can’t give up smoking but I’d like to. A.I wish I can give up smoking. B.I wish I could give up smoking. C. I wish I would give up smoking. D. I wish I could not give up smoking. Đáp án. B Giải thích: Vì tình huống ở thì hiện tại nên cần một điều ước cho hiện tại có cấu trúc S + wish + V (past simple). Dạng quá khứ của động từ can là could. Dịch nghĩa: Tôi ước tôi có thể bỏ hút thuốc.
  • 125. Question 28: The ocean probably distinguishes the earth from other planets of the solar system, for scientistsbelieve that large bodies of water are not existing on the other planets. A. from B. probably C. for D. are not existing Kiến thức về thì động từ Do câu này ám chỉ tới một sự thật nên động từ "exist" không chia dạng tiếp diễn (tobe V-ing) mà nên chia về hiện tại đơn. => Đáp án D (are not existing => don’t exist) Tạm dịch: Đại dương là dấu hiệu đặc trưng của Trái Đất khi so sánh với những hành tinh khác trong hệ mặt trời, bởi các nhà khoa học tin rằng các khối nước lớn không tồn tại ở những hành tinh khác.
  • 126. Question 29: The school officials are considering a comprehensive planning to alleviate the problem of overcrowding in the dormitories. A. are B. planning C. alleviate D. overcrowding Câu 29: Đáp án B Kiến thức về từ vựng Vị trí này ta cần một danh từ, vì phía trước có mạo từ “a” và tính từ “comprehensive” Þ Đáp án B (planning ® plan) Tạm dịch: Các cán bộ trong trường học đang xem xét một kế hoạch toàn diện để làm giảm bớt vấn đề quá tải trong ký túc xá.
  • 127. Question 30. The dog stood up slowly, wagged it tail, blinked its eyes, and barked. A B C D Kiến thức: sự hoà hợp giữa chủ ngữ và tính từ sở hữ cách Giải thích: chủ ngữ là số ít chỉ vật → sở hữu cách cũng phải là số ít chỉ vật Sửa: it → its Tạm dịch: Chú chó con đứng lên một cách chậm rãi, vẫy đuôi, nheo mắt và sủa.
  • 128. Question31: I really believe my letter came as a great surprise to John. A. John maybe very surprised to receive my letter. B. John mighthave been very surprised to receive my letter. C. John must be very surprised to receive my letter. D. John musthave been very surprised to receive my letter. Đề bài: “Tôi tin chắc rằng bức thư của tôi là một sự bất ngờ lớn đối với John.” A. John có thể rất bất ngờ khi nhận được thư của tôi. (May + V diễn tả một sự việc có khả năng xảy ra ở hiện tại nhưng không chắc chắn. Câu này sai vì hành động “came as a great surprise to John” là hành động đã xảy ra rồi.) B. John có thể rất bất ngờ khi nhận được thư của tôi (Might have Vp2: diễn tả sự việc có thể đã xảy ra trong quá khứ nhưng không chắc chắn. Câu này sai vì vế trước đã có “really believe” nó đã là căn cứ để thể hiện rằng phán đoán này là chắc chắn và có cơ sở) C. John chắc chắn sẽ rất bất ngờ khi nhận được thư của tôi. (Must + V: diễn tả một dự đoán chắc chắn sẽ xảy ra ở hiện tại. Câu này sai vì sự việc ở câu trên đã diễn ra trong quá khứ rồi.) D. John chắc chắn sẽ rất bất ngờ khi nhận được thư của tôi (Must have Vp2: diễn tả sự việc chắc chắn xảy ra trong quá khứ.) => Đúng, vì “really believe” đã thể hiện tính chắc chắn trong phán đoán của John.
  • 129. Question 32: He said: “I haven’t met her since she left school.” A. He said that he hadn’t met her since he had left school. B. He said that he hadn’t met her since he left school. C. He said that he hadn’t met her since she left school. D. He said that he hadn’t met her since she had left school. Đề bài: Anh ấy nói: “Tôi không gặp cô ấy kể từ khi cô ấy ra trường.” A. Anh ấy nói rằng anh ấy đã không gặp cô ấy kể từ khi anh ấy ra trường. (Không giống câu trên về nghĩa) B. Anh ấy nói rằng anh ấy đã không gặp cô ấy kể từ khi anh ấy ra trường. (Không giống câu trên về nghĩa) C. Anh ấy nói rằng anh ấy đã không gặp cô ấy kể từ khi cô ấy ra trường. (Đúng, theo quy tắc chuyển câu trực tiếp sang câu gián tiếp thì khi có mệnh đề chỉ thời gian thì không cần lùi, do đó chỉ lùi vế trước của câu, vế từ “since” trở đi không cần lùi) D. Anh ấy nói rằng anh ấy đã không gặp cô ấy kể từ khi cô ấy ra trường. (Sai vì ở mệnh đề với “since”, thì quá khứ ở câu trực tiếp vẫn giữ nguyên khi chuyển sang câu gián tiếp).
  • 130. Question 33. The last time he came back to his hometown was 5 years ago. A. It's 5 years since he last lived in his hometown. B. He didn't come back to his hometown 5 years ago. C. He started coming back to his hometown 5 years ago. D. He hasn’t come back to his hometown for 4 years. Dùng cấu trúc: S + have/ has + not + Ved/3………for/ since + time = The last time + S + Ved/2……was + time (years, month,..) + ago in + time (May, 1990,…) S + last + Ved/2 …… time + ago/ in + time
  • 131. Question 34. Most educational specialists believe that early schooling should provide children with a(n) (34) _______ of their own abilities and the self-confidence to use their abilities. A. attitude B. awareness C. knowledge D. attention Kiến thức : Đọc điền từ - câu hỏi từ vựng Giải thích: awareness (n) sự nhận thức Các đáp án còn lại không hợp về nghĩa: Attitude (n): thái độ knowledge (n) kiến thức attention (n) sự chú ý Most educational specialists believe that early schooling should provide children with a(n) awareness of their own abilities and the self-confidence to use their abilities. Tạm dịch: Hầu hết các chuyên gia giáo dục tin rằng việc đi học sớm nên cung cấp cho trẻ em một sự nhận thức về khả năng của mình và sự tự tin để sử dụng khả năng của mình.
  • 132. Question 35. One approach recognized by many (35) _______ as promoting these qualities is the Montessori method, first practised by Maria Montessori of Italy in the early 1900s. A. experts B. researchers C. teachers D. tutors Kiến thức : Đọc điền từ - câu hỏi từ vựng Giải thích: experts (n): chuyên gia Các đáp án còn lại không hợp về nghĩa. reseachers (n) nhà nghiên cứu teachers (n): giáo viênD. tutors (n): gia sư One approach recognized by many experts as promoting these qualities is the Montessori method, first practised by Maria Montessori of Italy in the early 1900s. Tạm dịch: Một phương pháp được nhiều chuyên gia công nhận để thúc đẩy những phẩm chất này là phương pháp Montessori, được thực hiện lần đầu iên bởi Maria Montessori của Ý vào đầu năm 1900.
  • 133. Question 36. Nancy McCormick Rambusch is credited with (36) _______ the method in the United States, (37) _______ today there are over 400 Montessorischools. A. popularly B. popularity C. popularizing D. popular Kiến thức : Đọc điền từ - từ loại Giải thích: Ở vị trí này cần một danh động từ (V-ing) vì sau giới từ nên chọn popularizing Popularly (adv): phổ biến popularity (n) sự phổ biến Popularize (v) làm cho phổ biến popular (adj) Tạm dịch: Nancy McComick Rambusch được tin tưởng trong việc làm phổ biến phương pháp này tại Hoa Kì, nơi hiện nay có hơn 400 trường năng khiếu.
  • 134. Question 37 Nancy McCormick Rambusch is credited with popularizing the method in the United States, (37)………….. today there are over 400 Montessori schools. A. which B. where C. x D. that Kiến thức : Đọc điền từ - đại từ quan hệ Giải thích: Ở vị trí này cần 1 đại từ quan hệ chỉ nơi chốn. Tạm dịch: Nancy McComick Rambusch được tin tưởng trong việc làm phổ biến phương pháp này tại Hoa Kì, nơi hiện nay có hơn 400 trường năng khiếu.
  • 135. Question 38 Through such exploration, children develop their sense of touch and learn how to do everyday tasks without adult assistance. (38) ______ benefits include improvement in language skills, and acquaintance with elements of science, music, and art. A. Other B. Others C. Another D. The others Kiến thức : Đọc điền từ - từ chỉ số lượng Giải thích: Other: (khác) dùng với danh từ số nhiều Others: = other + Noun (số nhiều) Another: dùng với danh từ số ít The others: những người kia/ những cái kia Do vậy chọn A (other) (29) Other benefits include improvement in language skills, and acquaintance with elements of science, music, and art Tạm dịch: Những lợi ích khác bao gồm cải thiện kĩ năng ngôn ngữ, và làm quen với các yếu tố khoa học, âm nhạc và nghệ thuật.
  • 136. Question39. what is the main idea of the passage? A. Wings are the most important physical attribute of birds. B. Different wing styles evolved for different types of flight. C. Birds have many specialized features that aid in their survival. D. Birds fly for many reasons. Dịch đề bài: Ý chính của đoạn văn này là gì? A. Cánh là đặc điểm thể chất quan trọng nhất của các loài chim. B. Các loại cánh chim khác nhau tiến hóa cho các kiểu bay khác nhau. C. Các loài chim có nhiều đặc điểm chuyên biệt để hỗ trợ sự sinh tồn của mình. D. Các loài chim bay bởi nhiều lý do. * Hướng dẫn làm bài: Câu đầu tiên của bài này là: “Birds have evolved many physical attributes that contribute to their flying ability.” Và câu cuối của bài này là: “Birds fly to find a prey, escape predators, and attract mates-in other words, to survive.” Như vậy, câu đầu tiên của bài nêu ra ý chính là về các đặc điểm thể chất được tiến hóa để phù hợp với việc bay. Các đoạn sau của bài nói sâu hơn về các đặc điểm đó. Câu cuối cùng của bài kết luận rằng đối với chim, bay là để tồn tại. Nghĩa là, các loài chim đã tiến hóa các đặc điểm thể chất để phù hợp với việc bay, và để tồn tại. Vậy nên, đáp án C đúng. Các đáp án còn lại chỉ nói về nội dung của một phần trong bài chứ không bao quát hết bài.
  • 137. Question40. According to the passage, what causes birds to rise when they start flying? A. Long wings with hollow feathers B. Higher air pressure below than above the wings C. Spreading out their tails D. Superior muscle strength Dịch đề bài: Theo như bài đọc, điều gì giúp các loài chim nâng được độ cao khi bắt đầu bay? A. Cánh dài với những chiếc lông rỗng ruột B. Áp suất không khí dưới cánh cao hơn ở trên cánh C. Trải hết đuôi của chúng ra D. Sức mạnh cơ bắp thượng đẳng * Hướng dẫn làm bài: Trích thông tin trong bài: “This creates higher pressure under the wings, called lift, which pushes the bird up” Tạm dịch: Điều này tạo ra áp suất cao hơn ở dưới cánh, được gọi là lực đẩy, chính lực đẩy này đẩy cho chim nâng độ cao. Như vậy, áp suất phía dưới cánh cao hơn trên cánh là thứ đẩy cho chim tăng độ cao.
  • 138. Question 41. The phrase “finches and sparrows” refers to_______. A. wings B. maneuvers C. ways of flying D. birds Dịch đề bài: Cụm từ “finches and sparrows” nói đến... A. cánh B. vận động/ diễn tập C. cách bay D. các loài chim Câu hỏi về từ vựng. “Finches and sparrows”: Các loài chim họ sẻ Thông và chim sẻ. Chọn đáp án D - birds: các loại chim - là gần nghĩa nhất với cụm từ bài cho. * Mở rộng: Đối với loại bài chọn từ gần nghĩa/ trái nghĩa, nếu không biết nghĩa của từ, có thể dựa vào dữ liệu xung quanh để đoán nghĩa của từ/ cụm từ bài cho. Ví dụ như câu hỏi này, bài đọc cho: “Finches and sparrows have short, broad wings”: Finches and sparrows có cánh ngắn và rộng. Vì bài đọc đang nói về cánh chim, nên có thể suy luận được “Finches and sparrows” là tên của hai loài chim (vì có cánh).
  • 139. Question42. According to the passage, what benefit comes from having built-in spoilers? A. an ability to fly faster B. a steeper angle of diving for prey C. prolonged flight D. superior maneuverability when climbing Dịch đề bài: Theo như bài đọc, lợi ích của việc có sẵn các cấu cản dòng là gì? (spoiler - cấu cản dòng - cấu cản không khí để phanh máy bay) A. khả năng bay nhanh hơn B. góc chúi xuống săn mồi đứng hơn C. các chuyến bay dài hơn D. tính cơ động cao hơn khi tăng độ cao Câu hỏi tìm thông tin. Đọc hướng dẫn làm bài ở câu 39. Từ khóa của câu hỏi là “spoiler”. Trích thông tin trong bài: “Faster birds, like hawks have built - in spoilers that reduce turbulence while flying. This allows a steeper angle of attack without stalling.” Như vậy, lợi ích của việc có sẵn cấu cản dòng là một góc đứng hơn khi tấn công con mồi. Vậy, chọn B là hợp lý.